HESI Health & Physical Assessment (Nursing Fundamentals)

Pataasin ang iyong marka sa homework at exams ngayon gamit ang Quizwiz!

Which complications does the nurse anticipate in the client who has blue-colored nail beds?

Cardiopulmonary Disease

A nurse assesses for hypocalcemia in a postoperative client. What is one of the initial signs that might be present? Headache Pallor Paresthesias Blurred vision

paresthesias

A nurse is caring for an elderly client with dementia who has developed dehydration as a result of vomiting and diarrhea. Which assessment best reflects the fluid balance of this client? Skin turgor Intake and output results Client's report about fluid intake Blood lab results

Blood lab results

Chronic Bronchitis

- inflammation of the lining of the bronchial tubes (cough/sputum)

A client is scheduled to receive phenytoin (Dilantin) 100 mg orally at 6 PM but is having difficulty swallowing capsules. What method should the nurse use to help the client take the medication? 1 Sprinkle the powder from the capsule into a cup of water. 2 Insert a rectal suppository containing 100 mg of phenytoin. 3 Contact the prescriber to determine if a change to a suspension form would be possible. 4 Obtain a change in the administration route to allow an intramuscular (IM) injection.

3 Contact the prescriber to determine if a change to a suspension form would be possible.

A patient is admitted to the hospital with severe dyspnea and wheezing. Arterial blood gas levels on admission are pH 7.26; PaCO2, 55 mm Hg; PaO2, 68 mm Hg; and image, 24. The nurse interprets these laboratory values to indicate: 1. Metabolic acidosis. 2. Metabolic alkalosis. 3. Respiratory acidosis. 4. Respiratory alkalosis.

3. Respiratory acidosis.

NormalRBC Count

4.7/6.1 M/ml

Normal WBC Count

4.8-10.8 K/ml

Biot's Respirations

Abnormal pattern of breathing characterized by groups of quick, shallow inspirations followed by regular or irregular periods of apnea.

Neutropenia

An abnormally low count of a type of white blood cell

A client is transferred to an acute care nursing unit after surgery. Which action of the nurse is most important and should be performed first?

Assess the patency of airway.

The nurse notes that his assigned patient has some blood clotting dysfunction. Which electrolyte could be responsible for this dysfunction? Potassium Magnesium Calcium Chloride

C. Calcium

Cardiopulmonary resuscitation

Initiated in a client w/ no pulse

Normal Hematocrit Percentages

Male: 41.5% - 50.4%, Female: 35.9% - 44.6% *The ratio of the volume of red blood cells to the volume of whole blood*

Low Pitched Crackles

May be present in pt w/ pneumonia

Needle thoracostomy

Performed in a client w/ absent breathing sounds

Which nursing intervention for opening the airway should be performed in an unconscious client with a spinal injury?

Performing Jaw Thrust Maneuver

The nurse is caring for a client with a platelet count of 50,000 cells per microliter. Which recommendation is inappropriate for the client?

Shaving with a straight blade razor.

An older adult client who complains of difficulty breathing after a surgery is found to have decreased vital capacity on spirometry. Which nursing intervention should be performed in this situation?

Teach the client coughing and deep breathing exercises

Providing oxygen via a nonrebreather mask is mainly performed when...

The client is conscious

Expectorant

lubricates the irritated respiratory tract, makes it easier to cough up secretions (example: guaifenesin).

Plantar warts

Viral

Verucca vulgaris

Viral

A client admitted to the hospital with a diagnosis of malabsorption syndrome exhibits signs of tetany. The nurse concludes that the tetany was precipitated by the inadequate absorption of which electrolyte? Sodium Calcium Potassium Phosphorus

calcium

7. What assessment does a nurse make before hanging an intravenous (IV) fluid that contains potassium? 1. Urine output 2. Arterial blood gases 3. Fullness of neck veins 4. Level of consciousness

1. Urine output

A client who had been receiving palliative care for cancer has deteriorated and now needs end-of-life care. The nurse identifies that which types of care will now be removed from the treatment plan? Select all that apply. Chemotherapy Repositioning Regular oral care Blood transfusion Radiation therapy

Chemotherapy Blood transfusion Radiation therapy

A nurse is caring for a client with pulmonary tuberculosis. What must the nurse determine before discontinuing airborne precautions?

Sputum is free of acid fast bacteria

Which action made by the client indicates that they are in the precontemplation stage of Transtheoretical Model of Change? 1 Refuses to think about changing 2 Intends to change in the next 60 days 3 Sustains the changed action for 6 months 4 Recognizes the advantages of the change

1 Refuses to think about changing

Which action of the nurse would be most important to convey interest in starting a conversation with a client who has hearing loss? 1 Smiling while seeing the client 2 Nodding head in front of the client 3 Making eye contact with the client 4 Leaning forward towards the client

3 Making eye contact with the client The nurse should make eye contact with the client to show interest in starting a conversation with a client with hearing loss. - Smiling while seeing the client would help to build a positive relationship. - Nodding in front of the client helps to regulate the conversation. - Leaning forward towards the client shows attention and awareness.

A registered nurse is examining the medical reports of different clients. Which client may need immediate assessment?

A client with acute asthma and 85% oxygen saturation

Pectus excavatum

A condition in which the breastbone sinks into the chest.

Sustained Fever Pattern

A constant body temperature continuously above 38° C (100.4° F) with little fluctuation

A client with hypertension receives a prescription for carteolol (Cartel) 7.5mg PO daily. The drug is available in 2.5 mg tablets. How many tablets should the nurse administer? (Enter numeric value only.)

Answer: 3 tablets Using D/H: 7.5mg / 2.5mg = 3 tablets

Which would the nurse consider to be a potential respiratory system-related complication of surgery?

Atelectasis

The nurse teaching a health awareness class identifies which situation as being the highest risk factor for the development of a deep vein thrombosis (DVT)? Multiple choice question Pregnancy Inactivity Aerobic exercise Tight clothing

Inactivity A DVT, or thrombus, may form as a result of venous stasis. It may lodge in a vein and can cause venous occlusion. Inactivity is a major cause of venous stasis leading to DVT. Pregnancy and tight clothing are also risk factors for DVT secondary to inactivity. Aerobic exercise is not a risk factor for DVT.

A client's breath has a sweet, fruity odor. Which condition is likely affecting this client? Multiple choice question Gum disease Uremic acidosis Diabetic acidosis Infection inside a cast

Diabetic acidosis A client with diabetic acidosis has a sweet, fruity odor to the breath. Gum disease is marked by halitosis. A stale urine smell indicates uremic acidosis. An infection inside a cast is accompanied by a musty odor of the casted body part.

Which diseases can be transmitted from client to client by droplet infection? Select all that apply.

Measles Pertussis Diphtheria

Apnea

Respirations cease for several seconds

Methemoglobinemia

a complication in which the mucous membranes appear blue in color due to increased deoxygenated blood in the body.

The nurse is teaching a group of student nurses about caring for a hospitalized immunosuppressed client. Which statement(s) made by the student nurse indicates the need for further teaching? Select all that apply.

"I should inspect the client's mouth at least every 4 hours."--should be every 8 not every 4 "I should monitor the vital signs of the client every 8 hours."--should be every 4 hrs not every 8 "I should ensure that that the client's room and bathroom are cleaned at least once a week."--should be cleaned at least once daily

Which physical assessment maneuver is the nurse performing when instructing the client to breathe in slowly and a little more deeply than normal through the mouth?

Auscultation

A client is admitted for dehydration, and an intravenous (IV) infusion of normal saline is started at 125 mL/hour. One hour later, the client begins screaming, "I can't breathe!" How should the nurse respond?

Elevate the head of the client's bed and obtain vital signs.

A client's arterial blood gas report indicates the pH is 7.52, PCO2 is 32 mm Hg, and HCO3 is 24 mEq/L. What does the nurse identify as a possible cause of these results? 1 Airway obstruction 2 Inadequate nutrition 3 Prolonged gastric suction 4 Excessive mechanical ventilation

Excessive mechanical ventilation The high pH and low carbon dioxide level are consistent with respiratory alkalosis, which can be caused by mechanical ventilation that is too aggressive. Airway obstruction causes carbon dioxide buildup, which leads to respiratory acidosis. Inadequate nutrition causes excess ketones, which can lead to metabolic acidosis. Prolonged gastric suction causes loss of hydrochloric acid, which can lead to metabolic alkalosis.

Relapsing Fever Pattern

Periods of febrile episodes coupled with periods of acceptable temperature values

A primary healthcare provider writes a prescription of "Restraints PRN" for a client who has a history of violent behavior. What is the nurse's responsibility in regard to this prescription?

Recognizing that PRN prescriptions are unacceptable for restraints

Hyperventilation

"over-breathing" - increased rate and/or depth - CO2 level low

A client becomes anxious after being scheduled for a colostomy. What is the most effective way for the nurse to help the client? 1 Administer the prescribed as needed (prn) sedative. 2 Encourage the client to express feelings. 3 Explain the postprocedure course of treatment. 4 Reassure the client that there are others with this problem.

2 Encourage the client to express feelings.

A nurse is transcribing a practitioner's orders for a group of clients. Which order should the nurse clarify with the practitioner? 1 Discharge in am 2 Blood glucose monitoring ac and bedtime 3 Erythropoietin (Procrit) 6000 units subcutaneously TIW 4 Dalteparin (Fragmin) 5000 international units Sub-Q BID

3 Erythropoietin (Procrit) 6000 units subcutaneously TIW

A client has a right-above-the-knee amputation after trauma sustained in a work-related accident. Upon awakening from surgery, the client states, "What happened to me? I don't remember a thing." What is the nurse's best response? 1 "Tell me what you think happened." 2 "You will remember more as you get better." 3 "You were in a work-related accident this morning." 4 "It was necessary to amputate your leg after the accident."

3 "You were in a work-related accident this morning."

For an individual experiencing an intracellular fluid deficit, an appropriate intervention would be : A. Restrict fluids B. Administer diuretics as ordered C. Observe for an increase in temperature D. Observe for dyspnea and shortness of breath

C. Observe for an increase in temperature

For which illness should airborne precautions be implemented?

Chickenpox

A nurse is assessing the integumentary system of four clients. Which client has the least chance of a false-positive result while undergoing assessment of capillary refill time?

Client with epilepsy - epilepsy does not cause circulatory inaccuracy

Atelectasis

Complete or partial collapse of a lung or a section (lobe) of a lung.

The nurse asks the client to shrug the shoulders and to turn the head against passive resistance. Which cranial nerve is involved in this action? Cranial nerve II Cranial nerve XI Cranial nerve VI Cranial nerve VII

Cranial nerve XI

A hospice nurse is caring for a dying client and the client's family members during the developing awareness stage of grief. What is the most important thing about the family that the nurse should assess before providing care? Cohesiveness Educational level Cultural background Socioeconomic status

Cultural background

A nurse is assessing the grief response of a family member whose relative has died. What must the nurse consider first about the family to conduct an effective assessment? Personality traits Educational level Socioeconomic class Cultural background

Cultural background

Which is an example of indirect contact transmission of microorganisms?

Dirty Hands

Intermittent Fever Pattern

Fever spikes are interspersed with normal temperature levels.

Candidiasis

Fungal infection

While assessing the muscle tone of a client, the client demonstrates a full range of muscle motion against gravity with some resistance. What score on the Lovett scale can be given to the client? Multiple choice question Fair (F) Good (G) Trace (T) Normal (N)

Good (G) According to the Lovett score, a full range of motion against gravity with some resistance can be categorized as G (good). F (fair) can be given if the client exhibits a full range of motion with no resistance. T (trace) score is given when the client exhibits slight contractility with no movement. N (normal) on the Lovett scale indicates full range of motion against gravity with full resistance.

While assessing a client, the nurse finds bluish coloration of the skin. The nurse finds that this discoloration is due to cyanosis. Which condition may be suspected? Multiple choice question Anemia Liver disease Heart disease Autoimmune disease

Heart disease A bluish discoloration of the skin indicates cyanosis. This condition may be caused by increased amounts of deoxygenated hemoglobin, which may lead in heart disease or lung disease. In clients with anemia, the skin has a pallor due to a reduced amount of oxyhemoglobin. In clients with liver disease, the skin appears yellow or orange due to increased deposits of bilirubin. In autoimmune diseases, the skin will lose its pigmentation.

The nurse is caring for clients in the pulmonary unit and suspects that one has tuberculosis. What is the priority nursing intervention in this situation?

Move the client to an airborne isolation room

A nurse is caring for a client diagnosed with methicillin-resistant Staphylococcus aureus (MRSA) in the urine. The healthcare provider orders an indwelling urinary catheter to be inserted. Which precaution should the nurse take during this procedure? 1 Droplet precautions 2 Reverse isolation 3 Surgical asepsis 4 Medical asepsis

Surgical asepsis Catheter insertion requires the procedure to be performed under sterile technique. Droplet precautions are used with certain respiratory illnesses. Reverse isolation is used with clients who may be immunocompromised. Medical asepsis involves clean technique/gloving

The registered nurse finds that a client cared for by a student nurse has developed an infection. Which action of the student nurse does the registered nurse suspect to be the cause of infection?

Use of a wet dressing

Orthopnea

difficulty breathing when supine (lying flat).

A client is admitted with severe diarrhea that resulted in hypokalemia. The nurse should monitor for what clinical manifestations of the electrolyte deficiency? Select all that apply. Diplopia Skin rash Leg cramps Tachycardia Muscle weakness

leg cramp muscle weakenss

5. A nurse assesses four patients. Which patient has greatest risk for hypomagnesemia? 1. A 72-year-old with chronic alcoholism 2. A 79-year-old with bone cancer 3. A 41-year-old with hypernatremia 4. A 46-year-old with respiratory acidosis

1. A 72-year-old with chronic alcoholism

A client's serum potassium level has increased to 5.8 mEq/L. What action should the nurse implement first? 1 Call the laboratory to repeat the test. 2 Take vital signs and notify the charge nurse or health care provider. 3 Inform the cardiac arrest team to place them on alert. 4 Take an electrocardiogram and have lidocaine available.

2 Take vital signs and notify the charge nurse or health care provider.

A client has a platelet count of 49,000/mL. The nurse should instruct the client to avoid which activity? 1 Ambulation 2 Blowing the nose 3 Visiting with children 4 The semi-Fowler's position

2 Patients with thrombocytopenia are at a greater risk of excessive bleeding in response to minimal trauma. The nurse should instruct the patient to avoid blowing their nose as this activity can increase the risk of bleeding. The following activities are not contraindicated with thrombocytopenia: ambulation, visiting with children, and semi-Fowler's position.

Which patients does a nurse plan to teach regarding water restriction? 1. A 23-year-old with extracellular fluid volume (ECV) deficit 2. A 34-year-old with hyponatremia 3. A 47-year-old with hypercalcemia 4. A 69-year-old with metabolic acidosis

2. A 34-year-old with hyponatremia

A nurse is evaluating the effectiveness of treatment for a client with excessive fluid volume. What clinical finding indicates that treatment has been successful? Clear breath sounds Positive pedal pulses Normal potassium level Decreased urine specific gravity

Clear breath sounds

A client is admitted to the intensive care unit in pulmonary edema. What should the nurse expect when performing the admission assessment? Weak rapid pulse Decreased blood pressure Radiating anterior chest pain Crackles at the base of each lung

Crackles at the base of each lung Crackles are the sound of air passing through fluid in the alveolar spaces. With pulmonary edema, fluid moves from the intravascular compartment into the alveoli. With hypervolemia, the pulse is bounding. The blood pressure is increased with hypervolemia. Radiating anterior chest pain will occur with angina or a myocardial infarction.

Which client assessment finding should the nurse document as subjective data? Multiple choice question Blood pressure 120/82 beats/min Pain rating of 5 Potassium 4.0 mEq Pulse oximetry reading of 96%

Pain rating of 5 Subjective data are obtained directly from a client. Subjective data are often recorded as direct quotations that reflect the client's feelings about a situation. Vital signs, laboratory results, and pulse oximetry are examples of objective data.

The nurse discovers several palpable elevated masses on a client's arms. Which term most accurately describes the assessment findings? Multiple choice question Erosions Macules Papules Vesicles

Papules The nurse discovers several palpable elevated masses on a client's arms. Which term most accurately describes the assessment findings?

A nurse is caring for a client who is having diarrhea. To prevent an adverse outcome, the nurse should most closely monitor what patient data or assessment finding? 1 Skin condition 2 Fluid and electrolyte balance 3 Food intake 4 Fluid intake and output

2 Fluid and electrolyte balance

A client is to receive a transfusion of packed red blood cells (PRBCs). The nurse should prepare for the transfusion by priming the blood IV tubing with which solution? 1 Lactated Ringer solution 2 5% dextrose and water 3 0.9% normal saline 4 0.45% normal saline

3 0/9% normal saline Blood and blood products for transfusion should be infused/diluted only with 0.9% normal saline solution. Solutions other than normal saline are incompatible and may cause RBC destruction by hemolysis.

Which client is at a high risk for a rise in blood pressure based on the given data? Client A- Age:20; HR: 70 beats/min; SV: normal Client B- Age: 30; HR: 90 beats/min; SV: decreased Client C- Age: 40; HR: 40 beats/min; SV: increased Client D- Age: 50; HR: 100 beats/min; SV: normal

Client C The blood pressure rises when the heart rate is decreased and the stroke volume is increased. In adults, the pulse rate should be between 60 and 100 beats/min. Client C's heart rate is 40 beats/min, which is less than normal, and the stroke volume is increased. Thus, client C has a high risk of high blood pressure.

A nurse is caring for a client who has been admitted with right-sided heart failure. The nurse notes that the client has dependent edema around the area of the feet and ankles. To characterize the severity of the edema, the nurse presses the medial malleolus area, noting an 8 mm depression after release. How should the edema be documented? Multiple choice question 1+ 2+ 3+ 4+

4+ Dependent edema around the area of feet and ankles often indicates right-sided heart failure or venous insufficiency. The nurse should assess for pitting edema by pressing firmly for several seconds, then releasing to assess for any depression left on the skin. The grading of 1+ to 4+ characterizes the severity of the edema. A grade of 4+ indicates an 8 mm depression. A grade of 1+ indicates a 2 mm depression. A grade of 2+ indicates a 4 mm depression. A grade of 3+ indicates a 6 mm depression.

Which change in data indicates to the nurse that the desired effect of the angiotensin II receptor antagonist valsartan (Diovan) has been achieved? A. Pulse rate reduced from 150 to 90 beats/minute. B. Dependent edema reduced from +3 to +1. C. Serum HDL increased from 35 to 55 mg/dl. D. Blood pressure reduced from 160/90 to 130/80.

Answer: Blood pressure reduced from 160/90 to 130/80 Diovan is an angiotensin receptor blocker, prescribed for the treatment of hypertension. The desired effect is a decrease in blood pressure (D). (A, B, and C) do not describe effects of Diovan.

A client is admitted to the hospital with a tentative diagnosis of infectious pulmonary tuberculosis. What infection control measures should the nurse take? 1 Don an N95 respirator mask before entering the room. 2 Put on a permeable gown each time before entering the room. 3 Implement contact precautions and post appropriate signage. 4 After finishing with patient care, remove the gown first and then remove the gloves.

1 Don an N95 respirator mask before entering the room.

A nurse is obtaining a health history from the newly admitted client who has chronic pain in the knee. What should the nurse include in the pain assessment? (Select all that apply.) 1 Pain history including location, intensity and quality of pain 2 Client's purposeful body movement in arranging the papers on the bedside table 3 Pain pattern including precipitating and alleviating factors 4 Vital signs such as increased blood pressure and heart rate 5 The client's family statement about increases in pain with ambulation

1 Pain history including location, intensity and quality of pain 3 Pain pattern including precipitating and alleviating factors

An older widow with lung cancer is now in the terminal stage of her illness. Her family is puzzled by her mood changes and apparent anger at them. The nurse explains to the family that the client is doing what? Trying to avoid her situation Coping with her impending death Attempting to reduce family dependence on her Hurting because the family will not take her home to die

Coping with her impending death

A child is to receive 60 mg of phenytoin (Dilantin). The medication is available as an oral suspension that contains 125 mg/5 mL. How many milliliters should the nurse administer? Record the answer using one decimal place. ______ mL

2.4

A nurse assesses drainage on a surgical dressing and documents the findings. Which documentation is most informative? 1 "Moderate amount of drainage." 2 "No change in drainage since yesterday." 3 "A 10-mm-diameter area of drainage at 1900 hours." 4 "Drainage is doubled in size since last dressing change."

3 "A 10-mm-diameter area of drainage at 1900 hours."

A nurse assesses for hypocalcemia in a postoperative client. One of the initial signs that might be present is: 1 Headache. 2 Pallor. 3 Paresthesias. 4 Blurred vision.

3 Paresthesias. Normally, calcium ions block the movement of sodium into cells. When calcium is low, this allows sodium to move freely into cells, creating increased excitability of the nervous system. Initial symptoms are paresthesias. This can lead to tetany if untreated. Headache, pallor, and blurred vision are not signs of hypocalcemia.

A client receiving intravenous vancomycin (Vancocin) reports ringing in both ears. Which initial action should the nurse take? 1 Notify the primary healthcare provider. 2 Consult an audiologist. 3 Stop the infusion. 4 Document the finding and continue to monitor the client.

3 Stop the infusion.

A nurse is caring for a client with pulmonary tuberculosis who is to receive several antitubercular medications. Which of the first-line antitubercular medications is associated with damage to the eighth cranial nerve? 1 Isoniazid (INH) 2 Rifampin (Rifadin) 3 Streptomycin 4 Ethambutol (Myambutol)

3 Streptomycin Streptomycin is ototoxic and can cause damage to the eighth cranial nerve, resulting in deafness. Assessment for ringing or roaring in the ears, vertigo, and hearing acuity should be made before, during, and after treatment.

A nurse reviews the medical record of a client with ascites. Which client condition may be contributing to the development of ascites? 1 Portal hypotension 2 Kidney malfunction 3 Diminished plasma protein level 4 Decreased production of potassium

3 Diminished plasma protein level The liver manufactures albumin, the major plasma protein. A deficiency of this protein lowers the osmotic (oncotic) pressure in the intravascular space, leading to a fluid shift. -An enlarged liver compresses the portal system, causing increased, rather than decreased, pressure. - The kidneys are not the primary source of the pathologic condition. - It is the liver's ability to manufacture albumin that maintains the colloid oncotic pressure. - Potassium is not produced by the body, nor is its major function the maintenance of fluid balance.

A client is receiving fresh frozen plasma (FFP). The nurse would expect to see improvement in which condition? 1 Thrombocytopenia 2 Oxygen deficiency 3 Clotting factor deficiency 4 Low hemoglobin

3 FFP is an unconcentrated form of blood plasma containing all of the clotting factors except platelets. It can be used to supplement red blood cells (RBCs) when other blood products are not available or to correct a bleeding problem of unknown cause. Thrombocytopenia is a condition of low platelet count and is not treated with FFP. An oxygen deficiency and low hemoglobin may be improved indirectly with FFP, but it is not a definitive treatment.

A nurse receives abnormal results of diagnostic testing. What action should the nurse take first? 1 Inform the client of the results. 2 Ensure that the results are placed in the client's medical record. 3 Notify the client's health care provider of the results. 4 Obtain results of the other lab tests that were performed.

3 The nurse is most ethically and legally accountable for reporting diagnostic testing results to the client's health care provider, whether the results are normal or, more important, abnormal. Informing the client of the results is an incorrect action in this situation. Placing the results in the client's record and obtaining normal values of the results from the lab are acceptable actions for the nurse after notifying the health care provider of the abnormal results.

What would be the respiratory rate in two-year-old child? Multiple choice question 20 30 40 50

30 The normal range for the respiratory rate in a two-year-old kid (toddler) is between 25 and 32 breaths per minute. Twenty breaths per minute is the normal respiratory rate in adolescents and adults. The normal respiratory rate in newborns is 40. The normal respiratory rate in infants is 50 breaths per minute.

While conducting an assessment, the nurse finds that the client shivers uncontrollably and experiences memory loss, depression, and poor judgment. What might the client's body temperature be? Multiple choice question 29° C 33° C 36° C 38° C

33° C A body temperature in the range of 36° to 38 ° C is normal. When skin temperature drops below 35° C, the client may exhibit uncontrolled shivering, loss of memory, depression, and poor judgment as a result of hypothermia. A body temperature lower than 30° C represents severe hyperthermia. In this condition, the client will demonstrate a lack of response to stimuli and extremely slow respiration and pulse. Based on the signs given, the client's temperature is most likely 33° C.

The nurse providing post-procedure care to a client who had a cardiac catheterization through the femoral artery discovers a large amount of blood under the client's buttocks. After donning gloves, which action should the nurse take first? 1 Apply pressure to the site. 2 Obtain vital signs. 3 Change the client's gown and bed linens. 4 Assess the catheterization site.

4 Assess the catheterization site. Observing standard precautions is the first priority when dealing with any body fluid, followed by assessment of the catheterization site as the second priority. This action establishes the source of the blood and determines how much blood has been lost. Once the source of the bleeding is determined the priority goal for this client is to stop the bleeding and ensure stability of the client by monitoring the vital signs.

A client is being discharged from the hospital with an indwelling urinary catheter. The client asks about the best way to prevent infection and keep the catheter clean. Which would be appropriate for the nurse to include in the client teaching? 1 Once a day, clean the tubing with a mild soap and water, starting at the drainage bag and moving toward the insertion site. 2 After cleaning the catheter site, it is important to keep the foreskin pushed back for 30 minutes to ensure adequate drying. 3 Clean the insertion site daily using a solution of one part vinegar to two parts water. 4 Change the drainage bag at least once a week as needed.

4 Change the drainage bag at least once a week as needed.

The client receives a prescription for tap water enemas until clear. The nurse is aware that no more than two enemas should be given at one time to prevent the occurrence of: 1 Hypercalcemia 2 Hypocalcemia 3 Hyperkalemia 4 Hypokalemia

4 Hypokalemia Repeated tap water enemas deplete cells and extracellular fluid of potassium and sodium resulting in hypokalemia, hyponatremia, and the potential for water intoxication.

The nurse is assessing a group of older adults. Which should the nurse consider to be least likely to be affected by aging? 1 Sense of taste or smell 2 Gastrointestinal motility 3 Muscle or motor strength 4 Strategies to handle stress

4 Strategies to handle stress

During history taking, a client reports experiencing black, tarry stools. The nurse recognizes that this may be an indication of: 1 Hemorrhoids, internal and external. 2 An overproduction of bile. 3 An iron deficiency. 4 Upper gastrointestinal bleeding.

4 Upper gastrointestinal bleeding.

The nurse is teaching a client about safe insulin administration. Which statement made by the client indicates the need for further education? 1 "I should see whether the insulin is expired." 2 "I should keep a daily logbook of times of insulin injection." 3 "I should keep my medication in its original labeled container." 4 "I should administer insulin only if there are any symptoms."

4 "I should administer insulin only if there are any symptoms."

A client injured in a motor vehicle accident was brought to the emergency and taken immediately for a scan. The client's family arrives later and asks about the client's health. What should the nurse tell the client's family? 1 "Please do not worry, everything will be alright." 2 "I am sorry; I do not have any information about the client." 3 "You will have to wait for the primary healthcare provider." 4 "Please wait; I will update you as soon as I have any information."

4 "Please wait; I will update you as soon as I have any information." The nurse should update the client's relatives once he or she receives relevant information. This action helps the nurse to maintain the nurse-client relationship. - The nurse must not provide false reassurances because this action affects the family's ability to adjust to any bad news. - If the nurse does not have any information about the client, the nurse must find out details and inform the client's family. - The nurse should not avoid the situation by asking the relatives to speak to the primary healthcare provider.

A nurse is caring for a postoperative client who had general anesthesia during surgery. What independent nursing intervention may prevent an accumulation of secretions? 1 Postural drainage 2 Cupping the chest 3 Nasotracheal suctioning 4 Frequent changes of position

4 Frequent changes of position minimize pooling of respiratory secretions and maximize chest expansion, which aids in the removal of secretions; this helps maintain the airway and is an independent nursing function. Postural drainage and cupping the chest are part of pulmonary therapy that requires a health care provider's prescription. Nasotracheal suctioning will remove secretions once they accumulate in the upper airway, not prevent their accumulation.

The nurse providing post-procedure care to a client who had a cardiac catheterization through the femoral artery discovers a large amount of blood under the client's buttocks. After donning gloves, which action should the nurse take first? 1 Apply pressure to the site. 2 Obtain vital signs. 3 Change the client's gown and bed linens. 4 Assess the catheterization site.

4 Observing standard precautions is the first priority when dealing with any body fluid, followed by assessment of the catheterization site as the second priority. This action establishes the source of the blood and determines how much blood has been lost. Once the source of the bleeding is determined the priority goal for this client is to stop the bleeding and ensure stability of the client by monitoring the vital signs. Changing the client's gown and bed linens is not necessary until the bleeding is controlled and the client is stabilized.

Which nursing activities are examples of primary prevention? Select all that apply. 1 Preventing disabilities 2 Correcting dietary deficiencies 3 Establishing goals for rehabilitation 4 Assisting with immunization programs 5 Facilitating a program about dangers of smoking

4, 5 Assisting with immunization programs Facilitating a program about dangers of smoking - Preventing disabilities - tertiary - Correcting dietary deficiencies - secondary - Establishing goals for rehabilitation - tertiary

A patient is hyperventilating from acute pain and hypoxia. Interventions to manage his pain and oxygenation will decrease his risk of which acid-base imbalance? 1. Metabolic acidosis 2. Metabolic alkalosis 3. Respiratory acidosis 4. Respiratory alkalosis

4. Respiratory alkalosis

A nurse assesses pain and redness at a vascular access device (VAD) site. Which action is taken first? 1. Apply a warm, moist compress 2. Monitor the patient's blood pressure 3. Aspirate the infusing fluid from the VAD 4. Stop the infusion and discontinue the intravenous infusion

4. Stop the infusion and discontinue the intravenous infusion

A client who weighs 176 pounds is receiving 8 mg/kg cyclosporine (Sandimmune) each day to prevent organ transplant rejection. How many milligrams should the nurse administer each day? Record your answer using a whole number. _________ mg

640 mg

The nurse was assessing an elderly client and recorded the pulse rate as 85. After assessment the nurse determined the cardiac output as 5950. What could be the approximate stroke volume? Multiple choice question 70 mL 60 mL 50 mL 40 mL

70 mL Cardiac output is obtained by multiplying the heart rate and the stroke volume. Therefore to obtain the stroke volume, the cardiac output should be divided by pulse rate. Dividing 5950 by 85 yields a stroke volume of 70 mL.

Mean Platelet Volume (MPV)

8.6-11.7 fl mL The average size of platelets in a volume of blood.

Normal Mean Corpuscular Volume (MCV)

81 - 99 fl (femtoliters, a fraction of one millionth of a liter) The average volume of a red blood cell.

The nurse who is working during the 8:00 am to 4:00 pm shift must document a client's fluid intake and output. An intravenous drip is infusing at 50 mL per hour. The client drinks 4 oz of orange juice and 6 oz of tea at 8:30 am and vomits 200 mL at 9:00 am. At 10:00 am the client drinks 60 mL of water with medications; the client voids 550 mL of urine at 11:00 am. At 12:30 pm, 3 oz of soup and 4 oz of ice cream are ingested. The client voids 450 mL at 2:00 pm. Calculate the total intake for the 8:00 am to 4:00 pm shift. Record your answer using a whole number.

970 mL

A registered nurse is teaching a nursing student about when a client with high blood pressure should follow up with the primary healthcare provider. Which statement made by the nursing student indicates effective learning? "I will advise a client with a blood pressure of 130/80 mm Hg to follow up in a year." "I will advise a client with a blood pressure of 110/70 mm Hg to follow up in a year." "I will advise a client with a blood pressure of 150/90 mm Hg to follow up in a month." "I will advise a client with a blood pressure of 185/115 mm Hg to follow up in a month."

A client with prehypertension tends to have a blood pressure (BP) between 120/80 and 139/89 mm Hg. These clients should be rechecked in a year. Clients with BP less than 120/80 mm Hg are considered normal. These clients should be rechecked in two years. Clients with stage 1 hypertension have a BP between 140/90 and 159/99 mm Hg. These clients should be rechecked in two months to confirm stage 1 hypertension. Clients with stage 2 hypertension have a BP greater than 160/100 mm Hg. These clients should be rechecked in one month. If a client's BP is greater than 180/110 mm Hg, then he or she should be treated immediately or within 1 week.

A hospitalized client experiences a fall after climbing over the bed's side rails. Upon reviewing the client's medical record, the nurse discovers that restraints had been prescribed but were not in place at the time of the fall. What information should the nurse include in the follow-up incident report?

A list of facts related to the incident as witnessed by the nurse. **In an incident report, fault or blame is subjective and should not be implied. It is not necessary to include names except for those of witnesses. Speculations or opinions as to the reason why the ordered restraints were not on the client are subjective and not appropriate to include in an incident or variance report.

A patient was recently told he has terminal lung cancer with a very poor prognosis. The nurse begins an assessment . The patient begins to explain to the nurse how he is planning a family trip to Australia in 2 years. The nurse identifies this as what stage of the grief cycle? Denial Anger Bargaining Depression Acceptance

A. Denial

A physically ill client is being verbally aggressive to the nursing staff. What is the most appropriate initial nursing response? 1 Accept the client's behavior. 2 Explore the situation with the client. 3 Withdraw from contact with the client. 4 Tell the client the reason for the staff's actions.

Accept the client's behavior. At this time the client is using this behavior as a defense mechanism. Acceptance can be an effective interpersonal technique because it is nonjudgmental. Eventually, limits may need to be set to address the behavior if it becomes more aggressive or hostile. During periods of overt hostility, perceptions are altered, making it difficult for the client to evaluate the situation rationally. Withdrawal signifies nonacceptance and rejection. The staff may be the target of a broad array of emotions; by focusing on only behaviors that affect the staff, the full scope of the client's feelings are not considered.

A client develops an increased temperature after surgery. Ceftriaxone (Rocephin) is prescribed. For which potential effect should the nurse monitor the client? Dehydration Heart failure Constipation Allergic response

Allergic response Rash, urticaria, pruritus, angioedema, and other signs and symptoms of an allergic response may occur a few days after therapy is instituted. Ceftriaxone does not cause dehydration, does not affect the heart, and may cause diarrhea, not constipation.

A client taking furosemide (Lasix), reports difficulty sleeping. What question is important for the nurse to ask the client? A. "Have you lost weight recently?" B. "At what time do you take your medication?" C. "Are you eating foods rich in potassium?" D. "What dose of medication are you taking?"

Answer: "At what time do you take your medication?" The nurse needs to first determine at what time of day the client takes the Lasix (D). Because of the diuretic effect of Lasix, clients should take the medication in the morning to prevent nocturia. The actual dose of medication (A) is of less importance than the time taken. (B) is not related to the insomnia. (C) is valuable information about the effect of the diuretic, but is not likely to be related to insomnia.

A client receives a prescription for levothyroxine sodium (Synthroid) 0.05 mg by mouth once daily. The medication is available in scored tablets labeled 0.1 mg. How many tablets should the nurse administer?

Answer: 0.5 tablets Using the formula, Desired / Available x 1 tablet = 0.05/0.1 x 1 = 0.5

A client with type 2 diabetes is receiving metformin (Glucophage) 1 gram PO twice daily. The medication is available in 500 mg tablets. How many tablets should the nurse administer? (Enter numeric value only.)

Answer: 2 tablets Using the known equivalent, 1 gram = 1000 mg, the nurse should first convert the dose to the same unit of measurement, which is 1 gram = 1000 mg. Using the formula, Desired / Available x 1 tablets: 1000 mg / 500 mg x 1 = 2 tablets

A client taking a thiazide diuretic for the past six months has a serum potassium level of 3. The nurse anticipates which change in prescription for the client? A. The dosage of the diuretic will be decreased. B. The dosage of the diuretic will be increased. C. A potassium supplement will be prescribed. D. The diuretic will be discontinued.

Answer: A potassium supplement will be prescribed This client's potassium level is too low (normal is 3.5 to 5). Taking a thiazide diuretic often results in a loss of potassium, so a potassium supplement needs to be prescribed to restore a normal serum potassium level (C). (A, B, and D) are not recommended actions for restoring a normal serum potassium level.

A male client with acquired immunodeficiency syndrome (AIDS) develops cryptococcal meningitis and tells the nurse he does not want to be resuscitated if his breathing stops. What action should the nurse implement? A. Inform the client that a written, notarized advance directive, is required to withhold resuscitation efforts. B. Ask the client if this decision has been discussed with his healthcare provider. C. Document the client's request in the medical record. D. Advise the client to designate a person to make healthcare decisions when the client is unable to do so.

Answer: Ask the client if this decision has been discussed with his healthcare provider. Advance directives are written statements of a person's wishes regarding medical care, and verbal directives may be given to a healthcare provider with specific instructions in the presence of two witnesses. To obtain this prescription, the client should discuss his choice with the healthcare provider (B). (A) is insufficient to implement the client's request without legal consequences. Although (C and D) provide legal protection of the client's wishes, the present request needs additional action.

All of the following interventions can be used to evaluate the effectiveness of nursing and medical interventions used to treat diarrhea. Which intervention is least useful in the nurse's evaluation of a 20-month-old child? A. Assessing fontanels. B. Weighing diapers. C. Observing mucous membranes for moisture. D. Checking skin turgor.

Answer: Assessing fontanels. All of these interventions evaluate fluid status in infants. But, how old is this child? Posterior fontanel closes at 2 months and anterior fontanel closes by 18 months of age (B)! Remember normal growth and development!

The parents of a child with hemophilia A ask the nurse about their probability of having another child with hemophilia A. Which information is the basis for the nurse's response? (Select all that apply.) A. Men with hemophilia have sons who also manifest the disease. B. The disease occurs in daughters of men with hemophilia. C. Autosomal dominance occurs with this disorder. D. Sons of female carriers have a 50% chance of inheriting hemophilia. E. Hemophilia is an X-linked recessive disorder.

Answer: B,E Correct choices are (B and E). Hemophilia is an inherited disease that manifests in male children whose mother is a carrier. With each pregnancy there is a 50% chance that a male child will inherit the defective gene and manifest hemophilia A (B), which is an X-linked recessive disorder (E). (A) is descriptive of a rare type of hemophilia, known as von Willebrand's disease. Hemophilia is inherited by male offspring of female carriers (C). Daughters (D) do not manifest the disease, but have a 50% chance of being a carrier.

A client who is known to abuse drugs is admitted to the psychiatric unit. Which medication should the nurse anticipate administering to a client who is exhibiting benzodiazepine withdrawal symptoms? A. Isocarboxazid (Marplan). B. Chlordiazepoxide (Librium). C. Diphenhydramine (Benadryl). D. Perphenazine (Trilafon).

Answer: Chlordiazepoxide (Librium). Librium (B), an antianxiety drug, as well as other benzodiazepines, is used in titrated doses to reduce the severity of abrupt benzodiazepine withdrawal. (A) is an antipsychotic agent. (C) is an antihistamine and antianxiety drug. (D) is an MAO inhibitor.

The nurse is assessing an 8-month-old child who has a medical diagnosis of Tetrology of Fallot. Which symptom is this client most likely to exhibit? A. Clubbed fingers B. Machinery murmur. C. pedal pulses. D. Bradycardia

Answer: Clubbed fingers Tetrology of Fallot, a cyanotic heart defect, causes clubbing of fingers and toes (D) due to tissue hypoxia. Tachycardia, not (A), is a manifestation of congenital heart disease. (B) is a classic sign of ventricular septal defect. (C) is characteristic of coarctation of the aorta.

The nurse is assessing a 2-year-old. What behavior indicates that the child's language development is within normal limits? A. Half of child's speech is understandable. B. Is able to name four colors. C. Is capable of making a three word sentence. D. Can count five blocks.

Answer: Half of child's speech is understandable. Between approximately 15 and 24 months of age, a child's speech is only half understandable (D). (A and B) usually occur between 3 and 5 years of age. (C) is usually accomplished by 18 months of age.

An 8-year-old boy who is recently diagnosed with diabestes mellitus is admitted to the intensive care unit with diabetic ketoacidosis (DKA). Which nursing action has the highest priority? A. Initiate an intravenous infusion. B. Obtain fingerstick glucose. C. Place on cardiac monitor. D. Collect specimen for serum electrolytes.

Answer: Initiate an intravenous infusion The priority for a child with DKA, an emergency life-threatening situation, is to obtain venous access for administration of fluids, electrolytes, and insulin (B). The child should be placed on a cardiac monitor and have serum electrolytes and glucose levels obtained but not before initiating venous access (A, C, and D).

The healthcare provider prescribes high-protein, high-fat, low-carbohydrate diet with limited fluids during meals for a client recovering from gastric surgery. The client asks the nurse what the purpose is for this type of diet. Which rationale should be included in the nurse's explanation to this client? A. It does not dilate the stomach. B. It is quickly digested. C. It is slow to leave the stomach. D. It does not cause diarrhea.

Answer: It is slow to leave the stomach. This type of diet is slowly digested and is slow to leave the stomach (D). Because of its density from proteins an fats, and the reduction of fluids with the meal, the possibility of dumping syndrome is reduced. (A, B, and C) are incorrect rationales.

The nurse is caring for a client who had an excision of a malignant pituitary tumor. Which findings should the nurse document that indicate the client is developing syndrome of inappropriate antidiuretic hormone (SIADH)? A. Increased urinary output and thirst. B. Hypernatremia and periorbial edema. C. Weight gain with low serum sodium. D. Muscle spasticity and hypertension.

Answer: Weight gain with low serum sodium. SIADH most frequently occurs when cancer cells manufacture and release ADH, which is manifested by water retention causing weight gain and hyponatremia (C). Other manifestations include oliguria, weakness, not (A, B, and D), anorexia, nausea, vomiting, personality changes, seizures, decrease in reflexes, and coma.

A nurse is admitting a client to the unit from the emergency room. The nurse asks the client about past medical history. What phase of the nursing process is the nurse in? Assessment Planning Implementation Evaluation

Assessment

While performing a physical assessment of a client, a nurse notices patchy areas with loss of pigmentation on the skin, hands, and arms. What is the probable etiology for this condition? Multiple choice question Anemia Pregnancy Lung disease Autoimmune disease

Autoimmune disease Patchy areas with loss of pigmentation on skin, hands, and arms are due to vitiligo, which is caused by an autoimmune or congenital disease. Anemia results in pallor due to a reduced amount of oxyhemoglobin. A tan-brown color of the skin is noticed in pregnancy due to an increased amount of melanin. Lung disease or heart failure can cause cyanosis due to an increased amount of deoxygenated hemoglobin.

You are the nurse caring for a patient admitted with dehydration? Which of the following orders would you anticipate the physician to write? (Select all that apply) Lasix 40mg IVP BID 0.9 % Normal saline Bolus Increase fluid intake Routine vital signs

B, C, D

A client's weight has increased in one day by approximately two pounds. The nurse knows that this increase in weight should equal approximately how much fluid? A. 500 cc B. 1 L C. 300 ml D. 2 L

B. 1 liter

A client with terminal cancer says to the nurse, "If I could just be free of pain for a few days, I might be able to eat more and regain strength." Which stage of grieving does the nurse concludes the client is experiencing? Bargaining Frustration Depression Rationalization

Bargaining

A nurse is teaching a parenting class. What should the nurse suggest about managing the behavior of a young school-age child? Multiple choice question Avoid answering questions. Give the child a list of expectations. Be consistent about established rules. Allow the child to plan the day's activities.

Be consistent about established rules. Because of a short attention span and distractibility, consistent limit setting is essential toward providing an environment that promotes concentration, prevents confusion, and minimizes conflicts. Questions should be answered, but the answers should not be judgmental. A list of expectations may be overwhelming at this age. Parents need to assist children with routine tasks; children this age may not be concerned with time frames.

Plasma proteins help contain blood within the blood vessels by exerting: A. Filtration pressure B. Hydrostatic pressure C. Colloid osmotic pressure D. Diffusion pressure

C. Colloid osmotic pressure

The movement of particles from an area of high solute concentration to an area of lower solute concentration is: A. Filtration B. Hydrostatic pressure C. Diffusion D. Osmosis

C. Diffusion

What is the stroke volume from a cardiac output of 5950 and a pulse rate of 85.

Cardiac output is obtained by multiplying the heart rate and the stroke volume. Therefore to obtain the stroke volume, the cardiac output should be divided by pulse rate. Dividing 5950 by 85 yields a stroke volume of 70 mL.

When trying to promote effective learning in a client with a newly diagnosed disease, what should the nurse consider? 1 Client's past experiences 2 Client's personal resources 3 Stress of the total situation 4 Type of onset of the disease

Client's past experiences Past experiences have the most meaningful influence on present learning. Although the client's personal resources, the stress of the total situation, and the type of onset of the disease affect learning, their influence is not as great as past experiences.

Which findings in the older client are associated with a urinary tract infection (UTI)? Select all that apply. 1 Fever 2 Urgency 3 Confusion 4 Incontinence 5 Slight rise in temperature

Confusion Incontinence Slight rise in temperature An older client with a urinary tract infection (UTI) is likely to appear confused. An older client may experience incontinence while a younger client may experience urgency. The older client may develop a slight rise in temperature. The classic symptoms of a UTI in a younger client are fever, dysuria, and urgency.

A nurse assesses the lungs of a client and auscultates soft, crackling, bubbling breath sounds that are more obvious on inspiration. This assessment should be documented as what? Vesicular Bronchial Crackles Rhonchi

Crackles

What is the priority nursing action for a client with delirium?

Creating a calm and safe environment

Thrombocytopenia

Decreased # of platelets, can cause small blue colored spots due to bleeding from the vessels into tissues.

What type of interview is most appropriate when a nurse admits a client to a clinic? Multiple choice question Directive Exploratory Problem solving Information giving

Directive The first step in the problem-solving process is data collection so that client needs can be identified. During the initial interview a direct approach obtains specific information, such as allergies, current medications, and health history. The exploratory approach is too broad, because in a nondirective interview the client controls the subject matter. Problem solving and information giving are premature at the initial visit.

What clinical finding does a nurse anticipate when admitting a client with an extracellular fluid volume excess? Multiple choice question Rapid, thready pulse Distended jugular veins Elevated hematocrit level Increased serum sodium level

Distended jugular veins Because of fluid overload in the intravascular space, the neck veins become visibly distended. Rapid, thready pulse and elevated hematocrit level occur with a fluid deficit. If sodium causes fluid retention, its concentration is unchanged; if fluid is retained independently of sodium, its concentration is decreased.

The nurse reviews a medical record and is concerned that the client may develop hyperkalemia. Which disease increases the risk of hyperkalemia? Crohn disease Cushing disease End-stage renal disease Gastroesophageal reflux disease

End-stage renal disease

A nurse is caring for a client who is having diarrhea. To prevent an adverse outcome, the nurse should most closely monitor what client data or assessment finding? Skin condition Fluid and electrolyte balance Food intake Fluid intake and output

Fluid and electrolyte balance

Which nursing interventions enhance comfort in an imminently dying client in the hospital? Select all that apply. Frequently repositioning the client Maintaining oral hygiene in the client Limiting frequent visits of the family members Measuring the vital signs of client frequently Applying body lotion to the client's skin daily

Frequently repositioning the client Maintaining oral hygiene in the client Applying body lotion to the client's skin daily

The nurse is teaching a client about adequate hand hygiene. What component of hand washing should the nurse include that is most important for removing microorganisms? 1 Soap 2 Time 3 Water 4 Friction

Friction Friction is necessary for the removal of microorganisms. Although soap reduces surface tension, which helps remove debris, without friction it has minimal value. Although the length of time the hands are washed is important, without friction it has minimal value. Although water flushes some microorganisms from the skin, without friction it has minimal value.

A nurse is providing colostomy care to a client with a nosocomial infection caused by methicillin-resistant Staphylococcus aureus (MRSA). Which personal protective equipment (PPE) should the nurse use? Select all that apply. 1 Gloves 2 Gown 3 Mask 4 Goggles 5 Shoe covers 6 Hair bonnet

Gloves Gown Goggles Standard PPE, which should be used for performing colostomy care in a client positive for MRSA, includes gloves, gown, and goggles. A combination mask/eye shield may be used when caring for this client; however, a mask is not necessary. Shoe covers and hair bonnet are not required for the patient care situation described.

While inspecting the external eye structure of a client, a nurse finds bulging of the eyes. Which condition can be suspected in the client? Multiple choice question Eye tumors Hypothyroidism Hyperthyroidism Neuromuscular injury

Hyperthyroidism Bulging eyes may indicate hyperthyroidism. Tumors are characterized by abnormal eye protrusions. Hypothyroidism can be revealed by the coarseness of the hair of the eyebrows and the failure of the eyebrows to extend beyond the temporal canthus. Crossed eyes or strabismus may result from neuromuscular injury or inherited abnormalities.

A client with a head injury underwent a physical examination. The nurse observes that the client's temperature assessments do not correspond with the client's condition. An injury to which part of the brain may be the reason for this condition? Multiple choice question Pons Medulla Thalamus Hypothalamus

Hypothalamus The hypothalamus controls the body temperature. Damage to the hypothalamus may cause abnormalities in the body temperature values during a physical assessment. The pons is responsible for maintaining level of consciousness. The medulla controls heart rate and breathing. The thalamus performs motor and sensory functions.

A nurse is preparing a community health program for senior citizens. The nurse teaches the group that what physical findings are typical in older adults? Multiple choice question Increased skin elasticity and a decrease in libido Impaired fat digestion and increased salivary secretions Increased blood pressure and decreased hormone production An increase in body warmth and some swallowing difficulties

Increased blood pressure and decreased hormone production With aging, narrowing of the arteries causes some increase in the systolic and diastolic blood pressures; hormone production decreases after menopause. There may or may not be changes in libido; there is a loss of skin elasticity. Salivary secretions decrease, not increase, causing more difficulty with swallowing; there is some impairment of fat digestion. There may be a decrease in subcutaneous fat and decreasing body warmth; some swallowing difficulties occur because of decreased oral secretions.

A nurse is assessing an 89-year-old client with a history of severe congenital spinal deformity. Which condition would most likely describe the nurse's finding? Lordosis Kyphosis Presbycusis Osteoporosis

Kyphosis Kyphosis is an increase in the curvature of the thoracic spine and may result from a congenital abnormality. Lordosis, also known as swayback, is an increased lumbar curvature and may not be a congenital abnormality. Presbycusis is the loss of acuity for high-frequency tones and is not related to the spine. Osteoporosis is a condition in which the bones become brittle and fragile from the loss of tissue and bone mass.

A client reports vomiting and diarrhea for 3 days. Which clinical indicator is most commonly used to determine whether the client has a fluid deficit? Presence of dry skin Loss of body weight Decrease in blood pressure Altered general appearance

Loss of body weight

The primary healthcare provider prescribes thioridazine and assigns the nurse to assess the client for orthostatic hypotension. Which interventions would the nurse perform? Select all that apply.

Measuring BP before dosing Measuring BP 1 hr after dosing Measuring BP 1 or 2 mins after clients stands or sits *Antipsychotic drugs such as thioridazine may cause orthostatic hypotension as a side effect. Before delivering a dose, the nurse should measure the client's blood pressure.

When monitoring fluids and electrolytes, the nurse recalls that the major cation-regulating intracellular osmolarity is what? Sodium Potassium Calcium Calcitonin

Potassium

What does a nurse consider the most significant influence on many clients' perception of pain when interpreting findings from a pain assessment? Multiple choice question Age and sex Physical and physiological status Intelligence and economic status Previous experience and cultural values

Previous experience and cultural values Interpretation of pain sensations is highly individual and is based on past experiences, which include cultural values. Age and sex affect pain perception only indirectly because they generally account for past experience to some degree. Overall physical condition may affect the ability to cope with stress; however, unless the nervous system is involved, it will not greatly affect perception. Intelligence is a factor in understanding pain so it can be tolerated better, but it does not affect the perception of intensity; economic status has no effect on pain perception.

The nurse cares for an unconscious client who underwent head surgery. Which site would be best used to monitor body temperature? Multiple choice question Skin Oral Axilla Rectal

Rectal Although the oral route is the most common route for monitoring body temperature, clients who are unconscious should have their temperatures monitored rectally. Skin temperature may be impaired due to diaphoresis; this measurement may not reliable. The axilla temperature may underestimate the core temperature.

A nurse who is caring for a client experiencing anginal pain expects that the pain will be: Unchanged by rest Precipitated by light activity Described as a knifelike sharpness Relieved by sublingual nitroglycerin

Relieved by sublingual nitroglycerin Relief by sublingual nitroglycerin is a classic reaction because it dilates coronary arteries, which increases oxygen to the myocardium, thus decreasing pain. Immediate rest frequently relieves anginal pain. Angina usually is precipitated by exertion, emotion, or a heavy meal. Angina usually is described as tightness, indigestion, or heaviness.

A client receives a prescription for morphine via patient-controlled analgesia (PCA). Before beginning administration of this medication, what should the nurse assess first?

Respirations

A client is admitted with metabolic acidosis. The nurse considers that two body systems interact with the bicarbonate buffer system to preserve healthy body fluid pH. What two body systems should the nurse assess for compensatory changes? Multiple choice question Skeletal and nervous Circulatory and urinary Respiratory and urinary Muscular and endocrine

Respiratory and urinary Increased respirations blow off carbon dioxide (CO 2), which decreases the hydrogen ion concentration and the pH increases (less acidity). Decreased respirations result in CO 2 buildup, which increases hydrogen ion concentration and the pH falls (more acidity). The kidneys either conserve or excrete bicarbonate and hydrogen ions, which helps to adjust the body's pH. The buffering capacity of the renal system is greater than that of the pulmonary system, but the pulmonary system is quicker to respond. Skeletal and nervous systems do not maintain the pH, nor do muscular and endocrine systems. Although the circulatory system carries fluids and electrolytes to the kidneys, it does not interact with the urinary system to regulate plasma pH.

The nurse finds the respiratory rate is 8 breaths per minute in a client who is on intravenous morphine sulfate. What should the nurse do immediately in this situation?

Stop administering the medication

Which warning signals should the nurse observe in a child suspected to be a victim of abuse? Select all that apply. 1 The child doesn't want to be touched by anyone. 2 The child sleeps for an average of 15 hours a day. 3 The child frequently visits the emergency department. 4 The child suffers from fever and tenderness in the abdomen. 5 The child looks at the caregiver before answering any question.

The child doesn't want to be touched by anyone. The child frequently visits the emergency department. The child looks at the caregiver before answering any question. The child may become scared if touched. The physical abuse may cause injuries and the child may visit the emergency department frequently. An abused child may look at the caregiver before answering any question due to fear. The child sleeping for an average of 15 hours a day does not indicate abuse. Fever and tenderness in the abdomen are not signs of abuse; it could indicate an organic cause.

The nurse assessing an adult understands that the client is experiencing a midlife crisis. Which factor should the nurse attribute to this condition? 1 The client is seeking an occupational direction. 2 The client is examining life goals and relationships. 3 The client is directing energy towards achievements. 4 The client is sharing responsibilities in a two-career family.

The client is examining life goals and relationships. Individuals between the age of 35 and 43 are vigorously examining their life goals and relationships. These individuals often experience stress or a midlife crisis during this reexamination, which may lead to changes in personal, social, and occupational areas. A young adult who is aware of his or her skills seeks to pursue a degree suitable to his or her desired occupation. A young adult between the ages of 29 to 34 directs enormous energy toward achievement and mastery of the surrounding world. A young adult must share all responsibilities in a two-career family to avoid stress.

The nurse is assessing a client who had knee replacement surgery. Which assessment finding gathered by the nurse is an example of subjective data? Multiple choice question The client weighs 151 lbs (68.5 Kg). The client's pain is 7 on a scale of 1 to 10. The client's fasting blood sugar is 95 mg/dL. The client's blood pressure is 140/90 mm/Hg.

The client's pain is 7 on a scale of 1 to 10. Subjective data is information conveyed to the nurse by the client, such as the client's feelings, perceptions, and self-reporting of symptoms. The client rates pain as a 7 on a scale of 1 to 10, therefore it is subjective data. Objective data are observations or measurements of a client's health status. The client's weight is measured on a weighing scale; therefore, it is objective data. A laboratory result such as fasting blood sugar and blood pressure are measurable quantities.

What is minimum urinary output nurses should watch for?

The urine output should be at least 30 ml per hour. Less than 30 ml per hour indicates the need for notifying the healthcare provider because low urine output indicates volume depletion that may result in renal damage.

What assessment findings indicate that an older client is at risk for developing an infection? Select all that apply.

Thin skin Weak Cough Sluggish bowl sounds Indwelling urinary catheter

While assessing a client, the nurse finds inflammation of the skin at the bases of the client's nails. What might be the reason behind this condition? Multiple choice question Trauma Trichinosis Pulmonary disease Iron-deficiency anemia

Trauma Paronychia is an abnormality of the nail bed. The condition is marked by inflammation of the skin at the base of the nail; this condition may be caused by trauma or a local infection. Trichinosis is associated with red or brown linear streaks in the nail bed. Pulmonary diseases can cause changes in the angle between nail and nail base, which is a phenomenon known as clubbing. Koilonychia, a concave curvature of the nails, may occur as a result of iron-deficiency anemia.

You are taking care of a patient with a potassium level of 6.4? What are some things you would anticipate a physician to order?

We could give a potassium wasting diuretic...look up some to see which would be appropriate. We could also give Kayexalate to a patient which will excrete K+ through the feces.

Pectus carinatum

also called pigeon chest, is a deformity of the chest characterized by a protrusion of the sternum and ribs.

Rhonchi

associated with obstruction by a foreign body.

Pleural Friction Rub

can be heard in cases of pleurisy.

Hypoventilation

decreased rate and/or depth - low oxygen level, high carbon dioxide level. 1. Bradypnea - too slow < 10 breaths/minute 2. Hypopnea - too shallow *If caused by opioids, can be reversed with Narcan

Remittent Fever Pattern

fever spikes and falls without returning to normal temperature levels

Dyspnea

subjective "I feel short of breath" or can be objective. "Exertional dyspnea"- caused by exercise/increased metabolic rate, increased demand for oxygen. "Conversational dyspnea" - inability to speak complete sentences without stopping to breathe.

A nurse is assessing an older adult during a regular checkup. Which findings during the assessment are normal? Multiple selection question Loss of turgor Urinary incontinence Decreased night vision Decreased mobility of ribs Increased sensitivity to odors

-Decreased night vision -Decreased mobility of ribs -Loss of turgor In older adults, the skin loses its turgor or elasticity and there is fat loss in the extremities. Visual acuity declines with age; therefore, decreased night vision is a normal finding in older adults. Decreased mobility of the ribs is found in older adults due to calcification of the costal cartilage. Urinary incontinence is an abnormal finding in older adults. In older adults, diminished sensitivity to odor, not increased sensitivity, is often found.

A prescription is written for famotidine (Pepcid) 20 mg intravenous piggyback (IVPB) every 12 hours. The vial is labeled 10 mg/1 mL. How many milliliters should the nurse administer? Record the answer using a whole number. _______ mL

2 mL

To prevent thrombophlebitis in the immediate postoperative period, which action is most important for a nurse to include in the client's plan of care? 1 Increase fluid intake. 2 Restrict fluids. 3 Encourage early mobility. 4 Elevate the knee gatch of the bed.

3 Encourage early mobility.

The nurse is providing post-procedure care for a client that had a liver biopsy. To prevent hemorrhage, it is the nurse's highest priority to place the client in what position? 1 Prone 2 High-Fowler's 3 On the right side 4 Trendelenburg

3 On the right side Placing a client on the right side after a liver biopsy compresses the liver against the abdominal wall, thus holding pressure on the biopsy site and allowing clot formation.

What would be the behavioral characteristic of a slow-to-warm up child according to the theory related to temperament? 1 Highly active 2 Irritable and irregular in habits 3 Negative reaction to new stimuli 4 A positive mild-to-moderately intense mood

3 Negative reaction to new stimuli

Which assessment does a nurse use as a clinical marker of vascular volume in a patient at high risk of extracellular fluid volume (ECV) deficit? 1. Dryness of mucous membranes 2. Presence or absence of edema 3. Fullness of neck veins when supine 4. Fullness of neck veins when upright

3. Fullness of neck veins when supine

A nurse is caring for a client with albuminuria resulting in edema. What pressure change does the nurse determine to be the cause of the edema? 1 Decrease in tissue hydrostatic pressure 2 Increase in plasma hydrostatic pressure 3 Increase in tissue colloid osmotic pressure 4 Decrease in plasma colloid oncotic pressure (COP)

4 Because the plasma COP is the major force drawing fluid from the interstitial spaces back into the capillaries, a drop in COP caused by albuminuria results in edema. Hydrostatic tissue pressure is unaffected by alteration of protein levels; colloidal pressure is affected. Hydrostatic pressure is influenced by the volume of fluid and the diameter of the blood vessel, not directly by the presence of albumin. The osmotic pressure of tissues is unchanged.

The registered nurse instructed the nursing student to care for a client who suffers from depression. During a follow up visit, the registered nurse finds that the client's symptoms have not improved. Which activity of the nursing student would the registered nurse relate this to? 1 Modifying the environment 2 Limiting the client's choices of diet and clothing 3 Encouraging fluid intake 4 Discouraging social interaction to avoid the client's distraction from outside environment

4 Discouraging social interaction to avoid the client's distraction from outside environment The nursing student's act of discouraging interactions due to fear of the client's distraction may result in a lack of improvement. Social interactions should be encouraged instead. - Modifying the environment may help to provide better healthcare. - The nurse should limit the client's choices of food and clothing to relieve any decision-making stress. - The nurse should also encourage fluid intake.

The nurse is performing nursing care therapies and including the client as an active participant in the care. Which basic step is involved in this situation? 1 Planning 2 Evaluation 3 Assessment 4 Implementation

4 Implementation

Hypoventilation

An abnormally low respiratory rate and the depth of ventilation is depressed.

What assessment finding should the nurse identify that indicates a client with an acute asthma exacerbation is beginning to improve after treatment? A. Vesicular breath sounds decrease B. Bronchodilators stimulate coughing C. Cough remains unproductive D. Wheezing becomes louder

Answer : Wheezing becomes louder. In an acute asthma attack, air flow may be so significantly restricted that wheezing is diminished. If the client is successfully responding to bronchodilators and respiratory treatments, wheezing becomes louder (A) as air flow increases in the airways. As the airways open and mucous is mobilized in response to treatment, the cough becomes more productive, not (B). Vesicular sounds are soft, low-pitched, gentle, rustling sounds heard over lung fields (C) and is not an indicator of improvement during asthma treatment. Bronchodilators do not stimulate coughing (D).

The nurse plans to mix a medication with food to make it more palatable for a pediatric client. Which food should the nurse choose? A. Formula or milk. B. Syrup. C. Applesauce. D. Orange juice.

Answer: Applesauce In order to prevent the child from developing a negative association with an essential food, a nonessential food such as applesauce is best for mixing with medications (B). Syrup is not used to mix with medications because of its high sugar content (A). Medications may alter the flavor of the food and cause the child to avoid those foods in the future, so orange juice (C), which provides essential nutritional elements, and formula or milk (D), which are essential foods in a child's diet, should not be mixed with medications.

An elderly client is admitted with a diagnosis of bacterial pneumonia. The nurse's assessment of the client will most likely reveal which sign/symptom? A. Pharyngitis and sputum production. B. Leukocytosis and febrile. C. Confusion and tachycardia. D. Polycythemia and crackles.

Answer: Confusion and tachycardia. The onset of pneumonia in the elderly may be signaled by general deterioration, confusion, increased heart rate or increased respiratory rate (D). (A, B, and C) are often absent in the elderly with bacterial pneumonia.

The nurse decides to teach deep-breathing exercises to a client recovering from a surgery. Which professional responsibility does the nurse display?

Autonomy

Fluids in the interstitial spaces are called: A. Intracellular fluids B. Extracellular fluids C. Electrolytes D. Intravascular fluids

B. Extracellular Fluids

What clinical indicators should the nurse expect a client with hyperkalemia to exhibit? Select all that apply. Tetany Seizures Confusion Weakness Dysrhythmias

Confusion Weakness Dysrhythmias

What is considered to be the highest priority for an assault victim who presents to the emergency department?

Ensuring the client's emotional and physical safety

Which are extrinsic factors responsible for falls in older adults? Select all that apply.

Environmental factors Inappropriate footwear Improper use of assistive devices

A nurse considers that communication links people with their surroundings. What should the nurse identify as the most important communication link? 1 Social 2 Physical 3 Materialistic 4 Environmental

Social Without some form of communication, there can be no socialization. People interact with other social beings, not with inanimate objects. Physical, materialistic, and environmental surroundings are all inanimate and cannot interact.

Which of the following is a description of the percussion technique? Multiple choice question Listening to sounds that the body makes Using the sense of touch to assess and collect data Carefully looking for abnormal findings Tapping the skin with the fingertips to vibrate underlying tissues

Tapping the skin with the fingertips to vibrate underlying tissues Percussion is a technique used to assess the skin by tapping the skin with the fingertips to vibrate underlying tissues and organs. Auscultation involves listening to the sounds that the body makes. Palpation involves using the sense of touch to assess and collect data. Generally during an inspection, the nurse should carefully look for abnormal findings.

What is the normal RBC count for a healthy male?

The normal RBC count for a healthy male is 4.7 to 6.1 million/mm3

Pruritis

severe itching of the skin

Which statement made by the nurse indicates that the client interview is coming to a close? Multiple choice question "I have just one more question for you." "I hope you are comfortable and not in pain." "I would like to spend some time to understand your concerns." "I assure you that information I gather now will be confidential."

"I have just one more question for you." The nurse should give the client a clue that the interview is drawing to a close. The nurse can do this by letting the client know that after one more question the interview will be over. The nurse sets the stage for the interview by ensuring that the client is comfortable and not in pain. The nurse begins the interview by stating that he or she would like to spend some time to understand the client's health concerns. The nurse informs the client at the beginning of the interview that the information shared by the client is confidential.

A registered nurse is teaching a student nurse about various sites for assessing body temperature. Which statements made by the student nurse indicates the need for further teaching? Multiple selection question "The axilla is not recommended to measure body temperature in unconscious clients." "The oral cavity is not suitable for clients with epilepsy to measure body temperature." "The tympanic membrane is not a preferred site of measuring body temperature in infants." "The rectum is not a preferred site of measuring body temperature in clients who underwent rectal surgeries." "The temporal artery is not a preferred site of thermometer placement to measure rapid changes in core temperature."

-"The axilla is not recommended to measure body temperature in unconscious clients." -"The tympanic membrane is not a preferred site of measuring body temperature in infants." -"The temporal artery is not a preferred site of thermometer placement to measure rapid changes in core temperature." The axilla is the preferred site for measuring body temperature in unconscious clients. The tympanic membrane is the preferred site for measuring body temperature in newborns to reduce infant handling and heat loss. The region of the temporal artery reflects rapid changes in core temperature. The oral cavity is not a preferred site to measure body temperature for a client with epilepsy, oral surgery, trauma, or shaking chills. In clients with diarrhea, rectal abnormalities, bleeding tendencies, and clients who underwent rectal surgeries, the rectum is not the preferred site for measuring body temperature.

The nurse recognizes that which are important components of a neurovascular assessment? Multiple selection question Orientation Capillary refill Pupillary response Respiratory rate Pulse and skin temperature Movement and sensation

-Capillary refill -Pulse and skin temperature -Movement and sensation A neurovascular assessment involves evaluation of nerve and blood supply to an extremity involved in an injury. The area involved may include an orthopedic or soft tissue injury. A correct neurovascular assessment should include evaluation of capillary refill, pulses, warmth and paresthesias, and movement and sensation. Orientation, pupillary response, and respiratory rate are components of a neurologic assessment.

A client is admitted with severe diarrhea that resulted in hypokalemia. The nurse should monitor for what clinical manifestations of the electrolyte deficiency? Diplopia Skin rash Leg cramps Tachycardia Muscle weakness

-Leg cramps -Muscle weakness Leg cramps occur with hypokalemia because of potassium defiit. Muscle weakness occurs with hypokalemia because of the alteration in the sodium potassium pump mechanism. Diplopia does not indicate an electrolyte deficit. A skin rash does not indicate an electrolyte deficit. Tachycardia is not associated with hypokalemia; bradycardia is.

While performing a physical assessment of a female client, the nurse positions the client in Sims' position. Which body system will be assessed in this position? Multiple selection question Heart Vagina Rectum Female genitalia Musculoskeletal system

-Vagina -Rectum Sims' position is indicated to examine vagina and rectum. Lithotomy to check female genitalia. Lateral recumbent position will aid in detecting murmurs of the heart. Prone position is indicated while assessing the musculoskeletal system.

A health care provider prescribes 250 mg of a medication. The vial reads 500 mg/mL. How much medication (mL) should the nurse administer? Include a leading zero if applicable. Record your answer using one decimal place. _____ mL

0.5 mL

A nurse is assigned to take care of a group of clients. Which client should the nurse see first? 1 A 2-yr-old male with diarrhea 2 A 35-yr-old male who is nauseated 3 A 40-yr-old female who has vomiting due to food poisoning 4 An 83-yr-old female whose last bowel movement was three days ago.

1 A 2-yr-old male with diarrhea The two-year-old child will be at higher risk for fluid and electrolyte imbalance due to higher fluid content of the body and decreased ability to regulate fluid balance, which put this client in life threatening situation.

A nurse is evaluating the effectiveness of treatment for a client with excessive fluid volume. What clinical finding indicates that treatment has been successful? 1 Clear breath sounds 2 Positive pedal pulses 3 Normal potassium level 4 Increased urine specific gravity

1 Clear breath sounds

Which questions should the nurse ask the client when obtaining the health history? Select all that apply. "Tell me about your food habits." "Do you use alcohol or tobacco?" "Have you sustained any personal loss recently?" "Have you ever experienced any allergic reactions?" "Does any family member have a long-term illness?"

1,2,4 The health history of a client includes the client's food habits so that the nurse can obtain an assessment of the client's nutrition status. The nurse also assesses the client's habits and lifestyle patterns. The use of alcohol and tobacco helps to determine the client's risk for diseases involving the liver or lungs. The health history includes descriptions of allergies and reactions to food, latex, drugs, or contact agents such as soap. While assessing the family history, the nurse assesses the client for stress-related problems by asking about recent personal losses. The family history provides information about family members to determine the risk for illnesses of a genetic or familial nature.

8. The health care provider's order is 500 mL 0.9% NaCl intravenously over 4 hours. Which rate does a nurse program into the infusion pump? 1. 125 mL/hr 2. 167 mL/hr 3. 200 mL/hr 4. 1000 mL/hr

1. 125 mL/hr

Normal Platelet Count

150 - 400 K/mL Number of platelets in a specified volume of blood.

A nurse is providing preoperative teaching for a client regarding use of an incentive spirometer and should include what instructions? 1 "Inhale completely and exhale in short, rapid breaths." 2 "Inhale deeply through the spirometer, hold it as long as possible, and slowly exhale." 3 "Exhale completely; take a slow, deep breath; hold it as long as possible, and slowly exhale." 4 "Exhale halfway, then inhale a rapid, small breath; repeat several times."

2 "Inhale deeply through the spirometer, hold it as long as possible, and slowly exhale." The correct procedure to maximize use of an incentive spirometer is to exhale completely, then take a slow, deep breath through the spirometer, and hold it as long as possible. This procedure will maximize inspiratory function by expanding the lungs. The client should practice using the incentive spirometer before surgery.

While receiving a preoperative enema, a client starts to cry and says, "I'm sorry you have to do this messy thing for me." What is the nurse's best response? 1 "I don't mind it." 2 "You seem upset." 3 "This is part of my job." 4 "Nurses get used to this."

2 "You seem upset."

A client is scheduled to receive conscious sedation during a colonoscopy. The client asks the nurse, "How will they 'knock me out' for this procedure?" Which answer by the nurse correctly describes the route of administration for conscious sedation? 1 "You will receive the anesthesia through a face mask." 2 "You will receive medication through an intravenous catheter." 3 "We will give you an oral medication about one hour before the procedure." 4 "The nurse anesthetist will inject the medication into the epidural space of your spine."

2 "You will receive medication through an intravenous catheter."

A nurse manager is evaluating the performance of the LPN/LVN who is supervising Unlicensed Assistive Personnel (UAP). What action indicates to the nurse manager that the LPN/LVN needs further instruction? 1 Requests that the UAP take vital signs on the clients assigned to their team. 2 Asks the UAP to assess the client's response to a respiratory treatment 3 Instructs the UAP to communicate to a client that the meal trays will be delayed. 4 Collaborates with the UAP to determine the best time to ambulate a client.

2 Asks the UAP to assess the client's response to a respiratory treatment

An adult client presents to the Emergency Department with a nosebleed. After applying pressure, what is the next nursing action? 1 Obtain a medication history from the client 2 Check the blood pressure 3 Instruct the client to avoid picking the nose 4 Check the pulse

2 Check the blood pressure Nosebleeds can be indicative of high blood pressure in an adult. Of the choices provided, the first action of the nurse should be to check the client's blood pressure. If elevated, the nurse can initiate measures to decrease the blood pressure.

A client who is to have brain surgery has a signed advance directive in the medical record. In what situation should this document be used? 1 Discharge planning is not covered by insurance. 2 Client cannot consent to his or her own surgery. 3 Postoperative complications occur that require additional treatment. 4 Client death and which client's belongings are to be given to family members.

2 Client cannot consent to his or her own surgery. Advance directives allow clients to designate another person to consent to procedures if they are unable to do so.

What clinical finding does a nurse anticipate when admitting a client with an extracellular fluid volume excess? 1 Rapid, thready pulse 2 Distended jugular veins 3 Elevated hematocrit level 4 Increased serum sodium level

2 Distended jugular veins Because of fluid overload in the intravascular space, the neck veins become visibly distended. Rapid, thready pulse and elevated hematocrit level occur with a fluid deficit. If sodium causes fluid retention, its concentration is unchanged; if fluid is retained independently of sodium, its concentration is decreased.

A client has a paracentesis, and the health care provider removes 1500 mL of fluid. To monitor for a serious postprocedure complication, the nurse should assess for: 1 Dry mouth 2 Tachycardia 3 Hypertensive crisis 4 Increased abdominal distention

2 Tachycardia

What are the desired outcomes that the nurse expects when administering a nonsteroidal antiinflammatory drug (NSAID)? (Select all that apply.) 1 Diuresis 2 Pain relief 3 Antipyresis 4 Bronchodilation 5 Anticoagulation 6 Reduced inflammation

2 Pain relief 3 Antipyresis 6 Reduced inflammation

A nurse is reviewing the laboratory report of a client with kidney problems. When ammonia is excreted by healthy kidneys, what mechanism usually is maintained? 1 Osmotic pressure of the blood 2 Acid-base balance of the body 3 Low bacterial levels in the urine 4 Normal red blood cell production

2 Acid-base balance of the body The excreted ammonia combines with hydrogen ions in the glomerular filtrate to form ammonium ions, which are excreted from the body. This mechanism helps rid the body of excess hydrogen, maintaining acid-base balance. - Osmotic pressure of the blood and normal red blood cell production are not affected by excretion of ammonia. - Ammonia is formed by the decomposition of bacteria in the urine; ammonia excretion is not related to the process and does not control bacterial levels

Which nursing intervention is most appropriate for a client in skeletal traction? 1 Add and remove weights as the client desires. 2 Assess the pin sites at least every shift and as needed. 3 Ensure that the knots in the rope are tied to the pulley. 4 Perform range of motion to joints proximal and distal to the fracture at least once a day.

2 Assess the pin sites at least every shift and as needed. Nursing care for a client in skeletal traction may include assessing pin sites every shift and as needed. - The needed weight for a client in skeletal traction is prescribed by the physician, not as desired by the client. - The nurse also should ensure that the knots are not tied to the pulley and move freely. - The performance of range of motion is indicated for all joints except the ones proximal and distal to the fracture because this area is immobilized by the skeletal traction to promote healing and prevent further injury and pain.

A nurse is caring for a client admitted with cardiovascular disease. During the assessment of the client's lower extremities, the nurse notes that the client has thin, shiny skin, decreased hair growth, and thickened toenails. The nurse understands that this may indicate: 1 Venous insufficiency 2 Arterial Insufficiency 3 Phlebitis 4 Lymphedema

2 Clients suffering from arterial insufficiency present with pale colored extremities when elevated and dusky red colored extremities when lowered. Lower extremities may also be cool to touch, pulses may be absent or mild, and skin may be shiny, thin, with decreased hair growth, and thickened nails. Clients suffering from venous insufficiency often have normal colored extremities, normal temperature, normal pulses, marked edema, and brown pigmentation around ankles. Phlebitis is an inflammation of a vein that occurs most often after trauma to the vessel wall, infection, and immobilization. Lymphedema is swelling in one or more extremities that is a direct result from impaired flow of the lymphatic system.

A client is hospitalized for treatment of severe hypertension. Captopril and alprazolam are prescribed. Shortly after admission, the client says, "I don't think any of you know what you are doing. You are just guessing what I need." What does the nurse determine as the probable cause of this behavior? 1 Denial of illness 2 Fear of the health problem 3 Response to cerebral anoxia 4 Reaction to the antihypertensive drug

2 Fear of the health problem Clients adapting to illness frequently feel afraid and helpless and strike out at health team members as a way of maintaining control or denying their fear. There is no evidence that the client denies the existence of the health problem. Although disorders such as brain attacks and atherosclerosis, which are associated with hypertension, may lead to cerebral anoxia, there is insufficient evidence to support this conclusion. Captopril (an antihypertensive) is a renin-angiotensin antagonist that reduces blood pressure and does not cause behavioral changes; alprazolam is prescribed to reduce anxiety.

A nurse is caring for a client who has paraplegia as a result of a spinal cord injury. Which rehabilitation plan will be most effective for this client? 1 Arrangements will be made by the client and the client's family. 2 The plan is formulated and implemented early in the client's care. 3 The rehabilitation is minimal and short term because the client will return to former activities. 4 Arrangements will be made for long-term care because the client is no longer capable of self-care.

2 To promote optimism and facilitate smooth functioning, rehabilitation planning should begin on admission to the hospital. The client and family often are unaware of the options available in the health care system; the nurse should be available to provide the necessary information and support. Rehabilitation helps a client adjust to a new lifestyle that must compensate for the paralysis. The goal of rehabilitation is to foster independence wherever the client may live after discharge.

A physician orders heparin 6,000 units subcutaneously daily. The pharmacy dispenses a vial containing 10,000 units per milliliter. To ensure the patient's safety, how many milliliters of heparin should the nurse administer? Include a leading zero if applicable. Record your answer using one decimal place. ______ mL

0.6 mL

A client is to receive a transfusion of packed red blood cells (PRBCs). The nurse should prepare for the transfusion by priming the blood IV tubing with which solution? Lactated Ringer solution 5% dextrose and water 0.9% normal saline 0.45% normal saline

0.9% normal saline

A nurse is assessing a child who is accompanied by a parent. The parent has remarried and has another child from the second marriage. What kind of a family does this child belong to? 1 Blended family 2 Extended family 3 Alternative family 4 Single-parent family

1 Blended family The child belongs to a blended family. Such a family is formed when parents bring unrelated children from prior relationships into a new, joint living situation. - Extended family comprises the husband, wife, children, uncles, aunts, cousins, and grandparents. - An alternative family may have grandparents caring for grandchildren. It may also be a multi-adult household with cohabiting partners or homosexual couples. - A single-parent family is formed when one parent cares for the children following the death, divorce, or desertion of the other parent. A single person may also decide to have or adopt a child.

An adolescent that had an inguinal hernia repair is being prepared for discharge home. The nurse provides instructions about resumption of physical activities. Which statement by the adolescent indicates that the client understands the instructions? 1 "I can ride my bike in about a week." 2 "I don't have to go to gym class for 3 months." 3 "I can't perform any weightlifting for at least 3 weeks." 4 "I can never participate in football again."

3 "I can't perform any weightlifting for at least 3 weeks."

When delegating input and output (I&O) measurement to nursing assistive personnel, a nurse instructs them to record what information for ice chips? 1. The total volume 2. Two-thirds of the volume 3. One-half of the volume 4. One-quarter of the volume

3. One-half of the volume

Tachypnea

The rate of breathing is regular, but abnormally rapid

Hyperpnea

The respirations are labored, the depth is increased, and the rate is increased.

What is the earliest sign of tissue deoxygenation?

Unexplained restlessness

A nurse is providing preoperative teaching for a client regarding use of an incentive spirometer and should include what instructions? 1 "Inhale completely and exhale in short, rapid breaths." 2 "Inhale deeply through the spirometer, hold it as long as possible, and slowly exhale." 3 "Exhale completely; take a slow, deep breath; hold it as long as possible, and slowly exhale." 4 "Exhale halfway, then inhale a rapid, small breath; repeat several times."

"Exhale completely; take a slow, deep breath; hold it as long as possible, and slowly exhale." The correct procedure to maximize use of an incentive spirometer is to exhale completely, then take a slow, deep breath through the spirometer and hold it as long as possible. This procedure will maximize inspiratory function by expanding the lungs. The client should practice using the incentive spirometer before surgery. When teaching clients, it is important to provide exact step-by-step instructions, thus not leaving out any critical points.

A client admitted to the hospital with a diagnosis of malabsorption syndrome exhibits signs of tetany. The nurse concludes that the tetany was precipitated by the inadequate absorption of which electrolyte? 1 Sodium 2 Calcium 3 Potassium 4 Phosphorus

2 Calcium The muscle contraction-relaxation cycle requires an adequate serum calcium-phosphorus ratio; the reduction of the ionized serum calcium level associated with malabsorption syndrome causes tetany (spastic muscle spasms).

A client asks the nurse, "Should I tell my partner that I just found out I'm human immunodeficiency virus (HIV) positive?" What is the nurse's most appropriate response? 1 "This is a decision you alone can make." 2 "Do not tell your partner unless asked." 3 "You are having difficulty deciding what to say." 4 "Tell your partner that you don't know how you became sick."

3 "You are having difficulty deciding what to say."

Which degree of edema will result in a 6-mm deep indentation upon pressure application? Multiple choice question 4+ 3+ 2+ 1+

3+ The depth of pitting determines the degree of pitting edema. An indentation of 6 mm is scored to be a 3+ degree edema. An indentation of 8 mm is scored as 4+. An indentation of 4 mm is scored as 2+. An indentation of 2 mm is scored as 1+.

A nurse assesses a client with dry and brittle hair, flaky skin, a beefy-red tongue and bleeding gums. The nurse recognizes that these clinical manifestations are most likely a result of: 1 A food allergy. 2 Noncompliance with medications. 3 Side effects from medications. 4 A nutritional deficiency.

4 A nutritional deficiency.

An infant weighs 7 lb at birth. How much should the nurse expect the infant to weigh at age 6-months? A. 12 lb. B. 17 lb. C. 14 lb. D. 21 lb

Answer: 14 lb. Infancy growth spurts double the birthweight by 4 to 6 months and triple it by one year. Twelve pounds (A) represents a lower-than-expected weight. A weight of 17 (C) or 21 (D) pounds is greater than expected.

The healthcare provider prescribes an IV infusion of 0.9% sodium chloride with 40 mEq KCl/500 ml to infuse over 3 hours for a client with hypokalemia. The nurse should program the infusion pump to deliver how many ml/hour? (Enter numeric value only. If rounding is required, round to the nearest whole number.)

Answer: 167 ml/hr To determine ml/hour: 500 ml : 3 hours :: as X ml : 1 hour 500/X :: 3/1 500 = 3X X = 166.66 (rounds to)= 167 ml/hour

When caring for a client with a fractured hip, the nurse should place pillows around the injured leg to specifically maintain what? 1 Abduction 2 Adduction 3 Traction 4 Elevation

Abduction Abduction means to move the limb away from the median plane, or axis, of the body. In care of the client with a fractured hip, the legs and hip must be aligned in an abducted position to prevent internal rotation, reduce the risk of dislocation, and decrease pain. In a client with a fractured hip, adduction of the limb, traction, and elevation are not appropriate procedures. Adduction means to move the limbs toward the medial plane, or axis, of the body, and traction involves the process of applying a pulling force in opposite directions using weights.

The charge nurse assigns the care of a client with diabetes who has hyperglycemia to a practical nurse (PN). In supervising the PN, what is the charge nurse's most important action? A. Confer with the PN about any manifestations the client is exhibiting. B. Decide which sliding scale insulin dose should be administered. C. Obtain the blood sugar results via skin puncture and glucometer. D. Notify the healthcare provider of the daily serum glucose results.

Answer: Confer with the PN about any manifestations the client is exhibiting. The nurse's expertise is needed to perform a critical assessment, such as assessing the client for signs of hyperglycemia and to supervise the ongoing monitoring of the client by the PN (D). (A, B, and C) are tasks which the PN can perform.

Bradypnea

breathing rate is regular, but it is abnormally slow

Which pulse site is used for the Allen's test? Multiple choice question Ulnar Popliteal Brachial Femoral

Ulnar The ulnar site is used for the Allen's test. The popliteal pulse is used to assess status of circulation to lower leg. The status of the circulation in the lower arm and blood pressure are assessed using the brachial pulse. The femoral pulse is used to assess the character of the pulse during physiological shock or cardiac arrest when other pulses are not palpable.

Which class of antiinfective drugs is contraindicated for use in children under 8 years of age? A. Quinolones. B. Tetracyclines. C. Penicillins. D. Aminoglycosides.

Answer: . Tetracyclines Tetracyclines (B) cause enamel hypoplasia and tooth discoloration in children under 8 years of age. (A, C, and D) are not contraindicated for use in children.

The nurse plans to administer labetalol hydrochloride (Trandate) 0.3 grams PO to a client with hypertension. Trandate is available in 200 mg scored tablets. How many tablets should the nurse administer? (Enter numeric value only.)

Answer: 1.5 tablets First convert grams to milligrams using the known conversion: 1 gram = 1000 mg. 0.1 gram = 100 mg as 0.3 gram = 300 mg Next using the formula, Desired/Available x 1 tablet = 300 mg/200 mg x 1 = 1.5 tablets

An IV infusion of 0.9% normal saline 500 ml with ammonium chloride 0.2 mEq/ml is prescribed for a client who was admitted for an amphetamine overdose. How many mEq of ammonium chloride should the nurse use to prepare the solution? (Enter numeric value only. If rounding is required, round to the nearest tenth.)

Answer: 100 mEq 0.2 mEq x 500 ml = 100 mEq

Asthma

reduced diameter of airways, bronchospasm. *Use of albuterol (Beta-2 Adrenergic Receptor agonist - triggers a response) = bronchodilator). Affects Beta-1 Adrenergic Receptors in the heart also - increase pulse, BP. Wouldn't give a Beta-1 Blocker (for example hypertension) to someone who needs a Beta-2 agonist. The blocking of Beta-1 would also block some of the Beta-2 agonist effect.

The findings of four clients who underwent eye examinations are given below. Which client is suspected to have sustained injury to the cranial nerve III? Client A: Drooping eyelids Client B: Nearsightedness Client C: Cross-eyes Client D: Protruding eyes

Client A: Drooping eyelids Injury to the third cranial nerve may result in edema or impairment of the third cranial nerve. This results in the abnormal drooping of the eyelids, a condition called ptosis. Myopia is nearsightedness, a refractive error in which rays of light enter the eye and focus in front of the retina. Cross-eyes result from strabismus, which results from neuromuscular injury or congenital anomaly. Protruding eyes (exophthalmoses) is indicative of hyperthyroidism.

What clinical indicators should the nurse expect a client with hyperkalemia to exhibit? Multiple selection question Tetany Seizures Diarrhea Weakness Dysrhythmias

-Diarrhea -Weakness -Dysrhythmias Tetany is caused by hypocalcemia. Seizures caused by electrolyte imbalances are associated with low calcium or sodium levels. Because of potassium's role in the sodium/potassium pump, hyperkalemia will cause diarrhea, weakness, and cardiac dysrhythmias.

When performing a postoperative assessment, which parameter would alert the nurse to a common side effect of epidural anesthesia? 1 Decreased blood pressure 2 Increased oral temperature 3 Diminished peripheral pulses 4 Unequal bilateral breath sounds

1 Decreased blood pressure The most important side effect to monitor in a client who has received epidural anesthesia is hypotension due to autonomic nervous system blockade. Therefore, in the immediate postoperative recovery period, the blood pressure should be assessed frequently. Other side effects include bradycardia, nausea, and vomiting. Increased oral temperature and unequal bilateral breath sounds are not effects associated with epidural anesthesia. Diminished peripheral pulses may result from hypotension, although they are not the most common side effects.

A nurse on the medical-surgical unit tells other staff members, "That client can just wait for the lorazepam (Ativan); I get so annoyed when people drink too much." What does this nurse's comment reflect? 1 Demonstration of a personal bias. 2 Problem solving based on assessment. 3 Determination of client acuity to set priorities. 4 Consideration of the complexity of client care.

1 Demonstration of a personal bias.

A nurse is teaching a community group about the basics of nutrition. A participant questions why fluoride is added to drinking water. The nurse should respond that it is a necessary element added to drinking water to promote: 1 Dental health. 2 Growth and development. 3 Improved hearing. 4 Night vision.

1 Dental health.

The nurse is preparing discharge instructions for a client who was prescribed enalapril maleate (Vasotec) for treatment of hypertension. Which is appropriate for the nurse to include in the client's teaching? 1 Do not change positions suddenly. 2 Light-headedness is a common adverse effect that need not be reported. 3 The medication may cause a sore throat for the first few days. 4 Schedule blood tests weekly for the first 2 months.

1 Do not change positions suddenly. Vasotec (enalapril) is classified as an ACE Inhibitor. ACE stands for angiotensin-converting enzyme. Vasotec is used to treat high blood pressure (hypertension) and congestive heart failure. It can also be used to treat a disorder of the ventricles. Angiotensin is a chemical that causes the arteries to become narrow. ACE inhibitors help the body produce less angiotensin, which helps the blood vessels relax and open up, which, in turn, lowers blood pressure. Clients should be advised to change position slowly to minimize orthostatic hypotension. A healthcare provider should be notified immediately if the client is experiencing light-headedness or feeling like he or she is about to faint, as this is a serious side effect.

The nurse assesses an edematous client and recalls that edema occurs in what extracellular fluid compartment? 1 Interstitial 2 Intercellular 3 Intravascular 4 Intracellular

1 Interstitial Edema is defined as the accumulation of fluid in the interstitial spaces. The incorrect answer options occur in other compartments: intercellular means between or among cells; intravascular means within a vascular space; and intracellular means within a cell.

An arterial blood gas report indicates the client's pH is 7.25, PCO2 is 35 mm Hg, and HCO3 is 20 mEq/L. Which disturbance should the nurse identify based on these results? 1 Metabolic acidosis 2 Metabolic alkalosis 3 Respiratory acidosis 4 Respiratory alkalosis

1 Metabolic acidosis

A nurse understands that the primary purpose for a client to undergo reconstructive surgery is to: 1 Restore function and/or appearance. 2 Replace an organ or tissue. 3 Relieve or reduce symptoms. 4 Remove or excise an organ or tissue.

1 Restore function and/or appearance.

A client has received instructions to take 650 mg aspirin (ASA) every 6 hours as needed for arthritic pain. What should the nurse include in the client's medication teaching? (Select all that apply.) 1 Take the aspirin with meals or a snack. 2 Make an appointment with a dentist if bleeding gums develop. 3 Do not chew enteric-coated tablets. 4 Switch to Tylenol (acetaminophen) if tinnitus occurs. 5 Report persistent abdominal pain

1 Take the aspirin with meals or a snack 3 Do not chew enteric-coated tablets. 5 Report persistent abdominal pain

When trying to promote effective learning in a client with a newly diagnosed disease, what should the nurse consider? 1 Client's past experiences 2 Client's personal resources 3 Stress of the total situation 4 Type of onset of the disease

1 Client's past experiences Past experiences have the most meaningful influence on present learning. Although the client's personal resources, the stress of the total situation, and the type of onset of the disease affect learning, their influence is not as great as past experiences.

Arrange the order of donning personal protective equipment (PPE) while caring for a client with isolation precautions.

1. Apply the cover gown, pull the sleeves down to the wrists, and tie the gown securely at the neck and waist 2.Apply either a surgical mask or a respirator around the mouth and nose 3.Apply eyewear or goggles snugly around the face and eyes 4.Apply clean gloves within the gown 5.Bring the glove cuffs over the edge of the gown sleeves

11. A patient has severe hypercalcemia. What are the priority nursing interventions? (Select all that apply.) 1. Fall prevention interventions 2. Teaching regarding sodium restriction 3. Encouraging increased fluid intake 4. Monitoring for constipation 5. Explaining how to take daily weights

1. Fall prevention interventions 3. Encouraging increased fluid intake 4. Monitoring for constipation

An intravenous (IV) fluid is infusing more slowly than ordered. The infusion pump is set correctly. Which factors could cause this slowing? (Select all that apply.) 1. Infiltration at vascular access device (VAD) site 2. Patient lying on tubing 3. Roller clamp wide open 4. Tubing kinked in bedrails 5. Circulatory overload

1. Infiltration at vascular access device (VAD) site 2. Patient lying on tubing 4. Tubing kinked in bedrails

A client is admitted with a sudden onset of dyspnea and chest pain. What are the interventions in the order in which the nurse will perform them to provide comfort to the client?

1. Notify the rapid response team 2. Reassuring the client and family members (helps stabilize the client) 3. Elevating the head of the bed 4. Preparing for oxygen therapy and blood gas analysis 5. Monitoring and assessing for other changes

Normal Red Cell Distribution Width (RDW)

11% - 15% A measurement of the variability of red cell size and shape.

A client is admitted to the hospital after an accident. The nurse uses the Glasgow Coma Scale (GCS) with the client. The client is alert and opens his or her eyes when there is a sound or when someone talks. When questions are asked, the client answers in a confused manner. The client obeys commands, such as being asked to move a leg. What would be the client's total score? Record your answer using a whole number.

13 The Glasgow Coma Scale (GCS) is used to measure the level of a client's consciousness and assigns a numerical score for each area of neurological status. The score for opening eyes on sound or speech is a 3. The score assigned for confused verbal responses is a 4. A score of 6 is assigned to the motor response of obeying commands. Therefore, the total score of the client is 13.

When the nurse arrives at 8:00 am, a client has a 1000 mL bag of D5W hanging, with 450 mL infused during the prior shift. The IV infusion is to deliver 100 mL per hour. At 11:00 am the healthcare provider changes the prescription for the intravenous solution to 1000 mL 0.9% sodium chloride to be administered at 75 mL per hour and changes the dietary order from nothing by mouth to clear liquids. From 1:00 pm to the end of the 12-hour shift at 8:00 pm, the client has 4 oz (120 mL) of apple juice, a half cup of tea, a half cup of gelatin, and 6 oz (180 mL) of water. How many milliliters should the nurse document as the client's total fluid intake for the 12-hour shift? Record your answer using a whole number.

1515 ml

A client is to receive 2000 mL of intravenous (IV) fluid in 12 hours. At what rate should the nurse set the electronic infusion control device? Record the answer using a whole number. ______ mL/hr

167 mL/hr

After several weeks of caring for clients who are in the terminal stage of illness, the nurse becomes aware of feeling depressed when coming to work. What should the nurse do? 1 Talk with other nurses on the unit. 2 Take several personal days off from work. 3 Limit emotional involvement with the clients. 4 Request a transfer to another area of the hospital.

1Talking with nurses who cope with similar issues allows the nurse to share feelings and obtain constructive emotional support. Avoidance may provide an immediate solution, but it works only for a short time. The nurse will eventually have to work through feelings. Limiting emotional involvement with the clients avoids personal feelings about death and dying and is an unacceptable attitude when caring for dying clients. Emotional withdrawal may be perceived by the clients as rejection. Avoidance may provide an immediate solution, but it works only for a short time. The nurse will eventually have to work through feelings.

The nurse understands that the action of an antidiuretic hormone (ADH) is to: 1 Reduce blood volume. 2 Decrease water loss in urine. 3 Increase urine output. 4 Initiate the thirst mechanism.

2 Decrease water loss in urine. ADH is released by the posterior pituitary gland. It is mainly released in response to a decrease in blood volume, or an increased concentration of sodium or other substances in plasma. It acts to decrease the production of urine by increasing the reabsorption of water by renal tubules. A decrease in ADH would cause reduced blood fluid volume, decreased ability of the kidneys to reabsorb water resulting in increased urine output, and an increase in the thirst mechanism

A client with Type I Diabetes complains of hunger, thirst, tiredness, and frequent urination. Based on these findings, the nurse should take what action? 1 Notify the physician immediately about the client's symptoms. 2 Determine the client's blood glucose level. 3 Administer the client's prescribed insulin. 4 Give the client a peanut butter and graham cracker snack.

2 Determine the client's blood glucose level.

A nurse is caring for a client who is receiving an intravenous (IV) infusion. What should the nurse do first if the IV infusion infiltrates? 1 Elevate the IV site. 2 Discontinue the infusion. 3 Attempt to flush the tubing. 4 Apply a warm, moist compress.

2 Discontinue the infusion. When an IV infusion infiltrates, it should be removed to prevent edema and pain. Elevation does not change the position of the IV cannula; the infusion must be discontinued. Flushing the tubing will add to the infiltration of fluid. Soaks may be applied, if prescribed, after the IV cannula is removed.

A visitor in the waiting room of the emergency department has a syncopal episode and collapses on the floor. The event is witnessed by a nurse, who provides initial care. The nurse assessed the client, maintained safety of the environment, and gave a report to the emergency department nurse, who will provide ongoing care. What should the nurse who witnessed the event do next? 1 Contact the family 2 Document the incident 3 Report the incident to the nurse manager 4 Escort the client to the radiology department

2 Document the incident

A client tells the nurse that the client's chest tube is scheduled to be removed soon. Before it is removed, the nurse is aware that: 1 The drainage system will be disconnected from the chest tube. 2 A chest x-ray will be performed to determine lung re-expansion. 3 An arterial blood gas will be obtained to determine oxygenation status. 4 The client will be sedated 30 minutes before the procedure.

2 A chest x-ray should be performed to ensure and to document that the lung is re-expanded and has remained expanded. The drainage system should not be disconnected from the actual chest tube while still in the client because this may cause a pneumothorax to recur. An arterial blood gas may be performed prior to removal but is not necessary. An oxygen saturation reading with a pulse oximeter is usually sufficient to determine oxygenation level. The client may be given pain medication before the procedure but not sedation, as this may decrease the oxygen status.

A day after an explanation of the effects of surgery to create an ileostomy, a 68-year-old male client remarks to the nurse, "It will be difficult for my wife to care for a helpless old man." This comment by the client regarding himself is an example of Erikson's conflict of:1 Initiative versus guilt 2 Integrity versus despair 3 Industry versus inferiority 4 Generativity versus stagnation

2 According to Erikson, poor self-concept and feelings of despair are conflicts manifested in those who are older than 65 years of age. The initiative versus guilt conflict is manifested in early childhood between 3 and 6 years of age. The industry versus inferiority conflict is manifested during the ages from 6 to 11 years. The generatively versus stagnation conflict is manifested during middle adulthood, 45 to 65 years of age.

A hospitalized client experiences a fall after climbing over the bed's side rails. Upon reviewing the client's medical record, the nurse discovers that restraints had been prescribed but were not in place at the time of the fall. What information should the nurse include in the follow-up incident report? 1 A statement that the nursing staff was not at fault because the client initiated the accident. 2 A listing of facts related to the incident as witnessed by the nurse. 3 The name of the nurse who was responsible for implementing the restraints. 4 The potential reasons why the restraints were not in place at the time of the fall.

2 The nurse filling out an incident or variance report needs to state only the objective facts surrounding the incident, no opinion or speculation. In an incident report fault or blame is subjective and should not be implied. It is not necessary to include names except for those of witnesses. Speculations or opinions as to the reason why the ordered restraints were not on the client are subjective and not appropriate to include in an incident or variance report.

A child is to receive 60 mg of phenytoin (Dilantin). The medication is available as an oral suspension that contains 125 mg/5 mL. How many milliliters should the nurse administer? Record the answer using one decimal place. ______ mL

2.4 mL

A client is being treated for Influenza A (H1N1). The nurse has provided instructions to the client about how to decrease the risk of transmission to others. Which patient statement indicates a need for further instruction/clarification? 1 "I should practice respiratory hygiene/cough etiquette." 2 "I should avoid contact with the elderly or children." 3 "I should obtain a pneumococcal vaccination each year." 4 "I should allow visitors for short periods of time only."

3 "I should obtain a pneumococcal vaccination each year."

A client has a right above-the-knee amputation after trauma sustained in a work-related accident. Upon awakening from surgery, the client states, "What happened to me? I don't remember a thing." What is the nurse's best initial response? 1 "Tell me what you think happened." 2 "You will remember more as you get better." 3 "You were in a work-related accident this morning." 4 "It was necessary to amputate your leg after the accident."

3 "You were in a work-related accident this morning."

A client has undergone a subtotal thyroidectomy. The client is being transferred from the post anesthesia care unit/recovery area to the inpatient nursing unit. What emergency equipment is most important for the nurse to have available for this client? 1 A defibrillator 2 An IV infusion pump 3 A tracheostomy tray 4 An electrocardiogram (ECG) monitor

3 A tracheostomy tray The client who has undergone a subtotal thyroidectomy is at high risk for airway occlusion resulting from postoperative edema. With this in mind, emergency airway equipment such as a tracheostomy set and intubation supplies should be immediately available to the client. A defibrillator, an IV infusion pump, and an electrocardiogram (ECG) monitor are all equipment items that should be available to all postoperative clients.

A client that is scheduled for a surgical resection of the colon and creation of a colostomy for a bowel malignancy asks why preoperative antibiotics have been prescribed. The nurse explains that the primary purpose is to: 1 Decrease peristalsis. 2 Minimize electrolyte imbalance. 3 Decrease bacteria in the intestines. 4 Treat inflammation caused by the malignancy.

3 Decrease bacteria in the intestines. To decrease the possibility of contamination, the bacteria count in the colon is lowered with antibiotics before surgery. Preoperative antibiotics do not have an effect on peristalsis, electrolyte balance, or treating inflammation.

An assessment of the client on total parenteral nutrition (TPN) reveals a bounding pulse, distended jugular veins, dyspnea, and cough. What is the priority nursing intervention? 1 Ask the registered nurse start the client's infusion at a peripheral site 2 Slow the rate of the client's infusion of the TPN 3 Interrupt the client's infusion and notify the charge nurse or health care provider 4 Obtain the vital signs and continue monitoring the client's status

3 Interrupt the client's infusion and notify the charge nurse or health care provider The client is experiencing pulmonary edema because of a fluid volume excess. The high concentration of TPN precipitates a fluid shift from the interstitial compartment into the intravascular compartment. Fluid will continue to be infused, which will continue to increase the intravascular volume. TPN is not infused in a peripheral IV and the rate is not to be changed by the LPN, especially without a health care provider's order.

The nurse is caring for a client who is on a low carbohydrate diet. With this diet, there is decreased glucose available for energy, and fat is metabolized for energy resulting in an increased production of which substance in the urine? 1 Protein 2 Glucose 3 Ketones 4 Uric acid

3 Ketones As a result of fat metabolism, ketone bodies are formed and the kidneys attempt to decrease the excess by filtration and excretion. Excessive ketones in the blood can cause metabolic acidosis. A low carbohydrate diet does not cause increased protein, glucose, or uric acid in the urine.

The nurse should monitor for which involuntary physiological response in a client who is experiencing pain? 1 Crying 2 Splinting 3 Perspiring 4 Grimacing

3 Perspiring Perspiration is an involuntary physiological response. It is mediated by the autonomic nervous system under a variety of circumstances, such as rising ambient temperature, high humidity, stress, and pain

A client is admitted voluntarily to a psychiatric unit. Later, the client develops severe pain in the right lower quadrant and is diagnosed as having acute appendicitis. How should the nurse prepare the client for the appendectomy? 1 Have two nurses witness the client signing the operative consent form. 2 Ensure that the health care provider and the psychiatrist sign for the surgery because it is an emergency procedure. 3 Ask the client to sign the operative consent form after the client has been informed of the procedure and required care. 4 Inform the client's next of kin that it will be necessary for one of them to sign the consent form because the client is on a psychiatric unit.

3 Because the client is not certified as incompetent, the right of informed consent is retained. The client can sign the consent, but the client's signature requires only one witness. Because there is no evidence of incompetence, the client should sign the consent.

A home health nurse on a first visit checks the client's vital signs and obtains a blood sample for an international normalization ratio (INR). After completion of these tasks, the client asks the nurse to straighten the blankets on the bed. What is the nurse's most appropriate response? 1 "I would, but my back hurts today." 2 "Okay. It will be my good deed for the day." 3 "Of course. I want to do whatever I can for you." 4 "I would like to, but it is not in my job description."

3 Helping the client to meet physical needs is within the role of the nurse; arranging blankets on the client's bed is an appropriate intervention. The nurse's comfort needs should not take precedence over the client's needs; the nurse should not assume responsibility for the role of care provider if incapable of providing care. It is not a good deed but fulfills the expected role of the nurse; this response sounds grudgingly compliant. This is within the nurse's job description.

The nurse discovers several palpable elevated masses on a client's arms. Which term most accurately describes the assessment findings? 1 Erosions 2 Macules 3 Papules 4 Vesicles

3 Papules are superficial and elevated up to 0.5 cm. Nodules and tumors are masses similar to papules but are elevated more than 0.5 cm and may infiltrate deeper into tissues. Erosions are characterized as loss of the epidermis layer; macules are nonpalpable, flat changes in skin color less than 1 cm in diameter; and vesicles are usually transparent, filled with serous fluid, and are a blisterlike elevation.

Heparin 20,000 units in 500 ml D5W at 50 ml/hour has been infusing for 5½ hours. How much heparin has the client received? A. 13,000 units. B. 11,000 units. C. 17,000 units. D. 15,000 units

Answer: 11,000 units (A) is the correct calculation: 20,000 units/500 ml = 40 units (the amount of units in one ml of fluid). 40 units/ml x 50 ml/hr = 2,000 units/hour (1,000 units in 1/2 hour). 5.5 x 2,000 = 11,000 (A). OR, multiply 5 x 2,000 and add the 1/2 hour amount of 1,000 to reach the same conclusion = 11,000 units.

At a support meeting of parents of a teenager with polysubstance dependency, a parent states, "Each time my son tries to quit taking drugs, he gets so depressed that I'm afraid he will commit suicide." The nurse's response should be based on which information? A. Feelings of depression frequently lead to drug abuse and addiction. B. Careful monitoring should be provided during withdrawal from the drugs. C. Addiction is a chronic, incurable disease. D. Tolerance to the effects of drugs causes feelings of depression.

Answer: Careful monitoring should be provided during withdrawal from the drugs. The priority is to teach the parents that their son will need monitoring and support during withdrawal (B) to ensure that he does not attempt suicide. Although (A and C) are true, they are not as relevant to the parent's expressed concern. There is no information to support (D).

A client with hyperlipidemia receives a prescription for niacin (Niaspan). Which client teaching is most important for the nurse to provide? A. Symptoms of hyperglycemia. B. Expected duration of flushing. C. Diets that minimize GI irritation. D. Comfort measures for pruritis.

Answer: Expected duration of flushing Flushing of the face and neck, lasting up to an hour, is a frequent reason for discontinuing niacin. Inclusion of this effect in client teaching (A) may promote compliance in taking the medication. While (B, C, and D) are all worthwhile instructions to help clients minimize or cope with normal side effects associated with niacin (Niaspan), flushing is intense and causes the most concern for the client.

A client with gastroesophageal reflux disease (GERD) has been experiencing severe reflux during sleep. Which recommendation by the nurse is most effective to assist the client? A. Raising the head of the bed on blocks. B. Avoiding large meals. C. Decreasing caffeine intake. D. Losing weight.

Answer: Raising the head of the bed on blocks. Raising the head of the bed on blocks (D) (reverse Trendelenburg position) to reduce reflux and subsequent aspiration is the most effective recommendation for a client experiencing severe gastroesophageal reflux during sleep. (A, B and C) may be effective recommendations but raising the head of the bed is more effective for relief during sleep.

While assessing the pupils of a client, a healthcare professional notices pupillary dilatation. Which drug intake might have resulted in this condition? Multiple choice question Heroin Atropine Morphine Pilocarpine

Atropine The intake of eye medications such as atropine will cause dilatation of the pupils. Heroin, morphine, and pilocarpine cause pupillary constriction.

While caring for a client with heat stroke, the nurse measured the temperature and noted it as 39ºC. What is this temperature in Fahrenheit? Round your answer to one decimal place

Celsius is converted to Fahrenheit by multiplying the Celsius reading by 9/5 and adding the product to 32. In this case, the calculation is: (9/5)(39) + 32 = 102.2.

Which integumentary finding is related to skin texture? Multiple choice question Elasticity Vascularity Fluid buildup Character of the surface

Character of the surface Assessing for texture refers to the character of the surface of the skin. Assessing for elasticity determines the turgor of the skin. Assessing for vascularity determines skin circulation. Fluid buildup in the tissues indicates edema.

Which nursing interventions would be beneficial for providing safe oxygen therapy? Select all that apply.

Checking the tubes for kinks Posting "no smoking" signs in the client's room

What finding would be consistent with long-standing hypoxemia in a client who reports shortness of breath?

Clubbing

While assessing the nails of a client with diabetes, the nurse finds that the skin on the client's hands and feet are dry due to infection. What could be the reason for this dryness? Multiple choice question Applying moisturizing lotion between toes Cutting nails after soaking them for 10 minutes in warm water Cutting nails straight across and even with the tops of the fingers or toes Using sharp objects to poke or dig under the toenail or around the cuticle

Cutting nails after soaking them for 10 minutes in warm water Normally, nails should be cut after soaking them in warm water for 10 minutes. This action should not be performed for diabetic patients because soaking the nails will dry out the hands and feet, which may lead to infection. Applying moisturizing lotion between the toes will promote microorganism growth; it will not dry the skin. Cutting nails straight across and even with the tops of the fingers or toes is the proper way to maintain nail hygiene. Diabetic clients are advised not to use sharp objects to poke or dig under the toenails or around the cuticles to avoid injury to the skin.

Which of the following stimulates the thirst center in the hypothalamus? A. High blood pressure B. Decreased release of angiotensin II C. Release of antidiuretic hormone from the pituitary gland D. Decreased blood volume

D. Decreased blood volume

Which intrinsic factors may contribute to falls in older adults? Select all that apply.

Deconditioning Impaired Vision

The nurse assesses a client's pulse and documents the strength of the pulse as 3+. The nurse understands that this pulse can be characterized as what? Multiple choice question Diminished Normal Full Bounding

Full The strength of a pulse is a measurement of the force at which blood is ejected against the arterial wall. A 3+ rating indicates a full increased pulse. A zero rating indicates an absent pulse. A rating of a 1+ indicates a diminished pulse that is barely palpable. A 2+ rating is an expected or normal pulse, and a 4+ rating is a bounding pulse.

Which error will result in false high diastolic readings while measuring a client's blood pressure during a physical examination? Multiple choice question Inflating the cuff too slowly Wrapping the cuff too loosely Applying the stethoscope too firmly Repeating the assessment too quickly

Inflating the cuff too slowly Which error will result in false high diastolic readings while measuring a client's blood pressure during a physical examination?

What clinical finding does a nurse anticipate when admitting a client with an extracellular fluid volume excess? Rapid, thready pulse Distended jugular veins Elevated hematocrit level Increased serum sodium level

JVD

A client shows an increase in rate respirations that are abnormally deep and regular. What condition would the nurse expect?

Kussmaul's respiration

The nurse is caring for an African American client with renal failure. The client states that the illness is a punishment for sins. Which cultural health belief does the client communicate? Multiple choice question Yin/Yang balance Biomedical belief Determinism belief Magicoreligious belief

Magicoreligious belief An African American client may have magicoreligious beliefs, which focuses on hexes or supernatural forces that cause illness. Such clients may believe that illness is a punishment for sins. The yin/yang belief system does not consider illness as a punishment. The biomedical belief system maintains that health and illness are related to physical and biochemical processes with disease being a breakdown of the processes. The belief of determinism focuses on outcomes that are externally preordained and cannot be changed.

Which physical assessment findings of a client suspected of having a respiratory disorder would be considered normal? Select all that apply.

Midline Trachea Pink Nasal Mucosa Nonlabored Respirations @ 14breaths/min

What interventions should the nurse perform while caring for an actively dying client? Select all that apply. Admit the client in hospice care. Perform aggressive laboratory tests. Provide client and family reassurance. Keep the client undisturbed for long time. Perform symptom management in the client.

Provide client and family reassurance. Perform symptom management in the client.

What assessment findings indicate that a client is experiencing an allergic reaction to antibiotic therapy? Select all that apply.

Pruritis Wheezing Bronchospasm

When assessing a client's fluid and electrolyte status, the nurse recalls that the regulator of extracellular osmolarity is what? Sodium Potassium Chloride Calcium

Sodium

A healthcare provider prescribes a standard walker (pick-up walker with rubber tips on all four legs). The nurse identifies what clinical findings that indicate the client is capable of using a standard walker? 1 Weak upper arm strength and impaired stamina 2 Weight bearing as tolerated and unilateral paralysis 3 Partial weight bearing on the affected extremity and kyphosis 4 Strong upper arm strength and non-weight bearing on the affected extremity

Strong upper arm strength and non-weight bearing on the affected extremity A walker with four rubber tips on the legs requires more upper body strength than a rolling walker. A client who is non-weight bearing on the affected extremity is able to use a standard walker. A rolling walker is more appropriate for a client with weak upper arm strength and impaired stamina who is less able to lift up and move a walker with four rubber tips. A client with unilateral paralysis is not a candidate for a standard walker; the client must be able to grip and lift the walker with both upper extremities and move the walker forward. A rolling walker is more appropriate for this client. A client with kyphosis is less able to lift up and move a walker with four rubber tips.

An elderly adult with Parkinson's disease falls while going to the bathroom and gets injured. The nurse taking care of the client informs the primary healthcare provider. What step should the nurse take to alert the risk management system?

The nurse should document the incident in the occurrence report tool.

Venturi Mask

can adjust oxygen amount with much more precision - for patients with chronic lung disease.

A client has been admitted with a diagnosis of intractable vomiting and can only tolerate sips of water. The initial blood work shows a sodium level of 122 mEq/L (122 mmol/L) and a potassium level of 3.6 mEq/L (3.6 mmol/L). Based on the lab results and symptoms, what is the client experiencing? Hypernatremia Hyponatremia Hyperkalemia Hypokalemia

hyponatremia

The nurse is questioning a client who reports pain. Which questions asked by the nurse are appropriate? Select all that apply. 1 "Where does it hurt?" 2 "What makes the pain worse?" 3 "How long have you noticed it?" 4 "Have you been treated for pain previously?" 5 "How severe is your pain on a scale of 0 to 10?"

"Where does it hurt?" "What makes the pain worse?" "How long have you noticed it?" "How severe is your pain on a scale of 0 to 10?" The nurse should follow an orderly and systematic approach when collecting information. The nurse should ask specific questions of the client such as "Where does it hurt?", "What makes the pain worse?", "How long you have noticed it?" and "How severe is your pain on a scale of 0 to 10?" Questions such as "Have you been treated for pain previously?" should be asked after understanding the characteristics of the pain.

While assessing an older adult during a regular health checkup, a nurse finds signs of elder abuse. Which physical findings would further confirm the nurse's suspicion? Multiple selection question Presence of hyoid bone damage Presence of cognitive impairment Presence of burns from cigarettes Presence of bed sores. Presence of unexplained bruises on the wrist(s)

-Presence of burns from cigarettes -Presence of bed sores. -Presence of unexplained bruises on the wrist(s) A physical finding of abuse in older adults can be the presence of burns from cigarettes. The physical presence of bed sores also indicates client abuse. Unexplained bruises on the wrist(s) may also be an indication of abuse in older adults. The presence of hyoid bone damage is an indication of intimate partner violence. The presence of cognitive impairment is a behavioral finding in older adult abuse.

The count of hydrocodone (Vicodin) is incorrect. After several minutes of searching the medication cart and Physiological Aspects of Care records, no explanation is found. Who should the primary nurse notify about the discrepancy? 1 Nursing unit manager 2 Hospital administrator 3 Quality control manager 4 Health care provider prescribing the medication

1 Nursing unit manager Controlled substance issues for a particular nursing unit are the responsibility of that unit's nurse manager. Responsibility flows directly from the staff of a nursing unit to the nurse manager; the nurse manager reports to a nurse administrator.

Which physiologic changes may occur during the first trimester of pregnancy? Select all that apply. 1 Fatigue 2 Increased libido 3 Morning sickness 4 Breast enlargement 5 Braxton Hicks contractions

1, 3, 4 Fatigue, morning sickness, and breast enlargement are observed during the first trimester of pregnancy. - Increased libido is observed during the second trimester of pregnancy. - Braxton Hicks contractions are observed during the third trimester of pregnancy.

A client with an abdominal wound infected with methicillin-resistant Staphylococcus aureus (MRSA) is scheduled for a computed tomography (CT) scan of the abdomen. To ensure client and visitor safety during transport, the nurse should implement which precaution? 1 No special precautions are required. 2 Cover the infected site with a dressing. 3 Drape the client with a covering labeled as biohazardous. 4 Place a surgical mask on the client.

2 Cover the infected site with a dressing.

The nurse is caring for a client that is hyperventilating. The nurse recalls that the client is at risk for: 1 Respiratory acidosis 2 Respiratory alkalosis 3 Respiratory compensation 4 Respiratory decompensation

2 Respiratory alkalosis Hyperventilation causes excess amounts of carbon dioxide (CO2) to be eliminated, causing respiratory alkalosis.

A client is to receive 125 mL of intravenous (IV) fluid every hour. The drop factor of the IV tubing is 10 gtt/mL. How many drops per minute should the nurse administer? Record your answer using a whole number. ______ gtts/min.

21 gtts/min

To minimize the side effects of the vincristine (Oncovin) that a client is receiving, what type of dietary plan does the nurse expect? 1 Low in fat 2 High in iron 3 High in fluids 4 Low in residue

3 High in fluids

9. An older-adult patient is receiving intravenous (IV) 0.9% NaCl. A nurse detects new onset of crackles in the lung bases. What is the priority action? 1. Notify a health care provider 2. Record in medical record 3. Decrease the IV flow rate 4. Discontinue the IV site

3. Decrease the IV flow rate

A client who has congestive heart failure with paroxysmal atrial tachycardia is receiving digoxin (Lanoxin) 0.45 mg IV as the inital digitalizing dose. The pharmacy provides 0.25 mg/ml. How many ml should the nurse administer? (Enter numeric value only. If rounding is required, round to the nearest tenth.)

Answer: 1.8 ml Using the formula D/H x Q 0.45 mg/0.25 mg x 1 mL = 1.8 mL.

A child who weighs 32 pounds receives a prescription for amoxicillin with clavulanate (Augmentin) 194 mg PO every 8 hours. The recommended dose is 40 mg/kg/day, and the bottle contains 125 mg/5ml. How many mg should the child receive in a 24-hour period? (Enter numeric value only. If rounding is required, round to the nearest whole number.)

Answer: 582 The child's weight in kg (2.2 : 1 kg :: 32 pounds : X kg = 14.54 kg) is used to calculate the recommended dose (40 mg x 14.54 kg/day = 581.6 = 582 mg). The prescribed dose in 24 hours is 194 mg x 3 doses = 582 mg.

The healthcare provider prescribes digitalis (Digoxin) for a client diagnosed with congestive heart failure. Which intervention should the nurse implement prior to administering the digoxin? A. Monitor the serum glucose level. B. Assess the serum potassium level. C. Observe respiratory rate and depth. D. Obtain the client's blood pressure.

Answer: Assess the serum potassium level Hypokalemia (decreased serum potassium) will precipitate digitalis toxicity in persons receiving digoxin (B). (A and C) will not affect the administration of digoxin. (D) should be monitored if he/she is a diabetic and is perhaps receiving insulin.

An infant in respiratory distress is placed on pulse oximetry. The oxygen saturation indicates 85%. What is the priority nursing intervention? A. Place the infant under a radiant warmer. B. Begin humidified oxygen via hood. C. Stimulate infant crying. D. Evaluate the blood pH.

Answer: Begin humidified oxygen via hood. An oxygen saturation of less than 90% (normal oxygen saturation is 96% to 98%) requires oxygen administration (B). (A) is not necessary. (C) may utilize additional oxygen and will not correct the problem. (D) is important because it may decrease energy use for respiratory effort, but it will not correct a low saturation level.

The charge nurse working in a long-term care facility is informed by the LPN that a client's son is unhappy with the care his mother is receiving. What action should the nurse take first? A. Discuss with the LPN the son's concerns about his mother's care. B. Ask the family member to come to the nurses' station to discuss the concerns. C. Provide the son with a complaint form and ask him to describe the situation. D. Notify the administrator of the long-term care facility about the son's discontent.

Answer: Discuss with the LPN the son's concerns about his mother's care. The nurse should first obtain information about the nature of the complaint and ask the LPN to describe what he/she knows of the situation. (C) should be the nurse's first action. (A, B, and D) may all need to be implemented after (C).

A 48-year-old client is experiencing a severe anaphylactic reaction to an injection of contrast media. What pharmacologic agent is of greatest use in this situation? A. Epinephrine (Adrenalin). B. Nitroprusside (Nipride). C. Dopamine (Intropin). D. Loratadine (Claritin).

Answer: Epinephrine (Adrenalin). Epinephrine (D) is the drug of choice in treating hypotension and circulatory failure associated with anaphylaxis because it is a potent vasoconstrictor. An anaphylactic reaction is an acute systemic hypersensitivity reaction that occurs within minutes of antigen exposure (such as with contrast material containing iodine) that can result in peripheral vascular collapse. (A) may eventually be necessary if the client does not respond to initial treatment of hypotension with epinephrine. Antihistamines, including (B), are useful adjunctive therapies. (C) is contraindicated.

Which task should the nurse delegate to an Unlicensed Assistive personnel (UAP)? A. Feed a client with minimal dysphagia. B. Determine a client's response to pain. C. Observe a client's central venous catheter site. D. Accompany the healthcare provider during client visits.

Answer: Feed a client with minimal dysphagia. Delegation of client care is delineated by state boards of nursing practice and include specific guidelines regarding which tasks are within the scope of practice for each level of care provider and include the components of delegation to the UAP. Feeding a client (D) is a basic client care measure that is within the scope of practice for a UAP. (A, B, and C) require assessment and analysis which require the expertise of a licensed nurse.

A medication that is classified as a beta-1 agonist is most commonly prescribed for a client with which condition? A. Glaucoma. B. Hypertension. C. Heart failure. D. Asthma.

Answer: Heart failure. Beta-1 agonists improve cardiac output by increasing the heart rate and blood pressure and are indicated in heart failure (C), shock, atrioventricular block dysrhythmias, and cardiac arrest. Glaucoma (A) is managed using adrenergic agents and beta-adrenergic blocking agents. Beta-1 blocking agents are used in the management of hypertension (B). Medications that stimulate beta-2 receptors in the bronchi are effective for bronchoconstriction in respiratory disorders, such as asthma (D).

The nurse is assessing a client's intelligence. Which factor should the nurse remember during this part of the mental status exam? A. Poor concentration skills suggests limited intelligence. B. The inability to think abstractly indicates limited intelligence. C, Acute psychiatric illnesses impair intelligence. D. Intelligence is influenced by social and cultural beliefs.

Answer: Intelligence is influenced by social and cultural beliefs. Social and cultural beliefs (D) have significant impact on intelligence. Chronic psychiatric illness may impair intelligence (C), especially if it remains untreated. Limited concentration does not suggest limited intelligence (A). Difficulties with abstractions are suggestive of psychotic thinking (B), not limited intelligence.

A healthcare provider prescribes cefadroxil (Duricef) for a client with a postoperative infection. It is most important for the nurse to assess for what additional drug allergy before administering this prescription? A. Penicillins. B. Aminoglycosides. C. Sulfonamides. D. Erythromycins.

Answer: Penicillins. Cross-allergies exist between penicillins (A) and cephalosporins, such as cefadroxil (Duricef), so checking for penicillin allergy is a wise precaution before administering this drug.

In assessing a client diagnosed with primary hyperaldosteronism, the nurse expects the laboratory test results to indicate a decreased serum level of which substance? A. Glucose. B. Sodium. C. Antidiuretic hormone. D. Potassium.

Answer: Potassium Clients with primary aldosteronism exhibit a profound decline in the serum levels of potassium (D) (hypokalemia)--hypertension is the most prominent and universal sign. (B) is normal or elevated, depending on the amount of water reabsorbed with the sodium. (A) is decreased with diabetes insipidus. (C) is not affected by primary aldosteronism.

The nurse is triaging a child with a fever brought to the emergency department by the parents. Which finding requires the nurse's immediate intervention? A. Frequent nonproductive cough. B. Prolonged exhalations. C. Oxygen saturation is 95% by pulse oximeter. D. Thick yellow rhinorrhea.

Answer: Prolonged exhalations. Prolonged exhalation (A) indicates breathing difficulty, and intervention for this should be taken immediately. Nasal discharge (B) and a productive cough (C) are not findings that indicate the child is in immediate distress. An oxygen saturation of 95% is a normal finding (D).

A 46-year-old female client is admitted for acute renal failure secondary to diabetes and hypertension. Which test is the best indicator of adequate glomerular filtration? A. Urine specific gravity. B. Serum creatinine. C. Blood Urea Nitrogen (BUN). D. Sedimentation rate.

Answer: Serum creatinine Creatinine (A) is a product of muscle metabolism that is filtered by the glomerulus, and blood levels of this substance are not affected by dietary or fluid intake. An elevated creatinine strongly indicates nephron loss, reducing filtration. (B) is also an indicator of renal activity, but it can be affected by non-renal factors such as hypovolemia and increased protein intake. (C) is a nonspecific test for acute or chronic inflammatory processes. (D) is useful in assessing hydration status, but not as useful in assessing glomerular function.

An antacid (Maalox) is prescribed for a client with peptic ulcer disease. The nurse knows that the purpose of this medication is to A. maintain a gastric pH of 3.5 or above. B. decrease gastric motor activity. C. produce an adherent barrier over the ulcer. D. decrease production of gastric secretions.

Answer: maintain a gastric pH of 3.5 or above The objective of antacids is to neutralize gastric acids and keep pH of 3.5 or above (C) which is necessary for pepsinogen inactivity. (A) is the purpose of H2 receptor antagonists (cimetidine, ranitidine). (B) is the purpose of sucralfate (Carafate). (D) is the purpose of anticholinergic drugs which are often used in conjunction with antacids to allow the antacid to remain in the stomach longer.

The nurse is caring for a client who survived a severe burn injury. Which action should the nurse perform immediately based on priority?

Assessing airway patency

The nurse assesses an elderly client with a diagnosis of dehydration and recognizes which finding as an early sign of dehydration? Sunken eyes Dry, flaky skin Change in mental status Decreased bowel sounds

Change in mental status

A child who reports shortness of breath, wheezing, and coughing is found to have pulmonary edema and is prescribed furosemide. Which nursing interventions would be beneficial to the client? Select all that apply.

Checking the child's weight every day Calculating the dose as carefully as possible Assessing the child regularly to prevent electrolyte loss

While assessing a client, a nurse finds that the ratio of the anteroposterior diameter and transverse diameter of the chest is 1:1. What is indicated by this finding? Multiple selection question Client has lordosis. Client is an older adult. Client has osteoporosis. Client has a history of smoking. Client has chronic lung disease.

Client is an older adult. Client has a history of smoking. Client has chronic lung disease. The 1:1 ratio of the anteroposterior diameter and transverse diameter of the chest indicates a barrel-shaped chest. This is a characteristic feature in an older adult who smokes and has chronic lung disease. In lordosis, there is an increase in lumbar curvature. Osteoporosis is a systemic skeletal condition in which there is a decreased bone mass and deterioration of bone tissue.

A client who experienced extensive burns is receiving intravenous fluids to replace fluid loss. The nurse should monitor for which initial sign of fluid overload? Crackles in the lungs Decreased heart rate Decreased blood pressure Cyanosis

Crackles in the lungs

While assessing the client's skin, a nurse notices a skin condition, the pathophysiology of which involves increased visibility of oxyhemoglobin caused by an increased blood flow due to capillary dilation. Which condition is associated with this client? Multiple choice question Pallor Vitiligo Cyanosis Erythema

Erythema Erythema occurs due to an increased visibility of oxyhemoglobin, which is caused by increased blood flow. Pallor is caused by a reduced amount of oxyhemoglobin or a reduced visibility of oxyhemoglobin. Vitiligo is a pigmentation disorder caused by autoimmune diseases. Cyanosis is a bluish discoloration of the skin around the lips; this occurs due to an increased amount of deoxygenated hemoglobin in the blood.

An older adult experiencing delirium suffers from a leg fracture caused by a fall. Which interventions should the nurse follow to prevent future falls? Select all that apply.

Minimizing medications Modifying the home environment Manage foot & footwear problems

A client is admitted for surgery. Although not physically distressed, the client appears apprehensive and withdrawn. What is the nurse's best action? 1 Orient the client to the unit environment. 2 Have a copy of hospital regulations available. 3 Explain that there is no reason to be concerned. 4 Reassure the client that the staff is available if the client has questions.

Orient the client to the unit environment. Orienting the client to the hospital unit provides knowledge that may reduce the strangeness of the environment. Having a copy of hospital regulations available is part of orienting the client to the unit. This alone is not enough when orienting a client to the hospital. Explaining that there is no reason to be concerned may be false reassurance because no one can guarantee that there is no reason to be concerned. Reassuring the client that the staff is available to answer questions implies that staff members are available only if the client has specific questions.

When assessing a client, the nurse auscultates a murmur at the second left intercostal space (ICS) along the sternal border. This reflects sound from which valve? Multiple choice question Aortic Mitral Pulmonic Tricuspid

Pulmonic The second left intercostal space (ICS) along the sternal border reflects sounds from the pulmonic valve. The correct landmark for auscultating the aortic valve is at the right second ICS at the sternal border; for the mitral valve (apical pulse) at the left fifth ICS in the midclavicular line; and for the tricuspid valve at the left fifth ICS at the sternal border.

A nurse in the emergency department is assessing a young child with a head injury. The child is accompanied by a parent. Which observation should prompt the nurse to assess the child for abuse? 1 The child has Mongolian spots on the back. 2 The child belongs to a single-parent family. 3 The child has received care for injuries twice earlier. 4 The child and parent narrate the same story about the injury.

The child has received care for injuries twice earlier. The nurse should assess the child for abuse if the child has received care for injuries on two earlier occasions. Frequent emergency visits for injuries should prompt the nurse to further investigate the case. Mongolian spots are normal variants of skin coloration obtained at birth and do not need further evaluation. A single-parent home does not indicate that the child is a victim of abuse. The nurse need not assess the child for abuse if both the parent and the child narrate the same story.

Jaw Thrust Maneuver

The recommended procedure for opening the airway of an unconscious client with a possible spinal or neck injury.

After assessing a client's breath sounds, the nurse suspects bronchospasm. Which adventitious breath sound has prompted the nurse's suspicion?

Wheezing

Polycythemia vera

characterized by brown spots on the skin caused by increased melanin production.

A nurse is caring for a client with diarrhea. The nurse anticipates a decrease in which clinical indicator? Pulse rate Tissue turgor Specific gravity Body temperature

tissue turgor

Which response by the nurse during a client interview is an example of back channeling? Multiple choice question "All right, go on..." "What else is bothering you?" "Tell me what brought you here." "How would you rate your pain on a scale of 0 to 10?"

"All right, go on..." Back channeling involves the use of active listening prompts such as "Go on...", "all right", and "uh-huh." Such prompts encourage the client to complete the full story. The nurse uses probing by asking the client, "What else is bothering you?" Such open-ended questions help to obtain more information until the client has nothing more to say. The statement, "Tell me what brought you here" is an open-ended statement that allows the client to explain his health concerns in his or her own words. Closed-ended questions such as, "How would you rate your pain on a scale of 0 to 10?" are used to obtain a definite answer. The client answers by stating a number to describe the severity of pain.

An older client asks, "How do I know that all the medications that I take are safe?" What information should the nurse include in response to this client's question? Select all that apply. 1 "Ask your healthcare provider how and when you should be taking your medications." 2 "Stop taking a prescribed medication if you are not feeling better in a few days." 3 "Discard medications into the toilet that have exceeded the expiration date on the bottle." 4 "Check the name, dose, and instructions about administration of drugs each time before leaving the pharmacy." 5 "Inform your healthcare provider of the over-the-counter drugs, recreational drugs, and amount of alcohol you ingest."

"Ask your healthcare provider how and when you should be taking your medications." "Check the name, dose, and instructions about administration of drugs each time before leaving the pharmacy." "Inform your healthcare provider of the over-the-counter drugs, recreational drugs, and amount of alcohol you ingest." If unsure about any information, the client should be encouraged to ask for further instructions and more information. A client needs to be proactive and should check all aspects of the prescription with the pharmacist before leaving the pharmacy. A pharmacist may have permission to substitute a generic form of the drug or may change the number of pills that deliver the prescribed dose, both of which can confuse the client (e.g., one tablet may deliver 50 milligrams of a drug and be equal to two 25-milligram tablets). Because of the risk of drug interactions associated with polypharmacy and altered age-related physiological functioning that can cause drug toxicity, the client should inform the health team about all drugs (e.g., prescription, over-the-counter, recreational), herbal preparations, and amount of alcohol ingested to ensure safety. A client should stop taking a prescribed medication only after consultation with the health care provider. Unused and expired medications should not be discarded into the toilet because they can contaminate groundwater.

A client presents with bilateral leg pain and cramping in the lower extremities. The client has a history of cardiovascular disease, diabetes, and varicose veins. To guide the assessment of the pain and cramping, the nurse should include which question when completing the initial assessment? Multiple choice question "Does walking for long periods of time increase your pain?" "Does standing without moving decrease your pain?" "Have you had your potassium level checked recently?" "Have you had any broken bones in your lower extremities?"

"Does walking for long periods of time increase your pain?" Clients with a medical history of heart disease, hypertension, phlebitis, diabetes, or varicose veins often develop vascular-related complications. The nurse should recognize that the relationship of symptoms to exercise will clarify whether the presenting problem is vascular or musculoskeletal. Pain caused by a vascular condition tends to increase with activity. Musculoskeletal pain is not usually relieved when exercise ends. Low potassium levels can cause cramping in the lower extremities; however, given the client's health history, vascular insufficiency should be suspected. Previously healed broken bones do not cause cramping and pain.

A registered nurse is teaching a nursing student about precautions to be taken for physical examination of a client. Which statements made by the nursing student indicate effective learning? Multiple selection question "I should examine the client in noise-free areas." "I should use latex gloves during the physical examination." "I should perform a physical examination in a cool room." "I should leave a combative client alone during a physical examination." "I should wear eye shields while examining a client with excessive drainage."

"I should examine the client in noise-free areas." "I should wear eye shields while examining a client with excessive drainage." Clients should be examined in noise-free areas to prevent interruptions. Wearing eye shields while examining a client with excessive drainage helps to reduce contamination. Latex gloves should be used with caution because they may cause allergy in clients who are allergic to latex. A physical examination should be performed in a warm room to minimize discomfort. Combative clients should never be left alone during physical examinations.

A registered nurse is teaching a nursing student about caring for a client before leaving the healthcare facility. Which statement made by the nursing student indicates the need for further education? 1 "I should teach the client about potential food-drug interactions." 2 "I should involve the client and his or her family in the referral process." 3 "I should give limited information about the client to the healthcare provider who received the referral." 4 "I should teach the client and his or her family about safe and effective use of medications and medical equipment."

"I should give limited information about the client to the healthcare provider who received the referral." The nurse should provide as much client information as possible to the healthcare provider who received the referral because this action helps to avoid the provider asking duplicate questions and helps to avoid the omission of important information. The nurse should instruct the client about any potential food-drug interactions. The nurse should involve the client and his or her family in the referral process. The nurse should teach the client about the safe and effective use of medications and medical equipment.

The registered nurse is teaching a nursing student about ways to minimize heat radiation. Which statements made by the nursing student indicate effective learning? Multiple selection question "I will apply an ice pack to the client." "I will cover the client with dark clothes." "I will instruct the client to remove extra clothes." "I will instruct the client to lie in the fetal position." "I will advise the client to wear sparsely woven clothes."

"I will apply an ice pack to the client." "I will cover the client with dark clothes." "I will instruct the client to lie in the fetal position." Applying an ice pack will increase conductive heat loss, which results in minimizing heat radiation. Wearing dark clothes and lying in the fetal position will minimize heat radiation. Removing extra clothes will increase heat radiation. Wearing sparsely woven clothes will enhance heat radiation.

While assessing a client's range of motion, the nurse explains adduction to the nursing student. Which statement made by the nursing student indicates effective learning? Multiple choice question "I will ask the client to move his or her arm towards the body." "I will ask the client to bend his or her limb by decreasing the angle." "I will ask the client to move his or her hand so that the ventral surface faces downward." "I will ask the client to move his or her head beyond its normal resting extended position."

"I will ask the client to move his or her arm towards the body." Adduction is moving the arm towards the body. Assessing the range of motion by bending the limb and decreasing the angle indicates flexion. Moving the hand by facing the ventral surface downwards indicates pronation. The movement of the head beyond the normal resting extended position indicates hyperextension.

The nurse manager asks the nurse, "How would you implement clinical decision making in a group of clients?" Which answer provided by the nurse shows effective critical thinking? Select all that apply. 1 "I will avoid involving clients as decision-makers and participants in care." 2 "I will discuss complex cases with other members of the healthcare team." 3 "I will identify the nursing diagnoses and collaborative problems of each client." 4 "I will consider the period it takes to care for clients whose problems have higher priority." 5 "I will decide to perform activities individually to resolve more than one client problem at a time."

"I will discuss complex cases with other members of the healthcare team." "I will identify the nursing diagnoses and collaborative problems of each client." "I will consider the period it takes to care for clients whose problems have higher priority." The nurse should discuss complex cases with the other members of the healthcare team. It ensures a smooth transition in the care requirements. As a part of effective critical thinking, the nurse should diagnose the collaborative problems of each client. The nurse should consider the care time for the clients having problems that require high priority. Effective critical thinking requires the nurse to involve clients as decision makers or participants in care. The nurse should decide on combining activities to resolve more than one client problem at a time.

The nurse manager asks the nurse, "How would you implement clinical decision making in a group of clients?" Which answer provided by the nurse shows effective critical thinking? Select all that apply. 1"I will avoid involving clients as decision-makers and participants in care."2"I will discuss complex cases with other members of the healthcare team."3"I will identify the nursing diagnoses and collaborative problems of each client."4"I will consider the period it takes to care for clients whose problems have higher priority."5"I will decide to perform activities individually to resolve more than one client problem at a time."

"I will discuss complex cases with other members of the healthcare team.""I will identify the nursing diagnoses and collaborative problems of each client.""I will consider the period it takes to care for clients whose problems have higher priority."

A nurse teaches a client about various measures to protect against food-borne illness. Which statement by the client indicates a need for further teaching? Multiple choice question "I'll clean the inside of my refrigerator and microwave regularly." "I'll wash my cooking utensils and cutting boards with tap water." "I'll wash my hands with warm, soapy water before touching or eating food." "I won't eat any leftovers in my refrigerator after they've been there for 5 days."

"I'll wash my cooking utensils and cutting boards with tap water." Eating leftovers that have been kept in a refrigerator for more than 2 days may result in a food-borne illness caused by microbial growth in the food. Cleaning the inside of the refrigerator and microwave regularly will help prevent microbial growth. Cooking utensils and cutting boards should be washed with hot, soapy tap water as a means of preventing food-borne illness. Washing the hands with warm, soapy water before touching or eating food is one technique for preventing food borne illness.

A registered nurse is teaching a nursing student how to assess for edema. Which statement made by the student indicates the need for further education? "Edema results in the separation of skin from pigmented and vascular tissue." "Pitting edema leaves an indentation on the site of application of pressure." "Trauma or impaired venous return should be suspected in clients with edema." "If the pressure on an edematous site leaves an indentation of 2 mm, a score of 2+ is given."

"If the pressure on an edematous site leaves an indentation of 2 mm, a score of 2+ is given."

The nurse administers a pneumococcal vaccine to a 70-year-old client. The client asks "Will I have to get this every year like I do with the flu shot?" How should the nurse respond? Multiple choice question "You need to receive the pneumococcal vaccine every other year." "The pneumococcal vaccine should be received in early autumn every year." "You should get the flu and pneumococcal vaccines at your annual physical examination." "It is unnecessary to have any follow-up injections of the pneumococcal vaccine after this dose."

"It is unnecessary to have any follow-up injections of the pneumococcal vaccine after this dose." The Centers for Disease Control and Prevention recommend that adults be immunized with pneumococcal vaccine at age 65 years or older with a single dose of the vaccine; if the pneumococcal vaccine was received before 65 years of age or if there is the highest risk of fatal pneumococcal infection, revaccination should occur 5 years after the initial vaccination. The pneumococcal vaccine should not be administered every 2 years. The pneumococcal vaccine should not be administered annually.

A client develops an allergic reaction when a student nurse is performing a physical assessment. Which statement made by the student nurse in response to this incident indicates the need for further teaching? Multiple choice question "Type I immune response to latex has an immediate onset." "Type I immune reaction to latex leads to release of IgE antibodies." "The client's first exposure to latex will cause a type IV allergic reaction." "Type IV immune response to latex occurs after 12 to 48 hours after exposure."

"The client's first exposure to latex will cause a type IV allergic reaction." Both type I and type IV hypersensitive reactions require prior exposure to cause an immune response in a subsequent exposure. The most immediate immune response is a type I reaction, in which the body produces IgE antibodies against the allergen. A type IV immune response occurs 12 to 48 hours after the exposure to the allergen and is referred to as a delayed hypersensitivity response.

The nurse is performing a breast assessment. Which statement made by the client indicates the risk of breast cancer? Multiple selection question "I had a late onset of menarche." "My first child was born when I was 32." "I noticed a slight discharge from a nipple." "I perform breast self-examinations frequently." "I consume two to four glasses of alcohol a day."

-"My first child was born when I was 32." -"I noticed a slight discharge from a nipple." -"I consume two to four glasses of alcohol a day." Clients who gave birth to a first child after the age of 30 are at a risk of breast cancer. Discharge from the nipple may indicate an early symptom of breast cancer. Consuming two to four glasses of alcohol daily may also increase the risk of breast cancer. An early onset of menarche is a risk factor for breast cancer. Performing breast self-examinations frequently may help to identify the early stages of breast cancer.

Which clients suffer from impaired near vision? Multiple selection question A client with myopia A client with presbyopia A client with hyperopia A client with retinopathy A client with macular degeneration

-A client with presbyopia -A client with hyperopia A loss of elasticity of the lens causes impaired near-vision in presbyopia. Light rays focusing behind the retina are the cause of impaired near vision in clients with hyperopia. Myopia is caused by a refractive error where the light rays focus in front of the retina. Retinopathy is a noninflammatory change in the retinal blood vessels. Macular degeneration is a blurring of central vision caused by progressive degeneration of the central retina.

Which actions by the nurse help set the stage for a patient-centered interview during the first visit after admission to the healthcare facility? Multiple selection question Close the door after entering the room. Greet the client using his or her last name. Open the curtains to allow plenty of light in the room. Introduce oneself with a smile and explain the reason for the visit. Obtain an authorization from the client after the interview.

-Close the door after entering the room. -Greet the client using his or her last name. -Introduce oneself with a smile and explain the reason for the visit. The nurse should maintain the client's privacy by closing the door after entering the room. The nurse should maintain the dignity of the client by greeting the client using his or her last name. Smiling is a positive sign of warmth and immediacy when first establishing the nurse-client relationship. The nurse should explain his or her role in the providing care for the client. The nurse should ensure the room is adequately lit, comfortable, and soothing for the client. The nurse need not open the curtains to allow plenty of light in the room. The Health Insurance Portability and Accountability Act (HIPAA) requires the nurse to obtain an authorization from the client before collecting personal health data.

An 85-year-old client has just been admitted to a nursing home. When designing a plan of care for this older adult, the nurse recalls what expected sensory losses associated with aging? Multiple selection question Difficulty in swallowing Diminished sensation of pain Heightened response to stimuli Impaired hearing of high frequency sounds Increased ability to tolerate environmental heat

-Diminished sensation of pain -Impaired hearing of high frequency sounds Because of aging of the nervous system, an older adult has a diminished sensation of pain and may be unaware of a serious illness, thermal extremes, or excessive pressure. As people age they experience atrophy of the organ of Corti and cochlear neurons, loss of the sensory hair cells, and degeneration of the stria vascularis, which affects an older person's ability to perceive high frequency sounds. An interference with swallowing is a motor loss, not a sensory loss, and it is not an expected response to aging. There is a decreased, not heightened, response to stimuli in older adults. There is a decreased, not increased, ability to physiologically adjust to extremes in environmental temperature.

A nurse is caring for an older adult with a hearing loss secondary to aging. What can the nurse expect to identify when assessing this client? Multiple selection question Dry cerumen Tears in the tympanic membrane Difficulty hearing high pitched voices Decrease of hair in the auditory canal Overgrowth of the epithelial auditory lining

-Dry cerumen -Difficulty hearing high pitched voices Cerumen (ear wax) becomes drier and harder as a person ages. Generally, female voices have a higher pitch than male voices; older adults with presbycusis (hearing loss caused by the aging process) have more difficulty hearing higher pitched sounds. There is no greater incidence of tympanic tears caused by the aging process. The hair in the auditory canal increases, not decreases. The epithelium of the lining of the ear becomes thinner and drier.

When should the nurse observe the client to assess his or her level of functioning? Multiple selection question During meal time When talking about pain When preparing medication During the assessment interview When administering insulin injections

-During meal time -When preparing medication -When administering insulin injections An observation of the functional level of the client often occurs during a return demonstration. The nurse may also observe the client while eating to determine if the client is able to eat without assistance. The nurse teaches the client how to prepare medications and asks for a return demonstration to assess the client's understanding. The nurse also observes the client administering insulin injections to ensure that the client is able to perform it properly. Observation of functional level differs from the observation during a physical examination. The nurse closely observes the client during the physical assessment when the client talks about pain. During the assessment interview, the nurse observes the client's facial expressions and eye contact to form accurate conclusions about the client's condition. The nurse does not assess the client's functional abilities during the subjective assessment.

The community nurse is assessing an elderly client who lives alone at home. The nurse finds that the client refrains from physical activity for fear of falling when walking. Which interventions by the nurse are most beneficial to promote a healthy lifestyle? Multiple selection question Instruct the client to apply bed side rails. Encourage the client to wear nonskid shoes. Suggest that the client use an assistive device. Ask the client to install hand rails in the bathroom. Help the client rearrange furniture in the house.

-Encourage the client to wear nonskid shoes. -Suggest that the client use an assistive device. -Help the client rearrange furniture in the house The nurse should encourage the client to wear nonskid shoes that will provide a firm grip while walking and help reduce the chance of falls. The nurse should suggest that the client use an assistive device such as a cane or walker for support while walking. The nurse should make environmental changes by helping the client rearrange the furniture in the house. This will help reduce the incidence of falls within the house. These interventions reduce the fear of falling and encourage the client to participate in physical activity indoors and outdoors. The bed side rails protect the client from falling from the bed. The hand rails in the bathroom assist provide support while using the bathroom.

A client with recent history of head trauma is at risk of orthostatic hypotension. Which assessment findings would help to diagnose the condition? Multiple selection question Fainting Headache Weakness Light headedness Shortness of breath

-Fainting -Weakness -Light headedness Head trauma may cause blood loss. Clients with recent blood loss are at risk of orthostatic hypotension. While obtaining the orthostatic measurements, the nurse should check for fainting, light-headedness, and weakness. Headaches and shortness of breath are the symptoms of hypertension.

A client with a recent history of head trauma is at risk for orthostatic hypotension. Which assessment findings observed by the nurse would relate to this diagnosis? Multiple selection question Fainting Headache Weakness Lightheadedness Shortness of breath

-Fainting -Weakness -Lightheadedness Head trauma may cause blood loss and clients with recent blood loss are at risk for orthostatic hypotension. Symptoms of hypotension include fainting, lightheadedness, and weakness. Headaches and shortness of breath are symptoms of hypertension.

What should the nurse do when the defining characteristics of assessment data for a client can apply to more than one diagnosis? Multiple selection question Reassess the client. Reject all diagnoses. Gather more information. Identify related factors. Review all defining characteristics.

-Gather more information. -Identify related factors. -Review all defining characteristics. The nurse must gather more information to clarify interpretations of assessment data. Correct interpretation of information allows the nurse to select the right diagnosis that applies to the client. A related factor is a condition or etiology that gives a context for the defining characteristics. The nurse should identify related factors to individualize a nursing diagnosis for the client. The nurse should review all the defining characteristics, eliminate irrelevant ones, and confirm the relevant ones. The nurse must interpret the data to form data clusters only after reassessing and validating it. At this stage, the nurse should have only validated assessment data in the database. The nurse need not reject all diagnoses. The nurse should review all the defining characteristics to support or eliminate the irrelevant ones.

A nurse is performing an eye assessment in an older adult. The older adult is unable to see near objects. Which conditions may be suspected in the older adult? Multiple selection question Cataract Glaucoma Hyperopia Presbyopia Macular degeneration

-Hyperopia -Presbyopia In hyperopia, the client has farsightedness. In this condition, the client is unable to see near objects. Presbyopia is an impaired near vision that may occur with aging. Therefore, the nurse can suspect either of the conditions. In cataracts, there is an increased opacity of the lens that blocks light rays from entering the eye, leading to impaired vision. Glaucoma is a condition in which there is intraocular structural damage resulting from elevated intraocular pressure. Macular degeneration is caused due to blurred central vision that often occurs suddenly. This is caused by a progressive degeneration of the center of the retina.

Which age-related change should the nurse consider when formulating a plan of care for an older adult? Multiple selection question Difficulty in swallowing Increased sensitivity to heat Increased sensitivity to glare Diminished sensation of pain Heightened response to stimuli

-Increased sensitivity to glare -Diminished sensation of pain Changes in the ciliary muscles, decrease in pupil size, and a more rigid pupil sphincter contribute to an increased sensitivity to glare. Diminished sensation of pain may make an older adult unaware of a serious illness, thermal extremes, or excessive pressure. There should be no interference with swallowing in older adults. Older adults tend to feel the cold and rarely complain of the heat. There is a decreased response to stimuli in older adults.

Which statement best describes a diagnostic label? Multiple choice question It is a condition that responds to nursing interventions. It describes the essence of the client's response to health conditions. It describes the characteristics of the client's response to health conditions. It is identified from the client's assessment data and associated with the diagnosis.

-It describes the essence of the client's response to health conditions. A diagnostic label is the name of the nursing diagnosis as approved by the North American Nursing Diagnosis Association International (NANDA-I). It describes the essence of the client's response to health conditions in as few words as possible. The etiology of a nursing diagnosis is a condition that responds to nursing interventions. All NANDA-I approved diagnoses have a definition that describes the characteristics of the client's response to health conditions. The related factor of a nursing diagnosis is identified from the client's assessment data and associated with the diagnosis.

A nurse is assessing a client with a history of marijuana use. Which long-term effects are associated with marijuana? Multiple selection question Lung cancer Emphysema Heart disease Laryngeal disorder Stroke Chronic nasal irritation

-Lung cancer -Emphysema -Heart disease Lung cancer, emphysema, and heart disease are outcomes that may occur due to marijuana use. Laryngeal disorders, stroke, and chronic nasal irritation are associated with the abuse of cocaine but are not associated with marijuana use.

Which features distinguish nursing diagnoses from medical diagnoses? Multiple selection question Nursing diagnoses involve the client when possible. Nursing diagnoses are based on results of diagnostic tests and procedures. Nursing diagnoses are the identification of a disease condition in the client. Nursing diagnoses involve the sorting of health problems within the nursing domain. Nursing diagnoses involve clinical judgment about the client's response to health problems.

-Nursing diagnoses involve the client when possible. -Nursing diagnoses involve the sorting of health problems within the nursing domain. -Nursing diagnoses involve clinical judgment about the client's response to health problems. Establishing a nursing diagnosis is the second step in the nursing process. It is unique and involves the client's participation in the process. Nursing diagnoses classify health problems to be treated primarily by nurses. The nurse reviews the client assessment, sees cues and patterns in the data, and identifies the client's specific health care problems. The nursing diagnosis is a clinical judgment about the client's actual or potential health problems that the nurse is licensed to treat. A medical diagnosis is based on results of diagnostic tests and procedures, whereas a nursing diagnosis is based on the results of the nursing assessment. A medical diagnosis identifies a disease condition in the client.

A nurse assesses the vital signs of a 50-year-old female client and documents the results. Which of the following are considered within normal range for this client? Multiple selection question Oral temperature of 98.2° F (36.8° C) Apical pulse of 88 beats per minute and regular Respiratory rate of 30 per minute Blood pressure of 116/78 mm Hg while in a sitting position Oxygen saturation of 92%

-Oral temperature of 98.2° F (36.8° C) -Apical pulse of 88 beats per minute and regular -Blood pressure of 116/78 mm Hg while in a sitting position The client's temperature, pulse, and blood pressure are within normal ranges for a 50-year-old female. The client's respirations are mildly elevated, and the oxygen saturation level is below normal. A normal respiratory rate for a female client in this age group would be 12 to 20 per minute, and oxygen saturation level should be 95%.

While demonstrating the method of measuring blood pressure to a student nurse, the registered nurse measures the blood pressure in a client as 130/80 mm Hg. After the demonstration, when the student nurse is measuring the blood pressure in the same client, it is found to be 120/90 mm Hg. What could be the possible reasons for this difference? Multiple selection question Poor fitting of the cuff Inflating the cuff too slowly Deflating the cuff too quickly Inflating the cuff inadequately Applying the stethoscope too firmly

-Poor fitting of the cuff -Deflating the cuff too quickly Poor fitting of the cuff or deflating the cuff too quickly causes false low systolic and false high diastolic readings. Inflating the cuff too slowly results in false high diastolic readings. Inflating the cuff inadequately yields false low systolic readings. Applying the stethoscope too firmly against antecubital fossa yields false low diastolic readings.

A client with a history of cardiac dysrhythmias is admitted to the hospital due to a fluid volume deficit caused by a pulmonary infection. The registered nurse is assessing the vital signs recorded by the student nurse. Which vital sign assessments require reassessment based on the data given by the student nurse? Multiple selection question Respiratory rate of 14 breaths/minute Blood pressure of 120/80 mmHg Oxygen saturation of 95% Temporal temperature of 37.4 °C Radial pulse rate of 72 and irregular

-Respiratory rate of 14 breaths/minute -Blood pressure of 120/80 mmHg -Oxygen saturation of 95% In pulmonary infections, the respiratory rate may increase and oxygen saturation may decrease. In fluid volume deficit, the blood pressure may be decreased. A respiratory rate of 14 breaths/minute, a blood pressure of 120/80 mmHg, and an oxygen saturation of 95% are normal readings. Therefore, the registered nurse should reassess these vital signs. The normal temperature range is 36 to 38 0C; this range is unaffected by a pulmonary infection. Therefore, the nurse does not need to reassess the temperature. Cardiac dysrhythmias are associated with a pulse deficit in which the radial pulse would be irregular. Therefore reassessment would not be required.

Which sites would be safe and inexpensive for temperature measurement? Multiple selection question Skin Oral Axilla Rectal Tympanic membrane

-Skin -Axilla The skin and axilla are safe and inexpensive sites of the body for temperature measurement. The oral route is an easily accessible site for temperature measurement but it may not be the safest route because of the exposure to body fluids. The rectal route may not be easily accessible and safe because a measurement via this route may increase the risk of body fluid exposure. The tympanic membrane is an easily accessible site for temperature measurement but care should be taken when used in neonates, infants, and children.

A nurse is assessing an older adult client. Which clinical findings are expected responses to the aging process? Multiple selection question Slowed neurologic responses Lowered intelligence quotient Long-term memory impairment Forgetfulness about recent events Reduced ability to maintain an erection

-Slowed neurologic responses -Forgetfulness about recent events -Reduced ability to maintain an erection Slowing of neurologic responses is part of the aging process. Memory for short-term situations and events is reduced. The ability of the male to attain and sustain an erection is reduced. There should not be a loss of intellectual ability. Memory of long-term experiences and events should not be impaired.

A registered nurse instructed the nursing assistive personnel (NAP) to measure the temperature of a client who reports chills and coldness. The nurse believes that the reading is inaccurate. What observations may have led to this conclusion? Multiple selection question The client has a habit of breathing through his or her mouth. The client smoked 40 minutes after his or her temperature was taken. The client ingested juice 20 minutes before his or her temperature was taken. The client ingested food 20 minutes after having his or her temperature was taken. The client ingested medications 10 minutes after having his or her temperature was taken.

-The client has a habit of breathing through his or her mouth. -The client ingested juice 20 minutes before his or her temperature was taken. Habitual mouth breathing may result in inaccurate temperature readings. A client who ingested any fluids or food orally or smoke should wait for 20 to 30 minutes his or her temperature was taken. Smoking, ingesting

During a physical assessment, a client was diagnosed with increased temperature due to an increased basal metabolic rate (BMR). Which hormonal imbalances may the client have? Multiple selection question Cortisol Thyroid Estrogen Testosterone Progesterone

-Thyroid -Testosterone Body temperature is assessed during physical assessment. An increased basal metabolism rate increases the body temperature. Hormonal imbalances may alter the basal metabolic rate (BMR). Testosterone regulates the BMR in males. Thyroid hormone regulates the BMR of the body. Increases in the levels of these hormones may increase the BMR, which may in turn raise body temperature. Cortisol regulates blood glucose levels. Estrogen and progesterone are female hormones that do not regulate the BMR.

A nurse is caring for an older adult who is taking acetaminophen (Tylenol) for the relief of chronic pain. Which substance is most important for the nurse to determine if the client is taking because it intensifies the most serious adverse effect of acetaminophen? 1 Alcohol 2 Caffeine 3 Saw palmetto 4 St. John's wort

1 Alcohol

A client reports nausea, vomiting, and seeing a yellow light around objects. A diagnosis of hypokalemia is made. Upon a review of the client's prescribed medication list, the nurse determines that what is the likely cause of the clinical findings? 1 Digoxin (Lanoxin) 2 Furosemide (Lasix) 3 Propranolol (Inderal) 4 Spironolactone (Aldactone)

1 Digoxin (Lanoxin)

After gastric surgery a client has a nasogastric tube in place. What should the nurse do when caring for this client? 1 Monitor for signs of electrolyte imbalance. 2 Change the tube at least once every 48 hours. 3 Connect the nasogastric tube to high continuous suction. 4 Assess placement by injecting 10 mL of water into the tube.

1 Monitor for signs of electrolyte imbalance. Gastric secretions, which are electrolyte rich, are lost through the nasogastric tube; the imbalances that result can be life threatening. Changing the nasogastric tube every 48 hours is unnecessary and can damage the suture line. High continuous suction can cause trauma to the suture line. Injecting 10 mL of water into the nasogastric tube to test for placement is unsafe; if respiratory intubation has occurred aspiration will result.

A client being treated for Influenza A (H1N1) is scheduled for a computed tomography (CT) scan. To ensure client and visitor safety during transport, the nurse should take which precaution? 1 Place a surgical mask on the client. 2 Other than Standard Precautions, no additional precautions are needed. 3 Minimize close physical contact. 4 Cover the client's legs with a blanket.

1 Place a surgical mask on the client. Nurses should provide influenza clients with face masks to wear for source control and tissues to contain secretions when outside of their room. Special precautions such as face masks should be taken to decrease the risk of further outbreak.

A health care provider prescribes an antibiotic intravenous piggyback (IVPB) twice a day for a client with an infection. The health care provider prescribes peak and trough levels 48 and 72 hours after initiation of the therapy. The client asks the nurse why there is a need for so many blood tests. The nurse's best response is, "These tests will: 1 determine adequate dosage levels of the drug." 2 detect if you are having an allergic reaction to the drug." 3 permit blood culture specimens to be obtained when the drug is at its lowest level." 4 allow comparison of your fever to when the blood level of the antibiotic is at its highest."

1 determine adequate dosage levels of the drug." Drug dosage and frequency are adjusted according to peak and trough levels to enhance efficacy by maintaining therapeutic levels. Peak and trough levels reveal nothing about allergic reactions. Blood cultures are obtained when the client spikes a temperature; they are not related to peak and trough levels of an antibiotic. A sustained decrease in fever is the desired outcome, not reduction just at peak serum levels of the medication.

When caring for a client with varicella and disseminated herpes zoster, the nurse should implement which types of precautions? (Select all that apply.) 1 Airborne 2 Contact 3 Droplet 4 Hazardous Wastes 5 Standard

1 Airborne 2 Contact 5 Standard Contact precautions are used for patients with known or suspected infections transmitted by direct contact or contact with items in the environment. Airborne precautions are used for clients known or suspected to have infections transmitted by the airborne transmission route. Varicella can be transmitted by airborne and contact.

What is a nurse's responsibility when administering prescribed opioid analgesics? (Select all that apply.) 1 Count the client's respirations. 2 Document the intensity of the client's pain. 3 Withhold the medication if the client reports pruritus. 4 Verify the number of doses in the locked cabinet before administering the prescribed dose. 5 Discard the medication in the client's toilet before leaving the room if the medication is refused.

1 Count the client's respirations. 2 Document the intensity of the client's pain. 4 Verify the number of doses in the locked cabinet before administering the prescribed dose.

A client with arthritis increases the dose of ibuprofen (Motrin, Advil) to abate joint discomfort. After several weeks the client becomes increasingly weak. The client is admitted to the hospital and is diagnosed with severe anemia. What clinical indicators does the nurse expect to identify when performing an admission assessment? (Select all that apply.) 1 Melena 2 Tachycardia 3 Constipation 4 Clay-colored stools 5 Painful bowel movements

1 Melena 2 Tachycardia Ibuprofen irritates the gastrointestinal (GI) mucosa and can cause mucosal erosion, resulting in bleeding; blood in the stool (melena) occurs as the digestive process acts on the blood in the upper GI tract.

The nurse manager is planning to assign unlicensed assistive personnel (UAP) to care for clients. What care can be delegated on a medical-surgical unit to UAP? (Select all that apply.) 1 Performing a bed bath for a client on bed rest. 2 Evaluating the effectiveness of acetaminophen and codeine (Tylenol #3). 3 Obtaining an apical pulse rate before oral digoxin (Lanoxin) is administered. 4 Assisting a client who has patient-controlled analgesia (PCA) to the bathroom. 5 Assessing the wound integrity of a client recovering from an abdominal laparotomy.

1 Performing a bed bath for a client on bed rest. 4 Assisting a client who has patient-controlled analgesia (PCA) to the bathroom. Performing a bed bath for a client on bed rest is within the scope of practice of UAP. Assisting a client who has PCA to the bathroom does not require professional nursing judgment and is within the job description of UAP. Evaluating human responses to medications requires the expertise of a licensed professional nurse. Obtaining an apical pulse rate requires a professional nursing judgment to determine whether or not the medication should be administered. Evaluating human responses to health care interventions requires the expertise of a licensed professional nurse.

A client has received instructions to take 650 mg aspirin (ASA) every 6 hours as needed for arthritic pain. What should the nurse include in the client's medication teaching? (Select all that apply.) 1 Take the aspirin with meals or a snack. 2 Make an appointment with a dentist if bleeding gums develop. 3 Do not chew enteric-coated tablets. 4 Switch to Tylenol (acetaminophen) if tinnitus occurs. 5 Report persistent abdominal pain.

1 Take the aspirin with meals or a snack. 3 Do not chew enteric-coated tablets. 5 Report persistent abdominal pain.

A nursing supervisor sends unlicensed assistive personnel (UAP) to help relieve the burden of care on a short-staffed medical-surgical unit. Which tasks can be delegated to UAP? (Select all that apply.) 1 Taking routine vital signs. 2 Applying a sterile dressing. 3 Answering clients' call lights. 4 Administering saline infusions. 5 Changing linens on an occupied bed. 6 Assessing client responses to ambulation.

1 Taking routine vital signs. 3 Answering clients' call lights. 5 Changing linens on an occupied bed.

On the second day of hospitalization a client is discussing with the nurse concerns about unhealthy family relationships. During the nurse-client interaction the client begins to talk about a job problem. The nurse's response is, "Let's go back to what we were just talking about." What therapeutic communication technique did the nurse use? 1 Focusing 2 Restating 3 Exploring 4 Accepting

1 Focusing Focusing is a technique that directs a client back to the original topic of discussion. - Restating the main idea of what the client has said encourages the client to continue speaking or clarifies what has been said. - Exploring permits the nurse to delve deeper into the subject when the client tends to stay on a superficial level. - Accepting is a technique used to understand and demonstrate regard for what the client stated.

A physician orders a urinalysis for a client with an indwelling catheter. To ensure that an appropriate specimen is obtained, the nurse would obtain the specimen from which site? 1Tubing injection port 2 Distal end of the tubing 3 Urinary drainage bag 4 Catheter insertion site

1 The appropriate site to obtain a urine specimen for a patient with an indwelling catheter is the injection port. The nurse should clean the injection port cap of the catheter drainage tubing with appropriate antiseptic, attach a sterile 5-mL syringe into the port, and aspirate the quantity desired. The nurse should apply a clamp to the drainage tubing, distal to the injection port, not obtain the specimen from this site. Urine in the bedside drainage bag is not an appropriate sample as the urine in the bag may have been there too long; thus a clean sample cannot be obtained from the bag. The client's urine will be contained in the indwelling catheter; there will be no urine at the insertion site.

A nurse is teaching members of a health care team how to help disabled clients stand and transfer from the bed to a chair. To protect the caregivers from injury, the nurse teaches them to lift the client by first placing their arms under the client's axillae and next: 1 Bending and then straightening their knees 2 Bending at the waist and then straightening the back 3 Placing one foot in front of the other and then leaning back 4 Placing pressure against the client's axillae and then raising their arms

1 The leg bones and muscles are used for weight bearing and are the strongest in the body. Using the knees for leverage while lifting the client shifts the stress of the transfer to the caregiver's legs. By using the strong muscles of the legs the back is protected from injury. Bending at the waist and then using the back for leverage is how many caregivers and people who must lift heavy objects sustain back injuries. The anatomical structure of the back is equipped only to bear the weight of the upper body. By leaning back, the client's weight is on the caregiver's arms, which are not equipped for heavy weight bearing. The caregiver's arms are not strong enough to lift the client. In the struggle to lift the client, the client and caregiver may be injured.

Filgrastim (Neupogen) 5 mcg/kg/day by injection is prescribed for a client who weighs 132 lb. The vial label reads filgrastim 300 mcg/mL. How many milliliters should the nurse administer? Record the answer using a whole number. ______ mL

1 mL

The nurse who is working during the 8:00 am to 4:00 pm shift must document a client's fluid intake and output. An intravenous drip is infusing at 50 mL per hour. The client drinks 4 oz of orange juice and 6 oz of tea at 8:30 am and vomits 200 mL at 9:00 am. At 10:00 am the client drinks 60 mL of water with medications; the client voids 550 mL of urine at 11:00 am. At 12:30 pm, 3 oz of soup and 4 oz of ice cream are ingested. The client voids 450 mL at 2:00 pm. Calculate the total intake for the 8:00 am to 4:00 pm shift. Record your answer using a whole number. ___mL

1 ounce = 30 mL; therefore the client ingested 120 mL of orange juice at 8:30 am, 180 mL of tea at 8:30 am, 60 mL of water with medications at 10:00 am, 90 mL of soup at 12:30 am, and 120 mL of ice cream at 12:30 pm (counted as a liquid because it melts at room temperature). The client received 400 mL of IV fluid (50 mL × 8 hours = 400). Total intake is 970 mL. Vomit and urine output should not be included in the patient's intake.

While assessing a client's vascular system, the nurse finds that pulse strength is diminished or barely palpable. Which documentation is appropriate in this situation? Multiple choice question 1+ 2+ 3+ 4+

1+ A diminished or barely palpable pulse is documented as 1+. A normal and expected pulse strength is documented as 2+. A full, strong pulse is documented as 3+. A bounding pulse is documented as 4+.

Which nursing assessment questions assess the faith, belief, fellowship, and community aspect of a client's spirituality? Select all that apply. 1 "What gives meaning to your life?" 2 "What is your source of power, hope, and belief during difficult times?" 3 "In what way do your beliefs help or strengthen you for coping with illness?" 4 "How has the illness affected your capability to express what is essential in life?" 5 "How do you feel the changes caused by the illness are affecting or will affect your life?"

1, 2, 3 "What gives meaning to your life?" "What is your source of power, hope, and belief during difficult times?" "In what way do your beliefs help or strengthen you for coping with illness?"

A nurse arrives for work to find that the medical unit is short-staffed. Nursing administration has called several staffing agencies, but they are unable to send a replacement nurse for three hours. The nursing care coordinator sends a recently oriented patient care assistant to help relieve the burden of care. Which activities should the nurse delegate to the patient care assistant? Select all that apply. 1Making occupied beds 2 Taking routine vital signs 3 Answering clients' call lights 4 Watching a client take oral medications 5 Emptying a closed chest drainage system for intake and output

1, 2, 3 Patient care assistants can make occupied and unoccupied beds. Taking routine vital signs is within the scope of practice of patient care assistants. Answering call lights and meeting clients' basic safety, hygiene, and comfort needs are within the scope of practice of patient care assistants. Watching a client take oral medications is part of procedure for administration of medications, which requires a professional license. Emptying a closed chest drainage system for intake and output is inappropriate; a closed chest drainage system is not emptied for intake and output. Documentation is indicated on the outside of the drainage collection chamber.

Which warning signals should the nurse observe in a child suspected to be a victim of abuse? Select all that apply. 1 The child doesn't want to be touched by anyone. 2 The child sleeps for an average of 15 hours a day. 3 The child frequently visits the emergency department. 4 The child suffers from fever and tenderness in the abdomen. 5 The child looks at the caregiver before answering any question.

1, 3, 5 The child may become scared if touched. The physical abuse may cause injuries and the child may visit the emergency department frequently. An abused child may look at the caregiver before answering any question due to fear.

A patient has hypokalemia with stable cardiac function. What are the priority nursing interventions? (Select all that apply.) 1. Fall prevention interventions 2. Teaching regarding sodium restriction 3. Encouraging increased fluid intake 4. Monitoring for constipation 5. Explaining how to take daily weights

1. Fall prevention interventions 4. Monitoring for constipation

A client is admitted to the hospital and benazepril hydrochloride (Lotensin) is prescribed for hypertension. Which is an appropriate nursing action for clients taking this medication? 1 Monitor the EEG. 2 Assess for dizziness. 3 Administer the drug after meals. 4 Assess for dark, tarry stools.

2 Assess for dizziness. Dizziness may occur during the first few weeks of therapy until the client adapts physiologically to the medication. An electroencephalogram is unnecessary. Cardiac monitoring may be instituted because of possible dysrhythmias. Administering the drug after meals is unnecessary; however, if nausea occurs, the medication may be taken with food or at bedtime. The blood pressure should be monitored before and after administration. Dark, tarry stools are not a side effect of Lotensin.

A nurse provides teaching for a client who is scheduled for a cholecystectomy. In the initial postoperative period, the nurse explains that the most important part of the treatment plan is: 1 Early ambulation 2 Coughing and deep breathing 3 Wearing anti-embolic elastic stockings 4 Maintenance of a nasogastric tube

2 Coughing and deep breathing

A visitor in the waiting room of the emergency department has a syncopal episode and collapses on the floor. The event is witnessed by a nurse, who provides initial care. The nurse assessed the client, maintained safety of the environment, and gave a report to the emergency department nurse, who will provide ongoing care. What should the nurse who witnessed the event do next? 1 Contact the family 2 Document the incident 3 Report the incident to the nurse manager 4 Escort the client to the radiology department

2 Document the incident

A client's intravenous (IV) infusion infiltrates. The nurse concludes that what is most likely the cause of the infiltration? 1 Excessive height of the IV bag 2 Failure to secure the catheter adequately 3 Contamination during the catheter insertion 4 Infusion of a chemically irritating medication

2 Failure to secure the catheter adequately Infiltration is caused by catheter displacement, allowing fluid to leak into the tissues. Excessive height of the IV bag will affect the flow rate, not cause infiltration. Contamination during the catheter insertion can lead to infection and phlebitis, not infiltration. Infusion of a chemically irritating medication can lead to phlebitis, not infiltration.

A client has been admitted with a diagnosis of intractable vomiting and can only tolerate sips of water. The initial blood work shows a sodium level of 122 mEq/L and a potassium level of 3.6 mEq/L. Based on the lab results and symptoms, what is the client experiencing? 1 Hypernatremia 2 Hyponatremia 3 Hyperkalemia 4 Hypokalemia

2 Hyponatremia The normal range for serum sodium is 135 to 145 mEq/L, and for serum potassium it is 3.5 to 5 mEq/L. Vomiting and use of diuretics, such as furosemide (Lasix), deplete the body of sodium. Without intervention, symptoms of hyponatremia may progress to include neurological symptoms such as confusion, lethargy, seizures, and coma. Hypernatremia results when serum sodium is greater than 145 mEq/L; hyperkalemia results when serum potassium is greater than 5.0 mEq/L; hypokalemia results when serum potassium is less than 3.5 mEq/L.

Based on the client's reported pain level, the nurse administers 8 mg of the prescribed morphine. The medication is available in a 10 mg syringe. Wasting of the remaining 2 mg of morphine should be done by the nurse and a witness. It is most appropriate for the nurse to ask which member of the health care team to be the witness? 1 Nursing supervisor 2 Licensed practical nurse (LPN) 3 Client's health care provider 4 Designated nursing assistant

2 Licensed practical nurse (LPN)

A client reports vomiting and diarrhea for three days. What clinical finding most accurately will indicate that the client has a fluid deficit? 1 Presence of dry skin 2 Loss of body weight 3 Decrease in blood pressure 4 Altered general appearance

2 Loss of body weight Dehydration is measured most readily and accurately by serial assessments of body weight; 1 L of fluid weighs 2.2 lb.

A nurse is preparing to administer an oil-retention enema and understands that it works primarily by: 1 Stimulating the urge to defecate. 2 Lubricating the sigmoid colon and rectum. 3 Dissolving the feces. 4 Softening the feces.

2 Lubricating the sigmoid colon and rectum. The primary purpose of an oil-retention enema is to lubricate the sigmoid colon and rectum. Secondary benefits of an oil-retention enema include stimulating the urge to defecate and softening feces. An oil-retention enema does not dissolve feces .

A health care provider prescribes transdermal fentanyl (Duragesic) 25 mcg/hr every 72 hours. During the first 24 hours after starting the fentanyl, what is the most important nursing intervention? 1 Change the dose until pain is tolerable. 2 Manage pain with oral pain medication. 3 Assess the client for anticholinergic side effects. 4 Instruct the client to take the medication with food.

2 Manage pain with oral pain medication.

A pain scale of 1 to 10 is used by a nurse to assess a client's degree of pain. The client rates the pain as an 8 before receiving an analgesic and a 7 after being medicated. What conclusion should the nurse make regarding the client's response to pain medication? 1 Client has a low pain tolerance. 2 Medication is not adequately effective. 3 Medication has sufficiently decreased the pain level. 4 Client needs more education about the use of the pain scale.

2 Medication is not adequately effective. The expected effect should be more than a 1-point decrease in the pain level.

When monitoring fluids and electrolytes, the nurse recalls that the major cation-regulating intracellular osmolarity is: 1 Sodium 2 Potassium 3 Calcium 4 Calcitonin

2 Potassium A decrease in serum potassium causes a decrease in the cell wall pressure gradient and results in water moving out of the cell. Besides intracellular osmolarity regulation, potassium also regulates metabolic activities, transmission and conduction of nerve impulses, cardiac conduction, and smooth and skeletal muscle contraction.

A client reports smoke coming from a utility room on the nursing unit. What is the initial action the nurse should take? 1 Pull the fire alarm on the unit. 2 Remove anyone that is in immediate danger. 3 Obtain a fire extinguisher and report to the fire area. 4 Close all windows and fire doors and await further instructions.

2 Remove anyone that is in immediate danger.

A client who sustained a large open wound as a result of an accident is receiving daily sterile dressing changes. To maintain sterility when changing the dressing, the nurse should: 1 Put the unopened sterile glove package carefully on the sterile field 2 Remove the sterile drape from its package by lifting it by the corners 3 Don sterile gloves before opening the package containing the field drape 4 Pour irrigation liquid from a height of at least three inches above the sterile container

2 Remove the sterile drape from its package by lifting it by the corners The outer one inch of the sterile field is considered contaminated and can be touched without wearing sterile gloves. The outside of an unopened sterile glove package is not sterile. The field will become contaminated if the unopened package is placed on the sterile field. The outer package, which contains a sterile field drape, is not sterile; if it is touched with sterile gloves, the sterile gloves will become contaminated. Liquids should be poured from a height of 4 to 6 inches; this ensures that the solution bottle does not contaminate the sterile container.

What are the clinical indicators that a nurse expects when an intravenous (IV) line has infiltrated? (Select all that apply.) 1 Heat 2 Pallor 3 Edema 4 Decreased flow rate 5 Increased blood pressure

2 Pallor 3 Edema 4 Decreased flow rate The accumulation of fluid in the tissues between the surface of the skin and the blood vessels makes the skin appear pale. The accumulation of fluid in the interstitial compartment causes swelling. As the needle/catheter is dislodged from the vein, the drip rate of the IV slows or ceases. Heat is associated with phlebitis; the accumulation of room temperature IV fluid in the tissue makes the site feel cool. Increased blood pressure is a sign of circulatory overload; when an IV infusion has infiltrated, the intravascular fluid volume does not increase.

A nurse provides discharge teaching related to intermittent urinary self-catheterization to a client with a new spinal cord injury. Which instruction is most important for the nurse to include? 1 "Wear sterile gloves when doing the procedure." 2 "Wash your hands before performing the procedure." 3 "Perform the self-catheterization every 12 hours." 4 "Dispose of the catheter after you have catheterized yourself."

2 "Wash your hands before performing the procedure." To prevent transferring organisms to the urinary system, the client is taught to wash his or her hands thoroughly with soap and water before inserting a clean catheter. - Sterile gloves are not required for this procedure in the home care setting. - Every 12 hours is too long of a time frame between catheterizations. The client should be taught to recognize when self-catheterization is needed and develop a 2- to 3-hour catheterization schedule. - Some home care settings may require the client to clean and re-use catheters.

What is a characteristic of the primary nursing model? 1 Care can be delegated. 2 Care is provided by the registered nurse to the client during a stay in a facility. 3 The registered nurse is responsible for all aspects of care for one or more clients during a shift of care. 4 The registered nurse leads a team of other registered nurses, practical nurses, and unlicensed assistive personnel.

2 Care is provided by the registered nurse to the client during a stay in a facility.

A client is admitted to the hospital for an elective surgical procedure. The client tells a nurse about the emotional stress of recently disclosing being a homosexual to family and friends. What is the nurse's first consideration when planning care? 1 Exploring the client's emotional conflict 2 Identifying personal feelings toward this client 3 Planning to discuss this with the client's family 4 Developing a rapport with the client's health care provider

2 Nurses must identify their own feelings and prejudices because these may affect the ability to provide objective, nonjudgmental nursing care. Exploring a client's emotional well-being can be accomplished only after the nurse works through one's own feelings. The focus should be on the client, not the family. Health team members should work together for the benefit of all clients, not just this client.

The nurse assessing an adult understands that the client is experiencing a midlife crisis. Which factor should the nurse attribute to this condition? 1 The client is seeking an occupational direction. 2 The client is examining life goals and relationships. 3 The client is directing energy towards achievements. 4 The client is sharing responsibilities in a two-career family.

2 The client is examining life goals and relationships. Individuals between the age of 35 and 43 are vigorously examining their life goals and relationships. These individuals often experience stress or a midlife crisis during this reexamination, which may lead to changes in personal, social, and occupational areas.

When suctioning a client with a tracheostomy, an important safety measure for the nurse is to: 1 Hyperventilate the client with room air prior to suctioning. 2 Apply suction only as the catheter is being withdrawn. 3 Insert the catheter until the cough reflex is stimulated. 4 Remove the inner cannula before inserting the suction catheter.

2 Use of suction upon withdrawal of a suction catheter reduces unnecessary removal of oxygen. In addition, suction should be applied intermittently during the withdrawal procedure to prevent hypoxia. A sterile catheter is used to prevent infection, and the catheter should only be inserted approximately 1 to 2 cm past the end of the tracheostomy tube to prevent tissue trauma. Hyperventilating a client before suctioning should always be with oxygen, not room air. Inserting the catheter until the cough reflex is stimulated frequently occurs and does help to mobilize secretions but is not a safety measure. Removal of the inner cannula before inserting the suction catheter is not necessary.

Normal Mean Corpuscular Hemoglobin (MCH)

27 - 34 PG (pictograms, one trillionth of a gram) The average amount of hemoglobin in the average red cell.

A client is being treated for Influenza A (H1N1). The nurse has provided instructions to the client about how to decrease the risk of transmission to others. Which patient statement indicates a need for further instruction/clarification? 1 "I should practice respiratory hygiene/cough etiquette." 2 "I should avoid contact with the elderly or children." 3 "I should obtain a pneumococcal vaccination each year." 4 "I should allow visitors for short periods of time only."

3 "I should obtain a pneumococcal vaccination each year."

Neomycin, 1 gram, is prescribed preoperatively for a client with cancer of the colon. The client asks why this is necessary. How should the nurse respond? 1 "It is used to prevent you from getting a bladder infection before surgery." 2 "It will decrease your kidney function and lessen urine production during surgery." 3 "It will kill the bacteria in your bowel and decrease the risk for infection after surgery." 4 "It is used to alter the body flora, which reduces spread of the tumor to adjacent organs."

3 "It will kill the bacteria in your bowel and decrease the risk for infection after surgery." Neomycin provides preoperative intestinal antisepsis. It is not administered to prevent bladder infection. Nephrotoxicity is an adverse, not a therapeutic, effect. Neomycin will not prevent metastasis of the tumor to other areas.

A client's chest tube has accidentally dislodged. What is the nursing action of highest priority? 1 Place the client in a left side-lying position. 2 Apply oxygen via non-rebreather mask. 3 Apply a petroleum gauze dressing over the site. 4 Prepare to reinsert a new chest tube.

3 Apply a petroleum gauze dressing over the site.

The nurse instructs a client that, in addition to building bones and teeth, calcium is also important for: 1 Bile production. 2 Blood production. 3 Blood clotting. 4 Digestion of fats

3 Blood clotting. Calcium is important for blood coagulation. When tissue damage occurs, serum calcium is necessary to promote coagulation by activating certain clotting factors. Calcium acts as a catalyst in the clotting process in both the extrinsic and intrinsic pathways. Calcium is responsible for a number of body functions such as bone health, blood clotting, and muscle contraction and nerve impulses

A nurse reviews a medical record of a client with ascites. What does the nurse identify that may be causing the ascites? 1 Portal hypotension 2 Kidney malfunction 3 Decreased liver function 4 Decreased production of potassium

3 Decreased liver function The liver manufactures albumin, the major plasma protein. A deficit of this protein lowers the osmotic (oncotic) pressure in the intravascular space, leading to a fluid shift. An enlarged liver compresses the portal system, causing increased, rather than decreased, pressure.

The nurse reviews a medical record and is concerned that the client may develop hyperkalemia. Which disease increases the risk of hyperkalemia? 1 Crohn's 2 Cushing's 3 End-stage renal 4 Gastroesophageal reflux

3 End-stage renal One of the kidneys' functions is to eliminate potassium from the body; diseases of the kidneys often interfere with this function, and hyperkalemia may develop, necessitating dialysis.

A primary nurse receives prescriptions for a newly admitted client and has difficulty reading the health care provider's writing. Who should the nurse ask for clarification of this prescription? 1 Nurse practitioner 2 House health care provider that is on-call 3 Health care provider who wrote the prescription 4 Nurse manager familiar with the health care provider's writing

3 Health care provider who wrote the prescription

A client has been admitted with a urinary tract infection. The nurse receives a urine culture and sensitivity report that reveals the client has Vancomycin Resistant Enterococcus (VRE). After notifying the physician, which action should the nurse take to decrease the risk of transmission to others? 1 Insert a urinary catheter. 2 Initiate Droplet Precautions. 3 Move the client to a private room. 4 Use a high efficiency particulate air (HEPA) respirator during care.

3 Move the client to a private room.

A client with a diagnosis of uncontrolled diabetes began receiving Lasix (Furosemide) two days ago. The nurse reviews the morning lab results and discovers that the client's potassium level is 2.8 mEq/L. What is the most appropriate action for the nurse to take? 1 Hold the morning dose of the diuretic and have the lab repeat the test. 2 Continue to monitor the level to ensure that it stays within the normal limits. 3 Notify the primary healthcare provider of the result, which is critically low. 4 Anticipate a prescription for an increase in the dosage of the Lasix.

3 Notify the primary healthcare provider of the result, which is critically low. The physician should be notified because a potassium level of 2.8 mEq/L is low. Normal range for serum potassium is 3.5 to 5 mEq/L. Clients who are on diuretics require monitoring of serum electrolytes, especially potassium and sodium, because they also are excreted with water. The nurse should not hold the diuretic or repeat the lab test unless advised by the physician. The client's serum potassium level is critically below the normal limit and the physician should be notified. An increase in Lasix would cause an increased loss of potassium

A client is admitted with metabolic acidosis. The nurse considers that two body systems interact with the bicarbonate buffer system to preserve healthy body fluid pH. What two body systems should the nurse assess for compensatory changes? 1 Skeletal and nervous 2 Circulatory and urinary 3 Respiratory and urinary 4 Muscular and endocrine

3 Respiratory and urinary Increased respirations blow off carbon dioxide (CO2 ), which decreases the hydrogen ion concentration and the pH increases (less acidity). Decreased respirations result in CO2 buildup, which increases hydrogen ion concentration and the pH falls (more acidity). The kidneys either conserve or excrete bicarbonate and hydrogen ions, which helps to adjust the body's pH . The buffering capacity of the renal system is greater than that of the pulmonary system, but the pulmonary system is quicker to respond.

What factors are most important for the nurse to consider when delegating responsibilities? 1 Preferences of the clients and staff 2 Physical layout of the unit and client rooms 3 Staff member's level of education and expertise 4 Client's diagnosis and length of time in the hospital

3 Staff member's level of education and expertise

A nurse is providing care to a client eight hours after the client had surgery to correct an upper urinary tract obstruction. Which assessment finding should the nurse report to the charge nurse or surgeon? 1 Incisional pain 2 Absent bowel sounds 3 Urine output of 20 mL/hour 4 Serosanguineous drainage on the dressing

3 Urine output of 20 mL/hour A urinary output of 50 mL/hr or greater is necessary to prevent stasis and consequent infections after this type of surgery. The nurse should notify the surgeon of the assessment findings, as this may indicate a urinary tract obstruction.

A client with Addison's disease is receiving cortisone therapy. The nurse expects what clinical indicators if the client abruptly stops the medication? (Select all that apply.) 1 Diplopia 2 Dysphagia 3 Tachypnea 4 Bradycardia 5 Hypotension

3 Tachypnea 5 Hypotension Tachypnea occurs with Addisonian crisis because of inadequate circulating glucocorticoids and mineralocorticoids. Inadequate circulating glucocorticoids and mineralocorticoids cause hypotension, pallor, weakness, tachycardia, and tachypnea. Double vision does not occur with Addisonian crisis. Difficulty swallowing does not occur with Addisonian crisis. Tachycardia, not bradycardia, occurs with Addisonian crisis.

A registered nurse is teaching a nursing student about caring for a client before leaving the healthcare facility. Which statement made by the nursing student indicates the need for further education? 1 "I should teach the client about potential food-drug interactions." 2 "I should involve the client and his or her family in the referral process." 3 "I should give limited information about the client to the healthcare provider who received the referral." 4 "I should teach the client and his or her family about safe and effective use of medications and medical equipment."

3 "I should give limited information about the client to the healthcare provider who received the referral." The nurse should provide as much client information as possible to the healthcare provider who received the referral because this action helps to avoid the provider asking duplicate questions and helps to avoid the omission of important information.

Nursing actions for an older adult should include health education and promotion of self-care. Which is most important when working with an older adult client? 1 Encouraging frequent naps 2 Strengthening the concept of ageism 3 Reinforcing the client's strengths and promoting reminiscing 4 Teaching the client to increase calories and focusing on a high-carbohydrate diet

3 Reinforcing the client's strengths and promoting reminiscing Reinforcing strengths promotes self-esteem; reminiscing is a therapeutic tool that provides a life review that assists adaptation and helps achieve the task of integrity associated with older adulthood. - Frequent naps may interfere with adequate sleep at night. - Reinforcing ageism may enhance devaluation of the older adult. - A well-balanced diet that includes protein and fiber should be encouraged; increasing calories may cause obesity.

Which developmental changes should be evaluated in girls around 12 years of age? 1 Motor skills 2 Visual acuity 3 Skeletal growth 4 Hormonal changes

3 Skeletal Growth Girls around the age of 12 years of age may develop scoliosis (a lateral curvature of the spine); therefore, skeletal growth should be evaluated.

A weak, dyspneic, terminally ill client is visited frequently by the spouse and teenage children. What should the client's plan of care include? 1 Foster self-activity whenever possible. 2 Plan care to be completed at one time followed by a long rest. 3 Teach family members how to assist with the client's basic care. 4 Limit visiting to evening hours before the client goes to sleep.

3 Teach family members how to assist with the client's basic care.

A client with coronary artery disease has a sudden episode of cyanosis and a change in respirations. The nurse starts oxygen administration immediately. Legally, should the nurse have administered the oxygen? 1 The oxygen had not been prescribed and therefore should not have been administered. 2 The symptoms were too vague for the nurse to determine a need for administering oxygen. 3 The nurse's observations were sufficient, and therefore oxygen should have been administered. 4 The health care provider should have been called for a prescription before the nurse administered the oxygen.

3 The Nurse Practice Act states that nurses diagnose and treat human responses to actual or potential health problems. Administration of oxygen in an emergency situation is within the scope of nursing practice. Because the client's clinical manifestations reflected an immediate need for oxygen, postponement of treatment could have resulted in further deterioration of the client's condition.

A client has a "prayer cloth" pinned to the hospital gown. The cloth is soiled from being touched frequently. What should the nurse do when changing the client's gown? 1 Make a new prayer cloth. 2 Discard the soiled prayer cloth. 3 Pin the prayer cloth to the clean gown. 4 Wash the prayer cloth with a detergent.

3 The prayer cloth has religious significance for the client and should be preserved as is. Making a new prayer cloth disregards what the prayer cloth means to the client. The prayer cloth is the property of the client and should not be discarded. Washing the prayer cloth with a detergent disregards what the prayer cloth means to the client; this never should be done without the client's permission.

A high-protein diet is recommended for a client recovering from a fracture. The nurse recalls that the rationale for a high-protein diet is to: 1 Promote gluconeogenesis. 2 Produce an anti-inflammatory effect. 3 Promote cell growth and bone union. 4 Decrease pain medication requirements.

3 There is an increased need for protein with any type of body tissue trauma. High protein intake in the client with a fractured bone promotes cell growth and therefore bone union. Intake of a high protein diet during recovery from a bone fracture is not related to gluconeogenesis, inflammation, or pain.

A physician orders guaifenesin (Humibid) 300 mg four times a day. The dosage strength is 200 milligrams/5 milliliters. To ensure the patient's safety, how many milliliters should the nurse administer for each dose? Record your answer using one decimal place. ____ mL

300mg/x = 200mg/5mL X = 7.5 mL

A client with a history of ulcerative colitis is admitted to the hospital because of severe rectal bleeding. The client engages in angry outbursts and places excessive demands on the staff. One day an unlicensed assistive personnel (UAP) tells the nurse, "I've had it. I am not putting up with that behavior. I'm not going in there again." What is the best response by the nurse? 1 "You need to try to be patient. The client is going through a lot right now." 2 "I'll talk with the client. Maybe I can figure out the best way for us to handle this." 3 "Just ignore it and get on with your work. I'll assign someone else to take a turn." 4 "The client's frightened and taking it out on the staff. Let's think of approaches we can take."

4 "The client's frightened and taking it out on the staff. Let's think of approaches we can take."

What is the maximum length of time a nurse should allow an intravenous (IV) bag of solution to infuse? 1 6 hours 2 12 hours 3 18 hours 4 24 hours

4 24 hours After 24 hours there is increased risk for contamination of the solution and the bag should be changed. It is unnecessary to change the bag any less often.

A client has been diagnosed with type 1 Diabetes Mellitus. When providing instructions on sharps disposal, the nurse should instruct the client to place the syringes in: 1 Bubble wrap/packaging wrap 2 A garbage bag in the trash can 3 A cardboard box with a firmly secured lid 4 A plastic liquid detergent bottle with a screw-top lid

4 A plastic liquid detergent bottle with a screw-top lid

What clinical finding indicates to the nurse that a client may have hypokalemia? 1 Edema 2 Muscle spasms 3 Kussmaul breathing 4 Abdominal distention

4 Abdominal distention Hypokalemia diminishes the magnitude of the neuronal and muscle cell resting potentials. Abdominal distention results from flaccidity of intestinal and abdominal musculature. Edema is a sign of sodium excess. Muscle spasms are a sign of hypocalcemia. Kussmaul breathing is a sign of metabolic acidosis.

What response should a nurse be particularly alert for when assessing a client for side effects of long-term cortisone therapy? 1 Hypoglycemia 2 Severe anorexia 3 Anaphylactic shock 4 Behavioral changes

4 Behavioral changes Development of mood swings and psychosis is possible during long-term therapy with glucocorticoids because of fluid and electrolyte alterations.

An adolescent is taken to the emergency department of the local hospital after stepping on a nail. The puncture wound is cleansed and a sterile dressing applied. The nurse asks about having had a tetanus immunization. The adolescent responds that all immunizations are up to date. Penicillin is administered, and the client is sent home with instructions to return if there is any change in the wound area. A few days later, the client is admitted to the hospital with a diagnosis of tetanus. Legally, what is the nurse's responsibility in this situation? 1 The nurse's judgment was adequate, and the client was treated accordingly. 2 The possibility of tetanus was not foreseen because the client was immunized. 3 Nurses should routinely administer immunization against tetanus after such an injury. 4 Data collection by the nurse was incomplete, and as a result the treatment was insufficient.

4 Data collection by the nurse was incomplete, and as a result the treatment was insufficient.

After abdominal surgery a client reports pain. What action should the nurse take first? 1 Reposition the client. 2 Obtain the client's vital signs. 3 Administer the prescribed analgesic. 4 Determine the characteristics of the pain.

4 Determine the characteristics of the pain. The exact nature of the pain must be determined to distinguish whether or not it is a result of the surgery. Repositioning the client, obtaining the client's vital signs, and administering the prescribed analgesic should be done later; the first action is to determine the cause of the pain.

The nurse is caring for a client who is receiving therapy for vitamin B12 deficiency. Which finding indicates that the therapy is having the desired effect? 1 Normal serum electrolyte levels 2 Healthy skin integrity 3 Resolution of peripheral edema 4 Improved hemoglobin and hematocrit levels

4 Improved hemoglobin and hematocrit levels Vitamin B12 is essential for appropriate maturation of red blood cells; therefore relieving the deficiency is expected to improve hemoglobin and hematocrit (H&H) levels and decrease hypoxia-related problems. This disorder is known as pernicious anemia.

A health care provider prescribes famotidine (Pepcid) and magnesium hydroxide/aluminum hydroxide (Maalox) for a client with a peptic ulcer. The nurse should teach the client to take the Maalox at what time? 1 Only at bedtime, when famotidine is not taken. 2 Only if famotidine is ineffective. 3 At the same time as famotidine, with a full glass or water. 4 One hour before or two hours after famotidine

4 One hour before or two hours after famotidine Antacids interfere with complete absorption of famotidine; therefore, antacids should be administered at least one hour before or two hours after famotidine. Magnesium hydroxide/aluminum hydroxide usually is taken one hour after meals and at bedtime. Famotidine usually is prescribed once a day at bedtime. The client has received a prescription for both medications; the client should not be instructed to omit one of the medications without checking with the health care provider first.

A nurse is reviewing a plan of care for a client who was admitted with dehydration as a result of prolonged watery diarrhea. Which prescription should the nurse question? 1 Oral psyllium (Metamucil) 2 Oral potassium supplement 3 Parenteral half normal 4 Parenteral albumin (Albuminar)

4 Parenteral albumin (Albuminar) Albumin is hypertonic and will draw additional fluid from the tissues into the intravascular space. Oral psyllium will absorb the watery diarrhea, giving more bulk to the stool. An oral potassium supplement is appropriate because diarrhea causes potassium loss. Parenteral half normal saline is a hypotonic solution, which can correct dehydration.

A nurse is reviewing a plan of care for a client who was admitted with dehydration as a result of prolonged watery diarrhea. Which prescription should the nurse question? 1 Oral psyllium (Metamucil) 2 Oral potassium supplement 3 Parenteral half normal saline 4 Parenteral albumin (Albuminar)

4 Parenteral albumin (Albuminar) Albumin is hypertonic and will draw additional fluid from the tissues into the intravascular space. Oral psyllium will absorb the watery diarrhea, giving more bulk to the stool. An oral potassium supplement is appropriate because diarrhea causes potassium loss. Parenteral half normal saline is a hypotonic solution, which can correct dehydration.

A nurse manager works on a unit where the nursing staff members are uncomfortable taking care of clients from cultures that are different from their own. How should the nurse manager address this situation? 1 Assign articles about various cultures so that they can become more knowledgeable. 2 Relocate the nurses to units where they will not have to care for clients from a variety of cultures. 3 Rotate the nurses' assignments so they have an equal opportunity to care for clients from other cultures. 4 Plan a workshop that offers opportunities to learn about the cultures they might encounter while at work.

4 Plan a workshop that offers opportunities to learn about the cultures they might encounter while at work.

A nurse assesses a client's serum electrolyte levels in the laboratory report. What electrolyte in intracellular fluid should the nurse consider most important? 1 Sodium 2 Calcium 3 Chloride 4 Potassium

4 Potassium

When providing preoperative teaching, the nurse should focus primarily on: 1 Helping the client and family decide if surgery is necessary. 2 Providing emotional support to the client and family. 3 Giving minute-by-minute details of the surgery to the client and family. 4 Providing general information to reduce client and family anxiety.

4 Providing general information to reduce client and family anxiety.

A client undergoes a bowel resection. When assessing the client 4 hours postoperatively, the nurse identifies which finding as an early sign of shock? 1 Respirations of 10 2 Urine output of 30 ml/hour 3 Lethargy 4 Restlessness

4 Restlessness n the early stage shock, the client has increased epinephrine secretion. This, in turn, causes the client to become restless, anxious, nervous, and irritable. Decreased respiratory rate is a late sign of shock. A urine output of 30 ml/hour is within normal limits.

A client with a terminal illness reaches the stage of acceptance. How can the nurse best help the client during this stage? 1 Acknowledge the client's crying. 2 Encourage unrestricted family visits. 3 Explain details of the care being given. 4 Stay nearby without initiating conversation

4 Stay nearby without initiating conversation The nurse's presence communicates concern and provides an opportunity for the client to initiate communication; silence is an effective interpersonal technique that permits the client to direct the content and extent of verbalizations without the nurse imposing on the client's privacy.

A nurse reinforces teaching a client about Coumadin (warfarin) and concludes that the teaching is effective when the client states, "I must not drink: 1 apple juice. 2 grape juice. 3 orange juice. 4 cranberry juice.

4 cranberry juice. Antioxidants in cranberry juice may inhibit the mechanism that metabolizes Coumadin, causing elevations in the international normalized ratio (INR), resulting in hemorrhage. Apple juice, grape juice, and orange juice are fine to drink.

The nurse is assessing a Latino-Caribbean client who was brought to the hospital by family members. The family reports the client started crying, shouting, trembling, had uncontrolled jerking of the extremities, and then fell into a trance-like state. What condition does the nurse suspect? 1 Bulimia nervosa 2 Anorexia nervosa 3 Shenjing shuairuo 4 Ataque de nervios

4 Ataque de nervios Ataque de nervios is a Latino-Caribbean culture-bound syndrome that usually happens in response to specific stressors. This culture-bound syndrome is characterized by crying, uncontrollable spasms, trembling, shouting, dissociation, and trance-like states.

A nurse is providing immediate postoperative care to a client who had a lung resection for a malignancy. The client has a closed chest tube drainage system connected to suction. Which assessment finding requires additional evaluation by the nurse? 1 A column of water 20 cm high in the suction control chamber 2 75 mL of bright red blood in the drainage collection chamber 3 An intact occlusive dressing at the insertion site 4 Constant bubbling in the water seal chamber

4 Constant bubbling in the water seal chamber Constant bubbling in the water seal chamber is indicative of an air leak. The nurse should assess the entire length of the system from the container to the client's chest wall tube insertion site to find the source of the air leak. If the source of the air leak is not found in the system and bubbling continues, the leak is most likely within the client's chest or at the insertion site. This could cause the lung to collapse because of a buildup of air pressure within the plural cavity, and therefore the healthcare provider should be notified. In this type of surgical procedure, 75 mL of blood in the chest tube collection chamber is an expected finding in the early postoperative period. A column of water 20 cm high in the suction control chamber and an intact occlusive dressing at the chest tube insertion site are also expected assessment findings.

The nurse is caring for a client with a temperature of 104.5 degrees Fahrenheit. The nurse applies a cooling blanket and administers an antipyretic medication. The nurse explains that the rationale for these interventions is to: 1 Promote equalization of osmotic pressures. 2 Prevent hypoxia associated with diaphoresis. 3 Promote integrity of intracerebral neurons. 4 Reduce brain metabolism and limit hypoxia.

4 Cooling blankets and antipyretic medications can induce hypothermia thus decreasing brain metabolism. This in turn makes the brain less vulnerable by decreasing the need for oxygen. The integrity of intracerebral neurons and osmotic pressure equalization depend on an adequate supply of oxygen, carbon dioxide, and glucose, and may occur as a result of decreased cerebral metabolism and hypoxia. Diaphoresis does not cause hypoxia. Antipyretic medications may cause diaphoresis as vasodilation occurs.

While assessing an older adult, the nurse observes visual impairment in the client. Which technique should the nurse use to communicate? 1 Face the caregiver while speaking 2 Provide bright, diffuse, glare lighting 3 Stand or sit far away from the client while remaining in the client's full view 4 Encourage the older adult to use assistive devices such as glasses

4 Encourage the older adult to use assistive devices such as glasses If an older adult has visual impairment, the nurse should encourage the older adult to use assistive devices such as glasses. The nurse should face the older adult while speaking and should not cover his or her mouth. The light should be bright and non-glaring so that the older adult can see properly. The nurse should stand or sit closely in front of the client in full view so that the client is able to identify.

The spouse of a comatose client who has severe internal bleeding refuses to allow transfusions of whole blood because they are Jehovah's Witnesses. The client does not have a Durable Power of Attorney for Healthcare. What action should the nurse take? 1 Institute the prescribed blood transfusion because the client's survival depends on volume replacement. 2 Clarify the reason why the transfusion is necessary and explain the implications if there is no transfusion. 3 Phone the primary healthcare provider for an administrative prescription to give the transfusion under these circumstances. 4 Give the spouse a treatment refusal form to sign and notify the primary healthcare provider that a court order now can be sought.

4 Give the spouse a treatment refusal form to sign and notify the primary healthcare provider that a court order now can be sought. The client is unconscious. Although the spouse can give consent, there is no legal power to refuse a treatment for the client unless previously authorized to do so by a power of attorney or a healthcare proxy; the court can make a decision for the client. Explanations will not be effective at this time and will not meet the client's needs. Instituting the prescribed blood transfusion and phoning the primary healthcare provider for an administrative prescription are without legal basis, and the nurse may be held liable.

While assessing an immobilized client, the nurse notes that the client has shortened muscles over a joint, preventing full extension. This condition is known as: 1 Osteoarthritis 2 Osteoporosis 3 Muscle atrophy 4 Contracture

4 Immobilized clients are at high risk for the development of contractures. Contractures are characterized by permanent shortening of the muscle covering a joint. Osteoarthritis is a disease process of the weight-bearing joints due to wear and tear. Osteoporosis is a metabolic disease process where the bones lose calcium. Muscle atrophy is a wasting and/or decrease in the strength and size of muscles due to a lack of physical activity or a neurological or musculoskeletal disorder.

The nurse receives information about a client through another nurse. The nurse then finds that information has some missing facts. Which critical thinking attitude would the nurse use to clarify the information after talking to the client directly? 1 Fairness 2 Humility 3 Discipline 4 Perseverance

4 Perserverance Perseverance requires the nurse to be cautious of an easy answer. If the nurse clarifies some information after talking to the client directly, he or she demonstrates perseverance. - Fairness requires the nurse to listen to both the sides in any discussion. - Humility is associated with recognizing the need for more information for making a decision. - When the nurse is thoroughly aware of what is required and manages his or her time effectively, he or she uses discipline.

A client is undergoing radiation therapy. The nurse reassures the client and stays with the client throughout the therapy. Which caring behavior does this nursing action reflect? 1 Touch 2 Spiritual caring 3 Knowing the client 4 Providing presence

4 Providing presence The nursing action of providing reassurance and being with the client reflects the caring behavior "providing presence." Providing presence conveys closeness and a sense of caring.

Which nursing action indicates that the nurse is actively listening to the client? 1 The nurse states his or her own opinions when the client is speaking. 2 The nurse refrains from telling his or her own story to the client. 3 The nurse reads the client's health record during the conversation. 4 The nurse interprets what the client is saying and reiterates in his or her own words.

4 The nurse interprets what the client is saying and reiterates in his or her own words. The nurse is listening actively if he or she is able to take in what the client says. A nurse who is listens attentively interprets and reiterates what the client is saying in his or her own words.

A client is being admitted to a medical unit with a diagnosis of pulmonary tuberculosis. The nurse should assign the client to which type of room? 1 Private room 2 Semi-private room 3 Room with windows that can be opened 4 Negative airflow room

4 Tuberculosis is an airborne contagious disease that is best contained in a negative airflow room. Negative airflow rooms are always private. A private room, semiprivate room, and a room with windows that can be opened are not appropriate for the standard of care for a client diagnosed with tuberculosis. Additionally, opening windows would present a possible safety hazard in a client's room.

6. Which assessment does a nurse interpret as a transfusion reaction? 1. Crackles in dependent lobes of lungs 2. High fever, severe hypotension 3. Anxiety, itching, confusion 4. Chills, tachycardia, and flushing

4. Chills, tachycardia, and flushing

The intake and output of a client over an eight-hour period is: 0800: Intravenous (IV) infusing; 900 mL left in bag; 0830: 150 mL voided; From 0900-1500 time period: 200 mL gastric tube formula + 50 mL water; Repeated x 2.; 1300: 220 mL voided; 1515: 235 mL voided; 1600: IV has 550 mL left in bag. What is the difference between the client's intake and output? Record the answer using a whole number. _________ mL

495 mL

The intake and output of a client over an 8-hour period (from 0800 to 1600) is as follows: 150 mL urine voided at 0800; 220 mL urine voided at 1200; 235 mL urine voided at 1600; 200 mL gastric tube formula + 50 mL water administered initially and then repeated x 2; IV had 900 mL in the bag at 0800, and 550 mL remains in the bag at 1600. What is the difference between the client's intake and output? Record your answer using a whole number.

495 ml

At the beginning of the shift at 7:00 am, a client has 650 mL of normal saline solution left in the intravenous bag, which is infusing at 125 mL/hr. At 9:30 am the healthcare provider changes the IV solution to lactated Ringer solution, which is to infuse at 100 mL/hr. What total amount of intravenous solution should the client have received by the end of the 8-hour shift? Record your answer using a whole number.

863 mL

Which client body temperatures are indicative of moderate hypothermia? Multiple selection question 80° F (26.7° C) 84° F (28.9° C) 88° F (31.1° C) 92° F (33.3° C) 96° F (35.6° C)

88° F (31.1° C) 92° F (33.3° C) Moderate hypothermia is a body temperature between 86°F and 93.2°F (30° C to 34° C). Therefore clients with body temperatures between 88°F and 92°F (31.1° C to 33.3° C) have moderate hypothermia. Mild hypothermia is a body temperature between 93.2°F and 96.8°F (34° C to 36° C). Therefore clients with body temperatures of 96°F (35.6° C) have mild hypothermia. Body temperature below 86°F (30° C) indicates severe hypothermia.

A nursing student is recording the radial pulse rate in a client with dysrhythmias and documented a radial pulse of 80 beats per minute. The registered nurse reassesses the client and notices a pulse deficit of 15. What is the client's apical pulse? Multiple choice question 95 85 75 65

95 Dysrhythmias are often associated with pulse deficits. A pulse deficit is the difference between the apical and radial pulse rates. Thus, when the radial pulse (80) and the pulse deficit (15) are added together, the apical pulse would be 95.

Which of the following clients is most at risk for fluid imbalance? A. An infant with diarrhea B. An adolescent mowing the lawn on a hot day C. A healthy 70-year-old man with a fractured wrist D. A middle-aged woman who is vomiting

A. And infant with diarrhea

When individuals are in a well or healthy state, their fluid output should be: A. Approximately the same as their fluid intake B. Correlated very little with their fluid intake C. Higher than their fluid intake D. Lower than their fluid intake

A. Approximately the same as their fluid intake

While assessing a client who experienced an accident, the nurse found that the client is unable to move eyeballs laterally. Which nerve damage led to this condition in the client? Multiple choice question Optic nerve Facial nerve Abducens nerve Oculomotor nerve

Abducens nerve The abducens nerve is the VI cranial nerve, which helps in lateral movement of the eyeballs. Damage to this nerve limits lateral movement of the eyeball. Injury to the optic nerve causes changes in visual acuity. Injury to the facial nerve results in loss of facial expressions and loss of taste perception from the anterior one third of the tongue. Injury to the oculomotor nerve limits the extraocular movements and pupillary responses.

Which of the following legal defenses is the most important for a nurse to develop? Dedication Certification Assertiveness Accountability

Accountability The concept of accountability is of high priority in nursing practice. As a licensed professional, the nurse is always accountable, which means liable and answerable for his or her actions. Dedication means to be committed, and assertiveness means to be confident. These are desired characteristics in a nurse but not legal defenses. Certification relates to achieving a higher level of knowledge or proficiency in one's area of specialization and is also not a legal defense.

The laboratory international normalized ratio (INR) results of a client receiving warfarin (Coumadin) have been variable. The nurse interviews the client to determine factors contributing to the problem. Which is most important for the nurse to identify? Use of analgesics Serum glucose level Serum potassium levels Adherence to the prescribed drug regimen

Adherence to the prescribed drug regimen The dosage of warfarin is adjusted according to INR results; if the client fails to take the drug as prescribed, test results will not be reliable in monitoring the client's response to therapy. Although some medications can affect the absorption or metabolism of warfarin and should be investigated, this is less likely to be a cause of fluctuations in laboratory values. Serum glucose level and serum potassium levels do not affect the absorption of warfarin.

Post-Op Atelectasis:

After surgery, it may be painful to breathe, patients don't move around as much as usual, so mucus clogs small airways, alveoli unable to open ("collapse"). To decrease risk of atelectasis - patient uses incentive spirometer. Have patient sit upright, seal lips around mouthpiece - breath in slowly and deeply (piston raises) & hold breath as long as possible (maintains a sustained inspiration, opens airways).

The nurse is teaching the parents of a 5-year-old with cystic fibrosis about respiratory treatments. Which statement indicates to the nurse that the parents understand? A. "Perform postural drainage before starting the aerosol therapy." B. "Ensure respiratory therapy is done daily during any respiratory infection." C. "Give respiratory treatments when the child is coughing a lot." D. "Administer aerosol therapy followed by postural drainage before meals."

Answer: "Administer aerosol therapy followed by postural drainage before meals." Postural drainage for a child with cystic fibrosis is most effective when performed after nebulization and before meals (C) or at least 1 hour after eating to prevent nausea and vomiting. Postural drainage uses gravity to promote mucous removal after nebulization (A) treatments which open the airways. Pulmonary toileting or respiratory treatments should be given 3 to 4 times daily, not episodically (B and D).

The nurse is assessing a 13-year-old girl with suspected hyperthyroidism. Which question is most important for the nurse to ask her during the admission interview? A. "Are you having any problems with your vision?" B. "When was the last time you took your synthroid?" C. "Have you lost any weight in the last month?" D. "Are you experiencing any type of nervousness?"

Answer: "Are you experiencing any type of nervousness?" Assessing the client's physiological state upon admission is a priority, and nervousness, apprehension, hyperexcitability, and palpitations are signs of hyperthyroidism (B). Weight loss (even with a hearty appetite) (A) occurs in those with hyperthyroidism, but assessing the client's neurological state has a higher priority. Hormone replacement is not administered to a client who is already producing too much thyroid (C). The client may have exophthalmus (bulging eyes) but hyperthyroidism does not cause vision problems (D).

The community health nurse teaches the parents of school-aged children about the need for fluoride as part of a dental health program. Which statement by the parents indicates that they understand the teaching? A. "Having our children brush with fluoride toothpaste is not effective." B. "Excessive amounts of fluoride will make teeth turn brittle and yellow." C. "Use of fluoride in water is mostly effective during initial tooth formation." D. "Dental caries can be prevented through fluoridation of public water."

Answer: "Dental caries can be prevented through fluoridation of public water." Dental caries can be prevented through fluoridation of public water (D). Large amounts of fluoride (A) produces yellow and discolored teeth, not brittle teeth. (B) is effective for young teeth. Fluoride is effective throughout the life span, not just during initial tooth formation (C).

A client who has been admitted to the psychiatric unit tells the nurse, "My problems are so bad that no one can help me." Which response is best for the nurse to make? A. "I hear how miserable you are, but things will get better soon." B. "Let's talk about what is right with your life." C. "How can I help?" D. "Things probably aren't as bad as they seem right now."

Answer: "How can I help?" Offering self shows empathy and caring (C), and is the best of the choices provided. Combining the first part of (C) with (B) would be the best response, but this is not a fill-in-the-blank or an essay test! Choose the best of those choices provided and move on. (D) dismisses the client, things are bad as far as this client is concerned. (A) avoids the client's problems and promotes denial. "I hear how miserable you are" is an example of reflective dialogue and would be the best choice if it were not for the rest of the sentence--"but things will get better" which is offering false reassurance.

A male client is admitted to a mental health unit on Friday afternoon and is very upset on Sunday because he has not had the opportunity to talk with the healthcare provider. Which response is best for the nurse to provide this client? A. "How can I help answer your questions?" B. "The healthcare provider should be here on Monday morning." C. "What concerns do you have at this time?" D. "Let me call and leave a message for your healthcare provider."

Answer: "Let me call and leave a message for your healthcare provider." It is best for the nurse to call the healthcare provider (D) because clients have the right to information about their treatment. Suggesting that the healthcare provider will be available the following day (B) does not provide immediate reassurance to the client. The nurse can also implement offer to assist the client (A and C), but the highest priority intervention is contacting the healthcare provider.

A client asks the nurse if glipizide (Glucotrol) is an oral insulin. Which response should the nurse provide? A. "No, it is not an oral insulin and can be used only when some beta cell function is present." B. "Yes, it is an oral insulin and has the same actions and properties as intermediate insulin." C. "No, it is not an oral insulin, but it is effective for those who are resistant to injectable insulins." D. "Yes, it is an oral insulin and is distributed, metabolized, and excreted in the same manner as insulin."

Answer: "No, it is not an oral insulin and can be used only when some beta cell function is present." An effective oral form of insulin has not yet been developed (C) because when insulin is taken orally, it is destroyed by digestive enzymes. Glipizide (Glucotrol) is an oral hypoglycemic agent that enhances pancreatic production of insulin. (A, B, and D) do not provide accurate information.

A premature newborn girl, born 24 hours ago, is diagnosed with a patent ductus arteriosus (PDA) and placed under an oxygen hood at 35%. The parents visit the nursery and ask to hold her. Which response should the nurse provide to the parents? A. "You can hold the baby with the oxygen blowing in the baby's face since the level is very close to room air." B. "Studies have shown that handling a sick newborn is not good for the baby and upsets the parents." C. "Since your baby has been doing well under oxygen for 24 hours, I can let you hold the baby without oxygen." D. "The oxygen hood is holding the baby's oxygen level just at the point which is needed. You may stroke and talk to her."

Answer: "The oxygen hood is holding the baby's oxygen level just at the point which is needed. You may stroke and talk to her." The baby is at 35% which is much more than room air (21%) and at this time the baby should not be moved from under the hood. The nurse should offer the parents an alternative such as to stroke and reassure the infant (B). Holding sick babies benefits the infant and the parents (A). The first consideration now has to be the infant's oxygenation. The nurse should not take the baby out from under the hood without a prescription from the healthcare provider, as this could severely compromise the infant (C). A P02 of 35% cannot be readily achieved with "blow by" oxygen (D).

A 9-month-old infant receives a prescription for digoxin 40 mcg PO daily. Digoxin Oral Solution, USP 50 mcg (0.05 mg) per ml is available. How many ml should the nurse administer? (Enter the numerical value only. If rounding is required, round to the nearest tenth.)

Answer: 0.8 ml Using ratio and proportion, 40 mcg : X ml :: 50 mcg : 1 ml = 0.8 ml

A child who is scheduled for a kidney transplant receives a prescription for basiliximab (Simulect) 20 mg IV 2 hours prior to surgery. The medication is available in a 20 mg vial that is reconstituted by adding 5 ml sterile water for injection, and administered in a 50 ml bag of normal saline over 30 minutes. The nurse should program the infusion pump to deliver how many ml/hour? (Enter the numeric value only.)

Answer: 110 ml/hr After reconstituting the medication vial, the nurse adds the 5 ml of medication to the 50 ml of sterile water to result in a 55 ml volume to infuse in 30 minutes. Using the formula, Volume/Time = 55 ml / 0.5 hours = 110 ml/hour

A client with hypernatremia is to receive an IV infusion of 1000 ml 5% dextrose in water to be infused over 6 hours. The nurse should program the infusion pump to deliver how many ml/hour? (Enter numeric value only. If rounding is required, round to the nearest whole number.)

Answer: 167 ml/hr Calculate using the ratio: 1000 ml : 6 hours :: X ml : 1 hour 1000/X :: 6/1 6X = 1000 X = 167 ml/hour

A client who is complaining of nausea and vomiting postoperatively receives a prescription for metoclopramide (Reglan) 10 mg IV. The medication is available in vials containing 5 mg/ml. How many ml should the nurse administer? (Enter numeric value only.)

Answer: 2 ml 10 mg : X ml = 5 mg : 1 ml 5X = 10 X = 2 ml

The healthcare provider prescribes furosemide (Lasix) 25 mg IV. The drug is available in a solution of 40 mg/4ml. How many ml should the nurse administer? (Enter numeric value only. If rounding is required, round to the nearest tenth.)

Answer: 2.5 ml Using the formula D/H x Q: 25 mg/40 mg x 4 ml = 2.5 ml

0.9% normal saline with inamrinone (Inocor) 0.1 grams/100 ml is prescribed for client with heart failure. The medication is to be delivered at a rate of 400 mcg/minute. The nurse should program the infusion pump to deliver how many ml/hour? (Enter numeric value only. If rounding is required, round to the nearest whole number.)

Answer: 24 ml/hr First calculate the number of mcg/hour: 400 mcg x 60 minutes = 24000 mcg/hour Next calculate the number of ml/hour needed to administer 24,000 mcg/hour: 100,000 mcg: 100ml :: 24,000 mcg : X 100,000/24,000 :: 100/X 100,000X = 2,400,000 X = 24 ml/hour

Which finding should the nurse identify as most significant for a client diagnosed with polycystic kidney disease (PKD)? A. 2 pounds weight gain. B. Hematuria. C. 3+ bacteria in urine. D. Steady, dull flank pain.

Answer: 3+ bacteria in urine. Urinary tract infections (UTI) for a client with PKD require prompt antibiotic therapy to prevent renal damage and scarring which may cause further progression of the disease, so bacteria in the urine (C) is the most significant finding at this time. (A) is an expected finding from the rupture of the cysts. (B) does not provide a time frame to determine if the weight gain is a significant fluid fluctuation, which is determined within a 24-hour time frame. Although kidney pain can also be abrupt, episodic, and colicky related to bleeding into the cysts, (D) is more likely an early symptom in PKD.

The nurse is administering the measles, mumps, rubella (MMR) vaccine to a 12-month old child during the well-baby visit. Which age range should the nurse advise the parents to plan for their child to receive the MMR booster based on the current recommendations and guidelines by the Center for Disease Control (CDC)? A. 18 to 24 months of age. B. 11 to 12 years of age. C. 13 to 18 years of age. D. 4 to 6 years of age.

Answer: 4 to 6 years of age. The second booster of the measles, mumps, rubella (MMR) vaccine is recommended by the CDC for routine immunization at 4 to 6 years of age (D), which is commonly required prior to entrance into elementary school. Those who have not previously received the second dose should complete the schedule by 11 to 12 years of age (A and B). The MMR may be administered during any visit, provided at least 4 weeks have elapsed since the first dose and both doses are administered beginning at or after 12 months of age (C).

Which pediatric client requires immediate intervention by the nurse? A. A 4-year-old with an easily palpable bladder and frequency. B. A 3-year-old with several episodes of nocturnal enuresis. C. A 5-year-old with diuresis following furosemide (Lasix) administration. D. A 2-year-old with a twenty-four hour urinary output of 500 ml.

Answer: A 4-year-old with an easily palpable bladder and frequency. Frequency and bladder distention (C) are indications of urinary retention, which requires immediate intervention by the nurse. (A) is the normal output for a child of this age. (B) describes bed-wetting, not uncommon in a child of this age, although if the problem persists in a child older than 5 years of age, further assessment and intervention is warranted. (D) is an expected response to the medication, which requires routine monitoring, but does not indicate a need for immediate intervention.

Which client should the nurse identify as the highest risk for the onset of stress-related problems? A. A man whose new business is growing slowly, who plans to adopt a child with his wife, and says, "I think I'm in control of my destiny." B. A woman who is graduating from college, getting married in one month, and states, "I'm anticipating the changes these events will make in my life." C. A person whose father died three months ago, who is losing a job due to company downsizing, and states, "Living with loss and the threat of loss makes me feel helpless." D. A client who is passed over for promotion, quits a job to start a new business, and states, "This is just one of a series of challenges I've faced in my life."

Answer: A person whose father died three months ago, who is losing a job due to company downsizing, and states, "Living with loss and the threat of loss makes me feel helpless." A client who is dealing with two stressful life events and expresses a cognitive appraisal of loss and helplessness (D) is at the highest risk for a stress-related health problem. (A, B, and C) describe persons who are coping with change using healthy strategies, such as perceiving change as challenging, expressing commitment to change, and believing they have control over their life paths.

Which growth and development characteristic should the nurse consider when monitoring the effects of a topical medication for an infant? A. A thin stratum corneum that increases topical absorption. B. A greater body surface area that requires larger dosages. C. A lower sensitivity reactions to skin irritants. D. A smaller percentage of muscle mass.

Answer: A thin stratum corneum that increases topical absorption. infants have a thin outer skin layer (stratum corneum), so the nurse should monitor the infant for a prompt onset and response to the application of topical medication (B). (A, C, and D) are unrelated to topical medication administration.

Which client should the nurse identify as being at highest risk for complications during the use of an opioid analgesic? A. A client with a open compound fracture. B. An older client with Type 2 diabetes mellitus. C. A client with chronic rheumatoid arthritis. D. A young adult with inflammatory bowel disease.

Answer: A young adult with inflammatory bowel disease The principal indication for opioid use is acute pain, and a client with inflammatory bowel disease (D) is at risk for toxic megacolon or paralytic ileus related to slowed peristalsis, a side effect of morphine. Adverse effects of morphine do not pose as great a risk for (A, B, and C) as the client with bowel disease.

A 17-year-old unmarried, pregnant client with drug addiction is a high school dropout, homeless, and has a history of past abuse arrives at the clinic for her first prenatal visit. Which findings should the nurse document as health risk factors for the client? (Select all that apply.) A. Age. B. History of abuse. C. Pregnancy. D. Drug addiction. E. School dropout. F. Homelessness. G. Unmarried.

Answer: A, B, C, D, F Health risk factors for this client include (A, C, D, E and F). Each factor should be considered individually. The client, as an adolescent mother, is at high risk for nutritional deficits, anemia, gestational diabetes and hypertension, which also impact the fetus' risk for small for gestational age, fetal anomalies, and fetal demise. (B and G) may impact the client's social adaptation, but do not directly constitute health risk factors.

The nurse is assessing a client admitted from the emergency room with gastrointestinal bleeding related to peptic ulcer disease (PUD). Which physiological factors can produce ulceration? (Select all that apply.) A. Decreased duodenal inhibition. B. An increased level of stress. C. An increased number of parietal cells D. Vagal stimulation. E. Hypersecretion of hydrochloric acid.

Answer: A,C,D,E Correct selections are (A, C, D, and E). Hypersecretion of gastric juices (D) and an increased number of parietal cells (E) that stimulate secretion are most often the causes of ulceration. Vagal stimulation (A) and decreased duodenal inhibition (C) also increase the secretion of caustic fluids. An increased stress level is not physiologic and is not a direct cause of ulceration (B).

A female nurse who sometimes tries to save time by putting medications in her uniform pocket to deliver to clients, confides that after arriving home she found a hydrocodone (Vicodin) tablet in her pocket. Which possible outcome of this situation should be the nurse's greatest concern? A. Accused of diversion. B. Accused of unprofessional conduct. C. Reported for stealing. D.Reported for a HIPAA violation.

Answer: Accused of diversion. Even if this is only one incident, the nurse may be suspected of taking medications on a regular basis and the incident could be interpreted as diversion (A), or diverting narcotics for her own use, which should be reported to the peer review committee and to the State Board of Nursing. (B, C, and D) are also of concern, but (A) is the most serious possible outcome.

When planning care for a client with right renal calculi, which nursing diagnosis has the highest priority? A. Risk for infection related to urinary stasis. B. Deficient knowledge related to need for prevention of recurrence of calculi. C. Acute pain related to movement of the stone. D. Impaired urinary elimination related to obstructed flow of urine.

Answer: Acute pain related to movement of the stone. The nursing diagnosis of highest priority is acute pain (A), which if unresolved can represent pathology affecting renal function. Impaired urinary elimination (B), risk for infection (C), and knowledge deficit (D) are components of the plan of care with less immediacy than management of the etiology of the client's pain.

An adult client has prescriptions for morphine sulfate 2.5 mg IV q6h and ketorolac (Toradol) 30 mg IV q6h. Which action should the nurse implement? A. Hold the ketorolac to prevent an antagonistic effect. B. Hold the morphine to prevent an additive drug interaction. C. Contact the healthcare provider to clarify the prescription. D. Administer both medications according to the prescription.

Answer: Administer both medications according to the prescription. Morphine and ketorolac (Toradol) can be administered concurrently (A), and may produce an additive analgesic effect, resulting in the ability to reduce the dose of morphine, as seen in this prescription. Toradol is an antiinflammatory analgesic, and does not have an antagonistic effect with morphine (B), like an agonist-antagonist medication would have. An additive analgesic effect is desirable (C), because it allows a reduced dose of morphine. This prescription does not require any clarification, and can be administered safely as written (D).

The primary nurse receives the 0700 shift report for 4 clients on a medical unit. When prioritizing care, which action should the nurse implement first? A. Flush the lumen of a client's triple lumen central venous catheter with saline. B. Review the potassium levels of a client who receives a daily loop diuretic. C. Administer insulin per sliding scale to a client with a capillary glucose of 285. D. Assess the lung sounds of a client with pneumonia who is ready to go home.

Answer: Administer insulin per sliding scale to a client with a capillary glucose of 285. The nurse should first administer the insulin per scaling scale (A) to the client with hyperglycemia to prevent further elevation of the serum glucose levels. (B and C) are of less immediacy and can be delayed until the higher priority interventions are completed. The client's potassium level should be checked before administering a loop diuretic (D), but this can be done after administering the sliding scale insulin.

A client receiving albuterol (Proventil) tablets complains of nausea every evening with her 9:00 p.m. dose. What action can the nurse take to alleviate this side effect? A. Administer the dose with a snack. B. Change the time of the dose. C. Hold the 9 p.m. dose. D. Administer an antiemetic with the dose.

Answer: Administer the dose with a snack. Administering oral doses with food (C) helps minimize GI discomfort. (A) would be appropriate only if changing the time of the dose corresponds to meal times while at the same time maintaining an appropriate time interval between doses. (B) would disrupt the dosing schedule, and could result in a nontherapeutic serum level of the medication. (D) should not be attempted before other interventions, such as (C), have been proven ineffective in relieving the nausea.

Based on the blood culture and sensitivity results, the healthcare provider prescribes an IV aminoglycoside antibiotic and discontinues the current prescription for another broad spectrum antibiotic. The medication administration record indicates that the client received the broad spectrum antibiotic two hours ago. Which action should the nurse implement? A. Obtain peak and trough serum levels so the aminoglycoside antibiotic can be initiated. B. Schedule the initial dose of the aminoglycoside antibiotic for the following day. C. Withhold antibiotic administration until the healthcare provider clarifies the prescriptions. D. Administer the initial dose of the aminoglycoside antibiotic as soon as possible.

Answer: Administer the initial dose of the aminoglycoside antibiotic as soon as possible. The blood culture and sensitivity results identify the specific antibiotic that is most effective in treating the client's infection, so the aminoglycoside antibiotic should be administered as soon as possible (B). Obtaining peak and trough levels (A) before starting administration of the aminoglycoside provides no useful data. The prescription does not need clarification (C) from the healthcare provider. The aminoglycoside antibiotic is the correct antibiotic to treat the infection and should be started as soon as possible, rather than waiting until the next day (D).

A client with heart failure is prescribed digoxin (Lanoxin) 0.125 mg PO. The client's apical heart rate is 70 beats per minute, blood pressure is 125/75 mmHg, and respirations are 18 breaths per minute. Which action should the nurse implement next? A. Administer the medication. B. Reassess the apical heart rate. C. Review the vital sign flowsheet. D. Inform the healthcare provider.

Answer: Administer the medication Obtaining the apical heart rate is a common parameter prior to administering digoxin, which may indicate early digoxin toxicity if the heart rate is less than 60 beats per minute, so the dose should be administered (A) since the client is not demonstrating any signs of toxicity. (B and D) are not necessary because the apical pulse is above 60 beats per minute. Review of the client's past vital signs (C) provides data for evaluation of the client's clinical progress, but based on the client's present clinical findings, the medication should be administered next.

A 45-year-old female client is admitted to the psychiatric unit for evaluation. Her husband states that she has been reluctant to leave home for the last six months. The client has not gone to work for a month and has been terminated from her job. She has not left the house since that time. This client is displaying symptoms of what condition? A. Acrophobia. B. Claustrophobia. C. Post-traumatic stress disorder. D. Agoraphobia

Answer: Agoraphobia Agoraphobia (C) is the fear of crowds or being in an open place. (B) is the fear of being in closed places. (A) is the fear of high places. Remember, a phobia is an unrealistic fear which is associated with severe anxiety. (D) consists of the development of anxiety symptoms following a life event that is particularly serious and stressful (war, witnessing a child killed, etc.) and is experienced with terror, fear, and helplessness--a phobia is different.

A 6-month-old boy and his mother are at the healthcare provider's office for a well-baby check-up and routine immunizations. The healthcare provider recommends to the mother that the child receive an influenza vaccine. What medications should the nurse plan to administer today? A. The routine immunizations and schedule another appointment to administer the influenza vaccine. B. The influenza vaccine and schedule another appointment to administer the immunizations. C. All the immunizations with the influenza vaccine given at a separate site from any other injection. The influenza vaccine and the polio vaccine and schedule another appointment to administer the remaining immunizations.

Answer: All the immunizations with the influenza vaccine given at a separate site from any other injection. At 6-months of age, the routine immunizations include Hepatitis B, DTaP, Hib (Haemophilus influenza type b) , PCV (Pneumococcal), IPV (inactivated poliovirus) and influenza. The influenza vaccine should be given at a separate site from any other injection (B). Scheduling a return visit (A, B, or C) increases the risk that the mother will not bring the child back for the immunizations.

A client is brought to the hospital in cardiac arrest by emergency personnel who are performing resuscitation. The spouse arrives as the client is taken into a treatment room and asks to stay with the client. What action should the nurse implement? A. Advise the spouse that if unsuccessful, the resuscitation scene should not be the last memory of a loved one. B. Insist that the spouse wait outside the room while resuscitation is being performed. C. Allow the spouse to be present and ensure that a member of the team explains the care given and answers questions D. Explain to the spouse that there will be no time for explanations during the resuscitation efforts.

Answer: Allow the spouse to be present and ensure that a member of the team explains the care given and answers questions. Research supports the positive benefits of family presence during invasive procedures and cardiopulmonary resuscitation to clients, family, and staff. Facilitating family presence allows family to view themselves of active participants and completes the last step of the secondary survey in the care of an emergency client (B). (A) does not facilitate family presence. Someone should be assigned to the family to explain the care being delivered and to answer questions, not (C). (D) does not offer the spouse support and is not recommended.

Within several days of hospitalization, a client is repeatedly washing the top of the same table. Which initial intervention is best for the nurse to implement to help the client cope with anxiety related to this behavior? A. Teach the client relaxation and thought stopping techniques. B. Administer a prescribed PRN antianxiety medication. C. Assist the client to identify stimuli that precipitates the ritualistic activity. D. Allow time for the ritualistic behavior, then redirect the client to other activities.

Answer: Allow time for the ritualistic behavior, then redirect the client to other activities. Initially, the nurse should allow time for the ritual (C) to prevent anxiety. (A) may help reduce the client's anxiety, but will not prevent ritualistic behavior resulting from the client's ineffective coping ability. (B) is a long-term goal of individual therapy, but is not directly related to controlling the behavior at this time. (D) lists techniques that can be used to assist the client in learning new ways of interrupting obsessive thoughts and resulting ritualistic behavior as treatment progresses.

Which rationale best supports an older client's risk of complications related to a dysrhythmia? A. Cardiac symptoms, such as confusion, are more difficult to recognize in an older client. B. An older client is intolerant of decreased cardiac output which may cause dizziness and falls. C. An older clients is more likely to eat high-fat diets which predisposes to heart disease. D. An older client usually lives alone and cannot summon help when symptoms appear.

Answer: An older client is intolerant of decreased cardiac output which may cause dizziness and falls. In an older client, cardiac output is decreased and a loss of contractility and elasticity reduces systemic and cerebral blood flow, so dysrhythmias, such as bradycardia or tachycardia is poorly tolerated, and increases the client's risk for syncope, falls, transient ischemic attacks, and possibly dementia. (B and C) are generalized statements that are not applicable to most individuals in the older population. Although many older persons do live alone, inability to summon help (A) cannot be assumed.

During routine screening at a school clinic, an otoscope examination of a child's ear reveals a tympanic membrane that is pearly gray, slightly bulging, and not movable. What action should the nurse take next? A. No action required, as this is an expected finding for a school-aged child. B. Ask the child if he/she has had a cold, runny nose, or any ear pain lately. C. Send a note home advising the parents to have the child evaluated by a healthcare provider as soon as possible. D. Call the parents and have them take the child home from school for the rest of the day.

Answer: Ask the child if he/she has had a cold, runny nose, or any ear pain lately. More information is needed to interpret these findings (B). The tympanic membrane is normally pearly gray, not bulging, and moves when the client blows against resistance or a small puff of air is blown into the ear canal. Since this child's findings are not completely normal, further assessment of history and related signs and symptoms is indicated for accurate interpretation of the findings. (A, C, and D) are inappropriate actions based on the data obtained from the otoscope examination.

The nurse notes a client's postoperative leg is cool with a capillary refill greater than 4 seconds and calls the healthcare provider. After 30 minutes of not receiving a return call from the healthcare provider, which action should the nurse take first? A. Continue to monitor and call if there is a change. B. Attempt to recall the same healthcare provider. C. Describe the problem to the answering service. D. Notify the hospital's "on call" nursing supervisor.

Answer: Attempt to recall the same healthcare provider. The healthcare provider may have inadvertently not received the first call, so (A) is the best action to take first. According to the TeamSTEPPS, two attempts should be made to notify the provider before proceeding through the chain of command (B). (C) should be implemented, but these assessment findings require immediate medical action. Although (D) is an option, the client's urgent condition needs treatment.

A client with pneumonia receives a prescription for tetracycline (Sumycin). What precaution should the nurse include in this client's teaching? A. Avoid diary products for 2 hours after taking the medication. B. Avoid over-the-counter medications containing alcohol. C. Do not use teeth whitening agents during the treatment regimen. D. Take the medication with a glass of orange juice

Answer: Avoid diary products for 2 hours after taking the medication. Dairy products should be ingested at least 2 hours after taking Sumycin (C) because calcium binds with tetracycline and decreases its absorption. Sumycin can be taken with orange juice (A) because it does not affect absorption of the medication. Sumycin does not cause a disulfiram-like reaction, so (B) is not indicated. Although Sumycin causes enamel hypoplasia and permanent yellow, gray, or brown staining of the teeth during the ages of tooth development (children younger than 8 years of age), it does not affect adult enamel, so (D) is not indicated.

A 4-year-old is brought to the emergency room for a laceration on the right foot. What action should the nurse implement to help the child in coping with the emergency room experience? A. Avoid using jargon, such as a "shot" when giving care B. Give the child some time after explaining procedures. C. Remind the preschooler how big children should act. D. Avoid the use of bandages to keep wounds open to air.

Answer: Avoid using jargon, such as a "shot" when giving care. Using positive terms and avoiding words that have frightening connotations (D) assist the preschool-age child in coping with an emergency room experience. Bandages (A) are important to preschool-aged children because this age group often believe bandages stop their insides from leaking out. Children need to feel comfortable expressing their fears and feelings and should not be shamed into cooperation by referencing expected "big" children behaviors (B). Preschool-age children should be told about procedures immediately before they are performed (C), which minimizes the time a child fantasies about the treatment, which causes increased anxiety.

Which task should the nurse delegate to an unlicensed assistive personnel (UAP)? A. Teach insulin self-administration for a client with Type 1 diabetes. B. Update the nutrition needs in the plan of care. C. Evaluate goal attainment for a client with a below-the-knee prosthesis. D. Bathe an unconscious client with decubitus ulcers.

Answer: Bathe an unconscious client with decubitus ulcers. Delegation requires determining which staff member is capable of performing what tasks. Basic hygiene (B) is within the role of the UAP. Coordination and planning of care (A), teaching (C), and evaluating desired goal attainment or client outcomes (D) are responsibilities outside the scope of practice for the UAP, and within that of the nurse.

A client who is intoxicated is admitted for alcohol and multiple substance detoxification. The nurse determines that the client is becoming increasingly anxious, agitated, and diaphoretic. The client is also experiencing sensory perceptual disturbances and a clouded sensorium. What is the priority nursing intervention for this client at this time? A. Begin one-on-one supervision immediately. B. Keep the room dimly lit and turn on the radio. C. Push fluids and provide calorie-rich nutritional supplements. D. Check on the client every 15 minutes.

Answer: Begin one-on-one supervision immediately. One-on-one supervision (B) ensures the client's physical safety until the client is sedated adequately to reduce feelings of terror and tactile and visual hallucinations. Checking every 15 minutes (A) does not provide sufficient assessment of the client's safety. Additional auditory stimulation and a dimly lit room (C) can create illusions that contribute to the client's altered sensory distress and should be avoided. Fluid replacement and nutritional supplements (D) should be initiated when the client is more stable because the risk for overhydration can occur as blood alcohol levels fall and fluids are retained.

The nurse notes that the only ECG for a 55-year-old male client scheduled for surgery in two hours is dated two years ago. The client reports that he has a history of "heart trouble," but has no problems at present. Hospital protocol requires that those over 50 years of age have a recent ECG prior to surgery. What nursing action is best for the nurse to implement? A. Notify the client's surgeon immediately. B. Ask the client what he means by "heart trouble." C. Notify surgery that the ECG is over two years old. D. Call for an ECG to be performed immediately.

Answer: Call for an ECG to be performed immediately Clients over the age of 40 and/or with a history of cardiovascular disease, should receive ECG evaluation prior to surgery, generally 24 hours to two weeks before. (D) should be implemented to ensure that the client's current cardiovascular status is stable. Additional data might be valuable (B), but since time is limited, the priority is to obtain the needed ECG. Documentation of vital signs is important, but does not replace the need for the ECG (C). The surgeon only needs to be notified if the ECG cannot be completed, or if there is a significant problem (A).

The nurse is instructing an adolescent with bulimia and a low potassium level about the risk for complications. Which medical problem should be the focus of the nurse's instruction to this client? A. Heightened neurologic reflexes. B. Gastrointestinal reflux. C. Anemia. D. Cardiac arrhythmias.

Answer: Cardiac arrhythmias. An adolescent with bulimia who purges by frequent self-induced vomiting, diuretic or laxative abuse can experience potassium depletion, which increases the risk for cardiac arrhythmias (B). (A) is more likely related to inadequate iron intake and absorption, not hypokalemia. (C) is related to frequent binging and gastric over-distention. Potassium depletion causes diminished reflexes, not (D)

The parents of a toddler brought to the clinic for a well-child visit tell the nurse that their child becomes upset if even the smallest things change in the environment. What information should the nurse provide the parents? A. A child is insecure because trust is not fostered and developed during infancy. B. A toddler should be exposed to different routines to promote adapting to new experiences. C. Children of this age are comfortable with ritualism and display global thinking. D. Should be frequently moved in the environment to teach the child to acclimate to change.

Answer: Children of this age are comfortable with ritualism and display global thinking. A 2-year-old is ritualistic and wants consistency and routine, so changes in the toddler's environment or schedule is upsetting. Another mark of the toddler's sensitivity to change is global thinking (change in one small part, such as a minor shift in room arrangement or changes in the whole environment), and the 2-year-old's equanimity disintegrates (C). There is not enough information to make the assumption the child did not develop trust (A). Frequent changes (B and D) in the schedule or the environment can lead to insecurity on the part of the toddler.

A client with chronic kidney disease (CKD) selects a scrambled egg for his breakfast. What action should the nurse take? A. Suggest that the client also select orange juice, to promote absorption. B. Commend the client for selecting a high biologic value protein. C. Encourage the client to attend classes on dietary management of CKD. B. Remind the client that protein in the diet should be avoided.

Answer: Commend the client for selecting a high biologic value protein. Foods such as eggs and milk (A) are high biologic proteins which are allowed because they are complete proteins and supply the essential amino acids that are necessary for growth and cell repair. Although a low-protein diet is followed (B), some protein is essential. Orange juice is rich in potassium, and should not be encouraged (C). The client has made a good diet choice, so (D) is not necessary.

A preschool-age child who is hospitalized for hypospadias repair is most strongly influenced by which behavior? A. Response to separation from family. B. Concern for body integrity. C. Socialization with other children. D. Ability to communicate verbally.

Answer: Concern for body integrity The preschooler's major stressor is concern for his body integrity (C). He fears that his "insides will leak out." A child undergoing surgery to his genitalia is even more concerned about body integrity. The preschooler is quite verbal, so comprehension of the words he uses or hears may be inaccurate, while his imagination and fears may fantasize the reality (A). (B) is a concern for all children, but of most concern to the toddler. (D) is not a prime concern in this situation.

A 2-year-old child with gastro-esophageal reflux has developed a fear of eating. What instruction should the nurse include in the parents' teaching plan? A. Consistently follow a set mealtime routine. B. Invite other children home to share meals. C. Reward the child with a nap after eating. D. Accept that he will eat when he is hungry.

Answer: Consistently follow a set mealtime routine. A 2-year-old child is comforted by consistency (D). (A) is contraindicated because two-year-olds may participate in parallel activities with other children but are too young to feel comfort and support by the presence of other children when anxious or afraid. (B) may or may not be true and does not address the child's fears. The child with reflux should remain upright at least two hours after eating (C) to reduce symptoms.

A client who has moderate, persistent, chronic neuropathic pain due to diabetic neuropathy takes gabapentin (Neurontin) and ibuprofen (Motrin, Advil) daily. If Step 2 of the World Health Organization (WHO) pain relief ladder is prescribed, which drug protocol should be implemented? A. Add oral methadone to the protocol. B. Discontinue ibuprofen. C. Continue gabapentin. D. Add aspirin to the protocol.

Answer: Continue gabapentin. Based on the WHO pain relief ladder, adjunct medications, such as gabapentin (Neurontin), an antiseizure medication, may be used at any step for anxiety and pain management, so (A) should be implemented. Nonopiod analgesics, such as ibuprofen (A) and aspirin (C) are Step 1 drugs. Step 2 and 3 include opioid narcotics (D), and to maintain freedom from pain, drugs should be given €Å"around the clock€� rather than by the client's PRN requests.

During report, the charge nurse informs a nurse that she must work on another unit. The nurse begins to sigh deeply and tosses about her belongings as she is preparing to leave, making it known that she is very unhappy about having to "float." What is the best immediate action for the charge nurse to take? A. In the presence of other staff members, inform the nurse that her behavior is inappropriate. B. Ask the nurse to call the supervisor to see if she can be reassigned. C. Continue with report, and talk to the nurse about the incident at a later time. D. Stop report and remind the nurse that all staff must "float" at some time.

Answer: Continue with report, and talk to the nurse about the incident at a later time. (A) is the best immediate action. At a later time (after the nurse has "cooled off") the charge nurse should discuss with the nurse in private her inappropriate conduct. (B) only reinforces inappropriate behavior and dismisses the problem to the supervisor. (C and D) would incite conflict in that both actions would likely encourage justification and argumentative behavior, and reprimanding the nurse in front of colleagues is poor management (D). The first priority is to provide care for clients--hopefully, traveling to the unit will provide a "cooling off" time for the nurse.

During the well-child assessment of an 18-month-old male toddler, the nurse determines the child does not walk while holding on to furniture but prefers to crawl, rarely speaks, has a flat affect, and is small for his age. Which nursing diagnosis should the nurse formulate? A. Delayed growth and development. B. Alteration in health maintenance. C. Alteration in parenting. D. Alteration in nutrition.

Answer: Delayed growth and development. This child does not demonstrate gross motor or psychosocial skills typical of an 18-month-old toddler, which best supports delayed growth and development (C). Additional information about the child's growth parameters is needed to support (A, B, or D).

A seven-month old infant is admitted with nonorganic failure to thrive (NFTT). To aid the child's growth and development, which intervention is most important for the nurse to implement? A. Provide instructions about formula preparation and feeding schedules. B. Demonstrate feeding strategies and infant cues that indicate hunger and satiation. C. Encourage the parents to participate in a planned program of play with the infant. D. Refer the parents for psychological counseling to identify parental detachment.

Answer: Demonstrate feeding strategies and infant cues that indicate hunger and satiation. NFTT most often occurs due to inadequate parent knowledge or a disturbance in maternal-child attachment, but the first goal for infants with NFTT is to provide nutrition to promote "catch-up" growth. The nurse should demonstrate positive feeding strategies that reduce parent and infant frustration, such as recognizing the infant's cues indicated by vigorous sucking and satiation (C). (A) encourages normal growth and development, but is not likely to teach the parents how to respond to the infant's nutritional needs. Although family dysfunction may contribute to NFTT and (B) may eventually be indicated, additional assessment is needed before such a referral is made. (D) provides a structured schedule, but positive infant feeding strategies should be implemented first.

A woman with Type 2 diabetes mellitus becomes pregnant, and her oral hypoglycemic agents are discontinued. Which intervention is most important for the nurse to implement? A. Describe diet changes that can improve the management of her diabetes. B. Evaluate the client's ability to do glucose monitoring. C. Inform the client that oral hypoglycemic agents are teratogenic during pregnancy. D. Demonstrate self-administration of insulin.

Answer: Describe diet changes that can improve the management of her diabetes. Diet modifications (A) are effective in managing Type 2 diabetes during pregnancy, and describing the necessary diet changes is the most important intervention for the nurse to implement with this client. (B, C, and D) are interventions that should also be implemented, but do not have the priority of (A).

What action should the nurse implement to provide analgesic titration for a client in pain? A. Teach the client to increase the time range between doses of pain medication. B. Monitor the effects of continuous intravenous infusion of narcotic analgesics. C. Determine the optimal analgesic dosage required that causes the least side effects. D. Plan with the client how to use a specific total dose of analgesic over a 24-hour period.

Answer: Determine the optimal analgesic dosage required that causes the least side effects. No given dosage of an analgesic provides the same level of pain relief for every patient, and so titration upward or downward is determined based on the client's response, so that the optimal dosage achieves adequate pain relief with minimal side effects (D) for the client. Titration does not necessarily mean continuous intravenous infusion (B), but considers dose adjustments to achieve a therapeutic analgesic response. An individual's response to the medication dosage is the assessment for titration, not a specific total dose over 24 hours (C). Although (A) may be a component of pain management, particularly during rehabilitation or remission, the titration dose should be implemented as long as analgesia is needed.

The nurse is caring for a male client who had an inguinal herniorrhaphy 3 hours ago. The nurse determines the client's lower abdomen is distended and assesses dullness to percussion. What is the priority nursing action? A. Assessment of the client's vital signs. B. Determine the time the client last voided. C. Document the finding as the only action. D. Insert a rectal tube for the passage of flatus.

Answer: Determine the time the client last voided. Swelling at the surgical site in the immediate postoperative period can impact the bladder and prostate area causing the client to experience difficulty voiding due to pressure on the urethra. To provide additional data supporting bladder distention, the last time the client voided (C) should be determined next. Documentation (B) should be made, but the client's distended bladder requires additional intervention. (A and D) are not priority actions based on the client's abdominal findings.

A 75-year-old client who has a history of end stage renal failure and advanced lung cancer, recently had a stroke. Two days ago the healthcare provider discontinued the client's dialysis treatments, stating that death is inevitable, but the client is disoriented and will not sign a DNR directive. What is the priority nursing intervention? A. Review the client's most recent laboratory reports. B. Determine who is legally empowered to make decisions. C. Notify the hospital ethics committee of the client situation. D. Refer the client and family members for hospice care.

Answer: Determine who is legally empowered to make decisions. When death is impending, it is essential for the nurse to determine who is legally empowered to make decisions regarding the use of life-saving measures for the client (D). (A) will be abnormal and will worsen without dialysis, so are not of immediate concern. (B) may help improve the client's quality of life prior to death, but is of less immediacy than determining whether actions should be taken to save a client's life. If the nurse remains unable to determine who is empowered to make decisions in this situation, the nurse may choose to contact the ethics committee (C) for a resolution.

The mother of a preschool-aged child asks the nurse if it is all right to administer Pepto Bismol to her son when he "has a tummy ache." After reminding the mother to check the label of all over-the-counter drugs for the presence of aspirin, which instruction should the nurse include when replying to this mother's question? A. Pepto Bismol may cause a rebound hyperacidity, worsening the "tummy ache." B. Do not give if the child has chickenpox, the flu, or any other viral illness. C. If the child's tongue darkens, discontinue the Pepto Bismol immediately. D. Avoid the use of Pepto Bismol until the child is at least 16 years old.

Answer: Do not give if the child has chickenpox, the flu, or any other viral illness. Pepto Bismol contains aspirin and there is the potential of Reye's syndrome (B). (A) is a common effect of Pepto Bismol and does not warrant discontinuation. Pepto Bismol can be used by children (C). Pepto Bismol does not cause rebound hyperacidity (D), which is a complication of antacids containing calcium.

A client who has been NPO for 3 days is receiving an infusion of D5W 0.45 normal saline (NS) with potassium chloride (KCl) 20 mEq at 83 ml/hour. The client's eight-hour urine output is 400 ml, blood urea nitrogen (BUN) is 15 mg/dl, lungs are clear bilaterally, serum glucose is 120 mg/dl, and the serum potassium is 3.7 mEq/L. Which action is most important for the nurse to implement? A. Decrease in the infusion rate of the current IV and report to the healthcare provider. B. Obtain potassium chloride 20 mEq in anticipation of a prescription to add to present IV. C. Notify healthcare provider and request to change the IV infusion to hypertonic D10W. D. Document in the medical record that these normal findings are expected outcomes.

Answer: Document in the medical record that these normal findings are expected outcomes. The results are all within normal range (C). No changes are needed (A, B, and D).

A young adult female client with panic disorder arrives in the Emergency Center with a 4-day history of chest pain that began when her boyfriend left her. Initial assessment reveals normal cardiopulmonary findings. Which information is most important for the nurse to obtain? A. Family history of suicide. B. Drugs taken in last 7 days. C. Frequency of anxiety attacks. D. Usual coping mechanisms.

Answer: Drugs taken in last 7 days. Use of prescribed, over-the-counter, and illicit drugs (A) is the most important information to obtain when planning care because drugs are likely to influence the client's behavior and ability to cope with stressful situations. (B, C, and D) are worthwhile assessment findings, but they do not have the priority of (A).

During the initial morning assessment, a male client denies dysuria but reports that his urine appears dark amber. Which intervention should the nurse implement? A. Encourage additional oral intake of juices and water. B. Provide additional coffee on the client's breakfast tray. C. Exchange the client's grape juice for cranberry juice. D. Bring the client additional fruit at mid-morning.

Answer: Encourage additional oral intake of juices and water. Dark amber urine is characteristic of fluid volume deficit, and the client should be encouraged to increase fluid intake (D). Caffeine, however, is a diuretic (A), and may worsen the fluid volume deficit. Any type of juice will be beneficial (B), since the client is not dysuric, a sign of an urinary tract infection. The client needs to restore fluid volume more than solid foods (C).

The nurse is assessing a client who is experiencing anaphylaxis from an insect sting. Which prescription should the nurse prepare to administer this client? A. Epinephrine. B. Dopamine. C. Diphenhydramine. D. Ephedrine.

Answer: Epinephrine Epinephrine (C) is an adrenergic agent that stimulate beta receptors to increase cardiac automaticity in cardiac arrest and relax bronchospasms in anaphylaxis. Dopamine (A) is a vasopressor used to treat clients with shock. Ephedrine (B) causes peripheral vasoconstriction and is used in the treatment of nasal congestion. Diphenhydramine (D) is an antihistamine decongestant used in the treatment of mild allergic reactions and motion sickness.

A client with sickle cell anemia is admitted with severe abdominal pain and the diagnosis is sickle cell crisis. What is the most important nursing action to implement? A. Evaluate the effectiveness of narcotic analgesics. B. Limit the client's intake of oral fluids and food. C. Teach the client about prevention of crises. D. Encourage the client to ambulate as tolerated.

Answer: Evaluate the effectiveness of narcotic analgesics. Pain management is the priority for a client during sickle cell crisis. Continuous narcotic analgesics are the mainstay of pain control, which should be evaluated (B) frequently to determine if the client's pain is adequately controlled. (A, C, and D) are not indicated at this time.

A client is receiving substitution therapy during withdrawal from benzodiazepines. Which expected outcome statement has the highest priority when planning nursing care? A. Excessive CNS stimulation will be reduced. B. Co-dependent behaviors will be decreased. C. Client's level of consciousness will increase. D. Client will not demonstrate cross-addiction.

Answer: Excessive CNS stimulation will be reduced. Substitution therapy with another CNS depressant is intended to decrease the excessive CNS stimulation that can occur during benzodiazepine withdrawal (A). (B, C, and D) are all appropriate outcome statements for the client described, but do not have the priority of (A).

A Spanish-speaking 5-year-old child starts kindergarten in an English-speaking school. The child cries most of the time, appears helpless and unable to function in the new situation. After assessing the child, how should the school nurse document the situation? A. Experiencing culture shock. B. Refuses to participate in school activities. C. Lacks the maturity needed in school. D. Going through minority group discrimination.

Answer: Experiencing culture shock. An inability to function may apply to persons of all ages undergoing transitions, such as moving to a new country and adjusting to a subculture within a larger culture that is unfamiliar. Culture shock (A) describes feelings of discomfort and disorientation when adapting to new cultural settings. Language barriers inhibit effective communication, so a child who is unable to communicate in the spoken language in the school environment may lack the skills necessary to participate, and is not refusing to participate (C). The child may be adequately mature (B), accepted by peers (D) within the environment, but continues to not join in because of the impact of culture shock.

The therapeutic effect of insulin in treating Type 1 diabetes mellitus is based on which physiologic action? A. Stimulates function of beta cells in the pancreas. B. Increases intracellular receptor site sensitivity. C. Facilitates transport of glucose into the cells D. Delays carbohydrate digestion and absorption.

Answer: Facilitates transport of glucose into the cells. Glucose moves across the cell membrane by using an active transport mechanism. Insulin acts as the carrier of glucose and is the only hormone that decreases blood glucose levels by facilitating transport of glucose into the cells (A). (B and C) describe actions of oral hypoglycemic agents, such as metformin (Glucophage), a biguanide oral antidiabetic drug. (D) describes the action of alpha-glucosidase inhibitors, such as acarbose (Precose) and miglitol (Glyset)

The nurse is caring for a client who was admitted for alcohol detoxification 2 days ago. Which finding is most critical for the nurse to report to the healthcare provider? A. Low-grade fever, diaphoresis, hypertension, and tachycardia. B. Global confusion and inability to recognize family members. C. Agitation, vomiting, and visual and auditory hallucinations. D. Restlessness, anxiety, and difficulty sleeping.

Answer: Global confusion and inability to recognize family members. Delirium tremens (DT) or alcohol withdrawal delirium usually peaks 2 to 3 days (48 to 72 hours) after cessation or reduction of intake (although it can occur later) and lasts 2 to 3 days. The risk of DT carries a 2% to 5% mortality rate, so this critical syndrome of alcohol withdrawal, manifested as global confusion and an inability to recognize family members (B), is life-threatening and requires emergency medical intervention. The early signs of withdrawal (A) develop within a few hours after cessation or reduction of alcohol (ethanol) intake; the signs peak after 24 to 48 hours (C and D) and then rapidly and dramatically disappear, unless the withdrawal progresses to alcohol withdrawal delirium.

A 12-month-old boy is admitted with a respiratory infection and possible pneumonia. He is placed in a mist tent with oxygen. Which nursing intervention has the greatest priority for this infant? A. Have a bulb syringe readily available to remove secretions. B. Give small, frequent feedings of fluids. C. Accurately chart observations regarding breath sounds. D. Encourage older siblings to visit.

Answer: Have a bulb syringe readily available to remove secretions A patent airway has the highest priority. Humidification will liquefy the nasal secretions thereby increasing the amount of secretions and making (C) the highest priority. (A) maintains hydration and prevent tiring, but an open airway has a higher priority! (B) is important for evaluation of therapy. When asked "priority" questions, REMEMBER MASLOW! Physical needs usually have a higher priority than psychosocial needs (D) and an open airway is the highest physiological need!

The nurse is giving discharge instructions to a client with chronic prostatitis. What instruction should the nurse provide the client to reduce the risk of spreading the infection to other areas of the client's urinary tract? A. Avoid consuming alcohol and caffeinated beverages. B. Wear a condom when having sexual intercourse. C. Have intercourse or masturbate at least twice a week. D. Empty the bladder completely with each voiding.

Answer: Have intercourse or masturbate at least twice a week. The prostate is not easily penetrated by antibiotics and can serve as a reservoir for microorganisms, which can infect other areas of the genitourinary tract. Draining the prostate regularly through intercourse or masturbation (D) decreases the number of microorganisms present and reduces the risk for further infection from stored contaminated fluids. (A, B, and C) do not reduce the risk of spreading the infection internally.

A middle-aged male client with diabetes continues to eat an abundance of foods that are high in sugar and fat. According to the Health Belief Model, which event is most likely to increase the client's willingness to become compliant with the prescribed diet? A. His wife expresses a sincere willingness to prepare meals that are within his prescribed diet. B. He comments on the community service announcements about preventing complications associated with diabetes. C. He is provided with the most current information about the dangers of untreated diabetes. D. He visits his diabetic brother who just had surgery to amputate an infected foot.

Answer: He visits his diabetic brother who just had surgery to amputate an infected foot. The loss of a limb by a family member (D) will be the strongest event or "cue to action" and is most likely to increase the perceived seriousness of the disease. (A, B, and C) may influence his behavior but do not have the personal impact of (D).

Which action should the nurse implement during the termination phase of the nurse-client relationship? A. Confront changes not completed. B. Explore the client's past in depth. C. Help summarize accomplishments. D. Identify new problem areas.

Answer: Help summarize accomplishments. By noting the client's accomplishments (D), the client's progress and self-confidence can be summarized. The working phase focuses on identifying new problem areas (A) and confronting necessary changes (B). The orientation phase includes an in-depth assessment of the client, including past history (C).

An 81-year-old male client has emphysema. He lives at home with his cat and manages self-care with no difficulty. When making a home visit, the nurse notices that his tongue is somewhat cracked and his eyeballs are sunken into his head. What nursing intervention is indicated? A. Encourage the client to slightly increase his use of oxygen at night and to always use humidified oxygen. B. Help the client to determine ways to increase his fluid intake. C. Obtain an appointment for the client to see an ear, nose, and throat specialist. D. Schedule an appointment with an allergist to determine if the client is allergic to the cat.

Answer: Help the client to determine ways to increase his fluid intake. The nurse should suggest creative methods to increase the intake of fluids (A), such as having disposable fruit juices readily available. Clients with COPD should have at least three liters of fluids a day. These clients often reduce fluid intake because of shortness of breath. (B) is not indicated. These symptoms are not indicative of an allergy (C). Many elderly depend on their pets for socialization and self-esteem. Humidified oxygen will not relieve these symptoms and increased oxygen levels will stifle the COPD client's trigger to breathe (D).

A 16-year-old female client returns to the clinic because she is pregnant for the third time by a new boyfriend. Which vaccine should the nurse plan to administer? A. Pneumococcal. B. Hepatitis B. C. Human papillomavirus. D. Measles-mumps-rubella.

Answer: Hepatitis B Multiple sexual contacts are associated with the risk for hepatitis B, so (B) has the highest priority for this client. The MMR vaccine (A), which contains attenuated live viruses that are teratogenic, is not recommended during pregnancy. The safety of the human papillomavirus (C) during pregnancy has not been determined and should not be given or completed during pregnancy. The pneumococcal vaccine (D) is not indicated at this time.

The nurse is assessing a middle-aged male client for risk factors related to chronic illness. Which finding should the nurse assess further? A. Thinning hair and dry scalp. B. Increase in muscle tone but decreased muscle strength. C. Increase in abdominal fat deposits. D. Increase in appetite and taste-bud acuity.

Answer: Increase in abdominal fat deposits. An increase in the abdominal girth (D) may be indicative of the onset of metabolic syndrome, which places the client at risk for cardiac disease and requires further assessment. During middle adulthood, common findings include thinning hair, dry skin and scalp (A), changes in taste bud acuity (B), and muscle size and strength (C), which are consistent with normal system functioning during aging.

The nurse is caring for a client who returns to the unit following a colonoscopy. Which finding should the nurse report to the healthcare provider immediately? A. Increased abdominal pain with rebound tenderness. B. Complaint of feeling weak with watery diarrheal stools. C. Large amounts of expelled flatus with mucus. D. Tympanic abdomen and hyperactive bowel sounds.

Answer: Increased abdominal pain with rebound tenderness. Positive rebound tenderness (C) may be an indication of peritonitis or perforation and needs follow-up immediately. Clients typically experience a large amount of flatus (A) and may have mucus from bowel irritation from the procedure. A tympanic abdomen on percussion and hyperactive bowel sounds are typical post procedure findings (B). Weakness and watery stools are a result from the preparation and are common symptoms experienced after a colonoscopy (D).

An older client is admitted to a psychiatric hospital with the diagnosis, "Major depression, single episode." Which laboratory value is most important for the nurse to report to the healthcare provider immediately? A. Elevated serum calcium level. B. Positive rapid plasma reagin (RPR). C. Increased serum creatinine level. D. Increased thyroid stimulating hormone (TSH).

Answer: Increased thyroid stimulating hormone (TSH). The healthcare provider should be notified of (D) immediately. An increased TSH suggests a low thyroxine level because the TSH is trying to stimulate thyroxine production, and hypothyroidism symptoms mimic those of depression. (A) often increases with aging. (B) is indicative of syphilis and should be reported, but does not have the priority of (C). (D) has implications for other illnesses, such as non-Hodgkin's lymphoma or hyperparathyroidism.

Three days following surgery, a male client observes his colostomy for the first time. He becomes quite upset and tells the nurse that it is much bigger than he expected. What is the best response by the nurse? A. Offer to contact a member of the local ostomy support group to help him with his concerns. B. Instruct the client that the stoma will become smaller when the initial swelling diminishes. C. Encourage the client to handle the stoma equipment to gain confidence with the procedure. D. Reassure the client that he will become accustomed to the stoma appearance in time.

Answer: Instruct the client that the stoma will become smaller when the initial swelling diminishes. Postoperative swelling causes enlargement of the stoma. The nurse can teach the client that the stoma will become smaller when the swelling is diminished (B). This will help reduce the client's anxiety and promote acceptance of the colostomy. (A) does not provide helpful teaching or support. (C) is a useful action, and may be taken after the nurse provides pertinent teaching. The client is not yet demonstrating readiness to learn colostomy care (D).

When assessing a client's emotional intelligence, which client capabilities should the nurse focus the interview on with a client diagnosed with a chronic mental illness? A. Logical mathematics and linguistic abilities. B. Bodily kinesthetic and spatial abilities. C. Linguistic and musical abilities. D. Interpersonal and intrapersonal skills.

Answer: Interpersonal and intrapersonal skills. Interpersonal and intrapersonal intelligence form one's personal intelligence or "emotional quotient," so the nurse should focus inquiries on social skills (B). (A and D) assesses cognitive and mental status. (C) determines neurophysical interpretation of one's body within the environment, but does not assess emotional intelligence.

The practical nurse (PN) is working with the registered nurse (RN) to provide care for several clients. Which task should the RN, rather than the PN, perform? A. Apply a neck brace prior to ambulating a client the first day after a cervical laminectomy. B. Assist a healthcare provider performing a joint fluid aspiration of a client's knee. C. Irrigate and pack the stage IV coccygeal pressure ulcer for a client with paraplegia. D. Remove the staples from a client's incision one week after hip arthroplasty.

Answer: Irrigate and pack the stage IV coccygeal pressure ulcer for a client with paraplegia. Care of a stage IV pressure ulcer (C) is a complex, sterile procedure that requires assessment of the wound, and evaluation of the effectiveness of the treatment plan, and should be performed by the RN. (A, B, and D) are procedures that require the skill and expertise of a licensed nurse, but are within the scope of practice for a PN (C).

The nurse is caring for a client with syndrome of inappropriate antidiuretic hormone (SIADH), which is manifested by which symptoms? A. Polydipsia, polyuria. B. Hypernatremia, tachypnea. C. Dependent edema, fever. D. Loss of thirst, weight gain

Answer: Loss of thirst, weight gain SIADH occurs when the posterior pituitary gland releases too much ADH, causing water retention, a urine output of less than 20 ml/hour, and dilutional hyponatremia. Other indications of SIADH are loss of thirst, weight gain (A), irritability, muscle weakness, and decreased level of consciousness. (B) is not associated with SIADH. (C) is a finding associated with diabetes insipidus (a water metabolism problem caused by an ADH deficiency), not SIADH. The increase in plasma volume causes an increase in the glomerular filtration rate that inhibits the release of rennin and aldosterone, which results in an increased sodium loss in urine, leading to greater hyponatremia, not (D).

A client who was prescribed atorvastatin (Lipitor) one month ago calls the triage nurse at the clinic complaining of muscle pain and weakness in his legs. Which statement reflects the correct drug-specific teaching the nurse should provide to this client? A. Make an appointment to see the healthcare provider, because muscle pain may be an indication of a serious side effect. B. Be sure to consume a low-cholesterol diet while taking the drug to enhance the effectiveness of the drug. C. Increase consumption of potassium-rich foods since low potassium levels can cause muscle spasms. D. Have serum electrolytes checked at the next scheduled appointment to assess hyponatremia, a cause of cramping..

Answer: Make an appointment to see the healthcare provider, because muscle pain may be an indication of a serious side effect. Myopathy, suggested by the leg pain and weakness, is a serious, and potentially life-threatening, complication of Lipitor, and should be evaluated immediately by the healthcare provider (C). Although electrolyte imbalances such as (A or B) can cause muscle spasms in some cases, this is not the likely cause of leg pain in the client receiving Lipitor, and evaluation by the healthcare provider should not be delayed for any reason. A low-cholesterol diet is recommended for those taking Lipitor since the drug is used to lower total cholesterol (D), but diet is not related to the leg pain symptom.`

A client with a 16-year history of diabetes mellitus is having renal function tests because of recent fatigue, weakness, elevated blood urea nitrogen, and serum creatinine levels. Which finding should the nurse conclude as an early symptom of renal insufficiency? A. Confusion. B. Stomatitis. C. Dyspnea. D. Nocturia.

Answer: Nocturia As the glomerular filtration rate decreases in early renal insufficiency, metabolic waste products, including urea, creatinine, and other substances, such phenols, hormones, electrolytes, accumulate in the blood. In the early stage of renal insufficiency, polyuria results from the inability of the kidneys to concentrate urine and contribute to nocturia (B). (A, C, and D) are more common in the later stages of renal failure.

A 4-month-old breastfeeding infant is at the 10th percentile for weight and the 75th percentile for height. How should the nurse interpret this finding? A. Inadequate milk supply in mother. B. Milk allergy. C. Normal growth curve of a breast-fed infant. D. Failure to thrive.

Answer: Normal growth curve of a breast-fed infant. When plotting weights and heights on a standard growth chart used for both breast-fed and formula-fed infants, the breast-fed infant grows more rapidly during the first 2 months of life, and then growth slows from 3 to 12 months. A breast-fed infant is leaner and has less body fat than a formula-fed infant. Normal patterns of infants who are breast fed (D) differ from those who are formula fed. (A) is an incorrect interpretation of the data. This finding is not consistent with failure to thrive (B) or an inadequate milk supply (C)

The nurse is preparing an adult client for an upper gastrointestinal (UGI) series. Which information should the nurse include in the teaching plan? A. The xray procedure may last for several hours. B. Enemas are given to empty the bowel after the procedure. C. Nothing by mouth is allowed for 6 to 8 hours before the study. D. A nasogastric tube (NGT) is inserted to instill the barium.

Answer: Nothing by mouth is allowed for 6 to 8 hours before the study. The client should be NPO for at least 6 hours before the UGI (D). (A) is not typical for this procedure. A NGT is not needed to instill the barium (B) unless the client is unable to swallow. A laxative, not enemas (C), is given after the procedure to help expel the barium.

In providing care for a terminally ill resident of a long-term care facility, the nurse determines that the resident is exhibiting signs of impending death and has a "do not resuscitate" or DNR status. What intervention should the nurse implement first? A. Report the client's acuity level to the nursing supervisor. B. Request hospice care for the client. C. Notify family members of the client's condition. D. Inform the chaplain that the client's death is imminent.

Answer: Notify family members of the client's condition. The nurse's first priority is to notify the family of the resident's impending death (C). The family may request that hospice care is initiated (A). Reporting the client's acuity level (B) does not have the priority of informing the family of the client's condition. Once the family is contacted, the nurse can also contact the chaplain (D).

An elderly resident of a long-term care facility is no longer able to perform self-care and is becoming progressively weaker. The resident previously requested that no resuscitative efforts be performed, and the family requests hospice care. What action should the nurse implement first? A. Prepare the family for the client's impending death. B. Notify the healthcare provider of the family's request. C. Reaffirm the client's desire for no resuscitative efforts. D. Transfer the client to a hospice inpatient facility.

Answer: Notify the healthcare provider of the family's request. The nurse should first communicate with the healthcare provider (D). Hospice care is provided for clients with a limited life expectancy, which must be identified by the healthcare provider. (A) is not necessary at this time. Once the healthcare provider supports the transfer to hospice care, the nurse can collaborate with the hospice staff and healthcare provider to determine when (B and C) should be implemented.

A client with osteoarthritis receives a new prescription for celecoxib (Celebrex) orally for symptom management. The nurse notes the client is allergic to sulfa. Which action is most important for the nurse to implement prior to administering the first dose? A. Review the client's hemoglobin results. B. Record the client's vital signs. C. Notify the healthcare provider. D. Inquire about the reaction to sulfa.

Answer: Notify the healthcare provider. Celebrex contains a sulfur molecule, which can lead to an allergic reaction in individuals who are sensitive to sulfonamides, so the healthcare provider should be notified of the client's allergies (B). Although (A, C, and D) are important assessments, it is most important to notify the healthcare provider for an alternate prescription.

A client with Paget's disease is started on calcitonin (Calcimar) 500 mcg subcutaneously daily. During the initial treatment, what is the priority nursing action? A. Assess the injection site for inflammation. B. Observe the client for signs of hypersensitivity. C. Evaluate the client's level of pain. D. Monitor the client's alkaline phosphatase levels.

Answer: Observe the client for signs of hypersensitivity. The nurse's highest priority is to observe for signs of hypersensitivity, such as skin rash, hives, or anaphylaxis (D). Calcitonin is given to a client with Paget's disease to lower serum calcium levels. However, hypersensitivity can cause life-threatening anaphylaxis. Calcitonin may cause local site inflammation, so (A) is important, but does not have the priority of (D). A reduction in (B and C) are indicators that the calcitonin is having the desired effect.

Which action should the nurse implement on the scheduled day of surgery for a client with type 1 diabetes mellitus (DM)? A. Give an insulin dose using parameters of a sliding scale. B. Withhold insulin while the client is NPO. C. Administer an oral anti-diabetic agent. D. Obtain a prescription for an adjusted dose of insulin.

Answer: Obtain a prescription for an adjusted dose of insulin. Stressors, such as surgery, increase serum glucose levels. A client with type 1 DM who is NPO for scheduled surgery should receive a prescribed adjusted dose of insulin (A). (B, C, and D) are not indicated.

Which menu selection by a child with celiac disease indicates to the nurse that the child understands necessary dietary considerations? A. Graham crackers and fruit juice. B. Oatmeal-raisin cookies and milk. C. Peanut butter and banana sandwich. D. Oven-baked potato chips and cola.

Answer: Oven-baked potato chips and cola. Celiac disease causes an intolerance to the protein gluten found in oats, rye, wheat, and barley. The child should avoid any products containing these ingredients to avoid symptoms such as diarrhea. (A) is the selection which avoids all of these ingredients. (B, C, and D) contain gluten in one form or another.

The nurse receives a lab report stating a child with asthma has a theophylline level of 15 mcg/dl. What action will the nurse take? A. Pass the information on in the report. B. Hold the next dose of theophylline. C. Notify the healthcare provider because the value is high. D. Repeat the lab study because the value is too high.

Answer: Pass the information on in the report. The therapeutic level of theophylline is 10 to 20 mcg/dl, so the child's level is within the therapeutic range. This information evaluates the prescribed therapy and should be communicated in the nurse's report (A). (B, C, and D) would be inappropriate actions in view of the laboratory finding.

The nurse is teaching a 12-year-old male adolescent and his family about taking injections of growth hormone for idiopathic hypopituitarism. Which adverse symptoms, commonly associated with growth hormone therapy, should the nurse plan to describe to the child and his family? A. Lethargy and fatigue. B. Polyuria and polydipsia. C. Facial bone structure changes. D. Increased facial hair.

Answer: Polyuria and polydipsia. Signs and symptoms of diabetes or hyperglycemia (A) need to be reported. Those receiving growth hormone should be monitored to detect elevated blood sugars and glucose intolerance. (B) is associated with any number of heath alterations, but is not associated with the growth hormone therapy. (C and D) are normal changes that occur with 12-year-old males.

The nurse is assessing a child's skin turgor and grasps the skin on the abdomen between the thumb and index finger, pulls it taut, and quickly releases it. The tissue remains suspended and tented for a few seconds, then slowly falls back on the abdomen. How should the nurse document this finding? A. Assessment inconclusive. B. Poor skin turgor. C. Adequate hydration. D. Normal skin elasticity

Answer: Poor skin turgor Tissue turgor refers to the amount of elasticity in the skin and is one of the best estimates of adequate hydration and nutrition. Elastic tissue immediately resumes its normal position without residual marks or creases. In a child with poor turgor (B), the skin remains tented or suspended for a few seconds before returning to a normal position. (A, C and D) are inaccurate.

The nurse is developing a plan of care for a newborn with a colostomy due to anal agenesis, and the infant has had three loose stools since surgery yesterday. Which nursing diagnosis has the highest priority? A. Pain related to postoperative condition. B. Potential for fluid volume deficit. C. Alteration in bowel elimination. D. Anxiety of parents related to newborn's condition.

Answer: Potential for fluid volume deficit. All stated nursing diagnoses are appropriate for a postoperative colostomy client. However, fluid balance is the priority concern (A) for any newborn infant. Though three loose stools in 24-hours is not significant, depending on the amount of fluid lost with each stool, potential for fluid volume deficit is always a concern for a postoperative infant. Newborns are extremely vulnerable to fluid imbalances due to immature body systems and a larger percentage of their body weight consisting of fluid. (B, C, and D) do not have the priority of (A).

A 3-year-old boy is brought to the emergency room because of a possible diazepam (Valium) overdose. He is lethargic and confused, and his vital signs are: pulse rate 100 beats/minute, respiratory rate 20 breaths/minute, and blood pressure 70/30. Which nursing intervention has the highest priority? A. Insert an orogastric tube for gastric lavage. B. Prepare a set-up for an endotracheal intubation. C.Draw blood for stat chemistries and blood gases. D. Insert a Foley catheter to monitor renal functioning.

Answer: Prepare a set-up for an endotracheal intubation. Diazepam causes respiratory depression, so preparation for intubation (B) to protect the airway is the priority intervention at this time. (A) may be necessary, but the child is lethargic and confused, with a lowered respiratory rate, so (B) takes priority. (C and D) are interventions that should be implemented, but they are both secondary to ensuring an open airway.

Which action should the nurse implement first for a client experiencing alcohol withdrawal? A. Apply vest or extremity restraints. B. Provide a diet high in protein and calories. C. Give an alpha-adrenergic blocker. D. Prepare the environment to prevent self-injury.

Answer: Prepare the environment to prevent self-injury. Self-destructive or violent behavior provides a potentially immediate and life-threatening risk to the client and others, so a safe environment should be provided (D) by removing any potential objects that could inflict self-injury. Secondary prevention strategies (frequent orientation to surroundings, restraints to prevent self-injury (A), and the administration of antianxiety agents or alpha-adrenergic blockers (B) for hallucinations, delusions, confusion, and agitation) should then be implemented. Once the client is stabilized, nutritional issues (C) should be addressed.

An infant is born with a ventricular septal defect (VSD) and surgery is planned to correct the defect. The nurse recognizes that surgical correction is designed to achieve which outcome? A. Reduce peripheral tissue hypoxia and nailbed clubbing. B. Prevent the return of oxygenated blood to the lungs. C. Increase the flow of unoxygenated blood to the lungs. D. Stop the flow of unoxygenated blood into systemic circulation.

Answer: Prevent the return of oxygenated blood to the lungs. Closure of VSDs stops oxygenated blood from being shunted from the left ventricle to the right ventricle (C). VSDs are acyanotic defects, which means that no unoxygenated blood enters the systemic circulation (A and B). (D) is common with Tetrology of Fallot, which is a cyanotic defect.

The nurse administers a booster dose of DTaP (diphtheria, tetanus, and pertussis) vaccine to an infant. Which level of prevention is the nurse implementing? A. Primary prevention. B. Secondary prevention. C. Primary nursing. D. Tertiary prevention.

Answer: Primary prevention. Primary prevention (A) involves activities that focus on reducing the potential for illness before it occurs, such as immunizations. Tertiary prevention (B) minimizes the consequences of a disorder or illness through aggressive management or rehabilitation. Secondary prevention (C) includes early diagnosis and implementing interventions aimed at a cure or reducing the progress of a disease. Primary nursing (D) describes a method of nursing management and nursing care assignments, not a healthcare strategy.

In developing a teaching plan for a 5-year-old child with diabetes, which component of diabetic management should the nurse plan for the child to manage first? A. Drawing up the correct insulin dose. B. Process of glucose testing. C. Administering insulin injections. D. Food planning and selection.

Answer: Process of glucose testing. Developmentally, a 5-year-old has the cognitive and psychomotor skills to use a glucometer (C) and to read the number (it is especially helpful if the nurse presents this activity as a game). (A, B, and D) require more advanced cognitive and psychomotor skills and have greater potential for errors.

During a health fair, a 72-year-old male client tells the nurse that he is experiencing shortness of breath. Auscultation reveals crackles and wheezing in both lungs. Suspecting that the client might have chronic bronchitis, which classic symptom would the nurse expect this client to have? A. Productive cough with grayish-white sputum. B. An increased chest diameter. C. Clubbing of the fingers. D. Racing pulse with exertion.

Answer: Productive cough with grayish-white sputum. Chronic bronchitis, one of the diseases comprising the diagnosis of COPD, is characterized by a productive cough with grayish-white sputum (A), which usually occurs in the morning and is often ignored by smokers. (D) is not related to chronic bronchitis; however, it is indicative of other problems such as ventricular tachycardia and should be explored. (B and C) are symptoms of emphysema and are not consistent with the other symptoms. (B) is usually referred to as a "barrel chest."

What is the expected outcome of esomeprazole (Nexium) when prescribed for a client with gastroesophageal reflux disease (GERD)? A. Improved esophageal peristalsis. B. Increased gastric emptying. C. Neutralization of gastric secretions. D. Promotion of rapid tissue healing.

Answer: Promotion of rapid tissue healing Proton pump inhibitors, such as esomeprazole (Nexium), act to inhibit gastric acid secretion and promote rapid healing of esophageal tissue (A). (B and C) are actions of prokinetic drugs, and (D) is the action of over-the-counter antacids.

A client with heart disease is on a continuous telemetry monitor and has developed sinus bradycardia. In determining the possible cause of the bradycardia, the nurse assesses the client's medication record. Which medication is most likely the cause of the bradycardia? A. Propanolol (Inderal). B. Furosemide (Lasix). C. Dobutamine (Dobutrex). D. Captopril (Capoten).

Answer: Propanolol (Inderal). Inderal (A) is a beta adrenergic blocking agent, which causes decreased heart rate and decreased contractility. Neither (D), an ACE inhibitor, nor (B), a loop diuretic, causes bradycardia. (C) is a sympathomimetic, direct acting cardiac stimulant, which would increase the heart rate.

A client with type II diabetes arrives at the clinic with a blood glucose of 50 mg/dL. The nurse provides the client with 6 ounces of orange juice. In 15 minutes the client's capillary glucose is 74 mg/dL. What action should the nurse take? A. Provide cheese and bread to eat. B. Collect a glycosylated hemoglobin specimen. C. Obtain a specimen for serum glucose level. D. Administer insulin per sliding scale.

Answer: Provide cheese and bread to eat. Once blood glucose is greater than 70 mg/dl, the client should eat a regularly scheduled meal or a snack to prevent hypoglycemia from recurring (C). Blood glucose has just been checked and a serum level is not indicated at this time (A). The blood glucose does not indicate a need for insulin (B) which may further exacerbate a hypoglycemic response. A glycosylated hemoglobin (hemoglobin A1C) level is not indicated at this time (D).

A 14-year-old is brought to the emergency room after a biking accident. How should the nurse interact with the adolescent? A. Provide clear explanations while encouraging questions. B. Limit the number of choices to be made by the adolescent. C. Have the parents remain with the adolescent at all times. D. Furnish rewards for cooperation during procedures.

Answer: Provide clear explanations while encouraging questions. Adolescents are capable of abstract thinking and understand explanations, so the opportunity to ask questions (C) should be provided. Giving rewards (A), such as stickers for cooperation with treatments or procedures are best used with a younger child. An adolescent's modesty should be respected, so the presence of the parents (B) at the bedside should be a choice made by the adolescent. An adolescent's ability to think abstractly engages problem solving, so the 14-year-old should be allowed to make decisions about care, not (D).

When caring for a child who has pertussis that is in the paroxysmal stage, which intervention should the nurse implement to support the child's nutritional needs? A. Maintain a liquid diet. B.Provide small, frequent meals. C. Offer the child a regular diet. D. Increase protein intake.

Answer: Provide small, frequent meals. The paroxysmal stage of pertussis is characterized by coughing with vomiting. Frequent small meals (A) are vomited less often than larger meals. (B, C, and D) are not useful interventions at this stage of the child's pertussis.

The healthcare provider prescribes aluminum and magnesium hydroxide (Maalox), 1 tablet PO PRN, for a client with chronic kidney disease (CKD) who is complaining of indigestion. What intervention should the nurse implement? A. Question the healthcare provider's prescription. B. Instruct the client to swallow the tablet whole. C. Administer 30 minutes before eating. D. Evaluate the effectiveness 1 hour after administration.

Answer: Question the healthcare provider's prescription. Magnesium agents are not usually used for clients with CKD due to the risk of hypermagnesemia, so this prescription should be questioned by the nurse (D). (A, B, and C) are not recommended nursing actions for the administration of aluminum and magnesium hydroxide (Maalox).

A client is taking sulfisoxazole (Gantrisin) for a urinary tract infection (UTI) and complains of nausea and gastric upset since starting the medication. Which additional adverse reaction should the nurse instruct the client to report? A. Diarrhea. B. Muscle cramping. C. Rash. D. Hematuria.

Answer: Rash Side effects of sulfisoxazole (Gantrisin), a sulfonamide antibiotic, include possible allergic response, manifested by skin rash (A) and itching, which can progress to Stevens-Johnson syndrome - erythema multiforme, a severe hypersensitivity reaction. Other gastrointestinal disturbances, such as diarrhea (B), crystalluria and photosensitivity are other side effects that commonly occur with "sulfa" agents but do not need the discontinuation of the prescription. Hematuria (C) is associated with a UTI. Muscle cramping (D) is mostly likely related to an electrolyte disturbance.

A client is receiving ampicillin sodium (Omnipen) for a sinus infection. The nurse should instruct the client to notify the healthcare provider immediately if which symptom occurs? A. Rash. B. Headache. C. Dizziness. D. Nausea.

Answer: Rash Rash (A) is the most common adverse effect of all penicillins, indicating an allergy to the medication which could result in anaphylactic shock, a medical emergency. (B, C, and D) are common side effects of penicillins that should subside after the body adjusts to the medication. These would not require immediate medical care unless the symptoms persist beyond the first few days or become extremely severe.

A client with heart failure is prescribed spironolactone (Aldactone). Which information is most important for the nurse to provide to the client about diet modifications? A. Refrain for eating foods high in potassium. B. Do not add salt to foods during preparation. C. Increase intake of milk and milk products. D. Restrict fluid intake to 1000 ml per day.

Answer: Refrain for eating foods high in potassium. Spironolactone (Aldactone), an aldosterone antagonist, is a potassium-sparing diuretic, so a diet high in potassium should be avoided (B), including potassium salt substitutes, which can lead to hyperkalemia. Although (A) is a common diet modification in heart failure, the risk of hyperkalemia is more important with Aldactone. Restriction of fluids (C) or increasing milk and milk products (D) are not indicated with this prescription.

The charge nurse assigns a nursing procedure to a new staff nurse who has not previously performed the procedure. What action is most important for the new staff nurse to take? A.Refuse to perform the task that is beyond the nurse's experience. B. Review the steps in the procedure manual. C. Ask another nurse to assist while implementing the procedure. D. Follow the agency's policy and procedure.

Answer: Refuse to perform the task that is beyond the nurse's experience. According to states' nurse practice acts, it is the responsibility of the nurse to function within the scope of competency (D), and in this case safe nursing practice constitutes refusal to perform the procedure because of a lack of experience. Although state mandates, agency policies, and continued education and experience identify tasks that are within the scope of nursing practice, nurses should first refuse to perform tasks that are beyond their proficiency, and then pursue opportunities to enhance their competency (A, B, and C).

A client receives a new prescription for an angiotensin-converting enzyme (ACE) inhibitor. What client history contraindicates its use? A. Asthma. B. Heart failure. C. Coronary artery disease. D. Renal artery stenosis.

Answer: Renal artery stenosis Angiotensin-converting enzyme (ACE) inhibitors can cause severe renal insufficiency in clients with bilateral renal artery stenosis (C) or stenosis in the artery to a single remaining kidney. ACE inhibitors should not be used during the second and third trimesters of pregnancy and should be used with caution in clients who are taking potassium-sparing diuretics or who have hyperkalemia. The use of ACE inhibitors is not contraindicated for clients with asthma (A). ACE inhibitors are indicated for clients with heart failure (B). Ramipril, an ACE inhibitor, is approved for use in high risk clients for future cardiac events, including those with a history of coronary artery disease (D).

After checking the urinary drainage system for kinks in the tubing, the nurse determines that a client who has returned from the post-anesthesia care has a dark, concentrated urinary output of 54 ml for the last 2 hours. What priority nursing action should be implemented? A. Increase the IV flow rate for 15 minutes. B. Irrigate the indwelling urinary catheter. C. Report the findings to the surgeon. D. Apply manual pressure to the bladder.

Answer: Report the findings to the surgeon An adult who weighs 132 pounds (60 kg) should produce about 60 ml of urine hourly (1 ml/kg/hour). Dark, concentrated, and low volume of urine output should be reported to the surgeon. Although other actions (B, C, and D) may be indicated, the assessment findings should be reported to the healthcare provider.

The nurse assigns a UAP to obtain vital signs from a very anxious client. What instructions should the nurse give the UAP? A. Reassure the client that the vital signs are normal. B. Notify the medication nurse immediately if the pulse or blood pressure is low. C. Report the results of the vital signs to the nurse. D. Remain calm with the client and record abnormal results in the chart.

Answer: Report the results of the vital signs to the nurse. Interpretation of vital signs is the responsibility of the nurse, so the UAP should report vital sign measurements to the nurse (C). (A, B, and D) require the UAP to interpret the vital signs, which is beyond the scope of the UAP's authority.

A nurse receives an emphatic complaint from a client in a semi-private room that the night shift nurse did not come into the room the entire night. What action should the nurse implement first? A. Verify occurrence with client's roommate while he's ambulating in the hall. B. Telephone the night shift nurse as soon as possible to ask about the situation. C. Discuss the situation with staff to determine if this client has a history of complaining. D. Review the night shift nurse's documentation with the charge nurse.

Answer: Review the night shift nurse's documentation with the charge nurse. The client's concern needs to be assessed immediately. This can best be accomplished by reviewing the documentation with administration, i.e., the charge nurse, to determine the client's needs and the night nurse's response. The night shift nurse may need to be contacted at some point (A) but reviewing the documentation should occur first. The nurse should not discuss the situation with (C or D).

What is the most effective time management strategy for a nurse who needs to review 10 client records in 2 weeks? A. Delegate other nursing responsibilities to the team members. B. Designate 15 minutes a day to respond to each time-waster. C. Schedule specific times on a written calendar to review 2 charts per day. D. Review all records 2 days before the due date to focus on the deadline.

Answer: Schedule specific times on a written calendar to review 2 charts per day. Creating a disciplined approach by scheduling time periods for each issue is the most effective time-management strategy (C). Although (A and B) are options, the priority responsibility is to accomplish the goal within the designated time frame without imposing on others. (D) can create more stress that may hinder accomplishing the goal.

Which technique is the most important therapeutic tool a nurse should use to provide quality care to a psychiatric client? A. Context. B. Counter transference. C. Therapeutic self-disclosure. D. Self-analysis.

Answer: Self-analysis Self-analysis is a tool for the nurse to examine oneself, view one's responses in various mental and emotional moments, and provide a sense of how sensitive care should be provided relative to one's own needs, so (B) is a primary tool used by the nurse to establish therapeutic empathy and achieve authentic, open, and personal communication with a client. Although (A, C, and D) may occur in a nurse-client relationship, they may not contribute to establishing a therapeutic relationship.

A 3-year-old client with sickle cell anemia is admitted to the Emergency Department with abdominal pain. The nurse palpates an enlarged liver, an x-ray reveals an enlarged spleen, and a CBC reveals anemia. These findings indicate which type of crisis? A. Aplastic. B. Vaso-occlusive. C. Hyperhemolytic. D. Sequestration.

Answer: Sequestration. The findings support a sequestration crisis (B), where blood pools in the spleen, and is characterized by abdominal pain and anemia. (A and C) crises produce anemia but no abdominal pain or splenic enlargement. (D) crisis may produce abdominal pain, but no splenic enlargement or exacerbation of anemia.

A client with chronic renal disease is admitted to the hospital for evaluation prior to a surgical procedure. Which laboratory test indicates the client's protein status for the longest length of time? A. Prealbumin. B. Transferrin. C. Serum albumin. D. Urine urea nitrogen.

Answer: Serum albumin Serum albumin has a long half-life and is the best long-term indicator of the body's entry into a catabolic state following protein depletion from malnutrition or stress of chronic illness (C). While (A) is a good indicator of iron-binding capacity in a healthy adult, it is an unreliable measure in the client with a chronic illness. (B) has a short half-life, and is a sensitive indicator of recent catabolic changes, but it is not as effective as (C) in indicating long-term protein depletion. While (D) is a good indicator of a negative nitrogen balance, it is not as good an indicator of long-term protein catabolism as is (C).

A client is receiving methylprednisolone (Solu-Medrol) 40 mg IV daily. The nurse anticipates an increase in which laboratory value as the result of this medication? A. Serum potassium. B. Red blood cells. C. Serum glucose. D. Serum calcium.

Answer: Serum glucose Solu-Medrol is a corticosteroid with glucocorticoid and mineralocorticoid actions. These effects can lead to hyperglycemia (A), which is reflected as an increase in the serum glucose value. The client taking Solu-Medrol is at risk for hypocalcemia (B) and hypokalemia (D), which result in a decrease, not an increase, in the serum calcium and serum potassium levels. This medication does not adversely affect the RBC count (C).

The nurse is teaching a client about precautions for a new prescription for lovastatin (Mevacor). Which symptom should the nurse instruct the client to report to the healthcare provider immediately? A. Severe muscle pain. B. Visual disturbances. C. Terrible nightmares. D. Increased nocturia.

Answer: Severe muscle pain. A potential, serious side effect of statin therapy that is used to lower both LDL-C and triglyceride levels is rhabdomyolysis, which is manifested by severe muscle pain and aching (C). (A) is a side effect, but not life threatening. (B) is not related to statin therapy. Blurred vision (D) is a transient side effect that does not need immediate medical treatment.

The nurse calculates a 4 ml dose of prescribed digoxin a 9-month-old infant. What action should the nurse implement? A. Check heart rate and administer dose by letting the infant suck it through a nipple. B. Check heart rate and administer dose by placing it to the back and side of mouth. C. Suspect dosage error and do not give dose. D. Mix dose with juice to disguise its taste.

Answer: Suspect dosage error and do not give dose. Digoxin narrow margin of safety for an infant should not exceed 1 ml (50 mcg) in one dose. The nurse's calculation indicates a dosage error and should not be given (B). Digoxin is given without mixing with any other fluids or foods (A) because the infant may refuse to consume the total amount which results in an inaccurate drug dose. Although (C and D) are interventions for administration of oral digoxin to an infant, the dose is an error and should not be administered.

Which ego-defense mechanisms are exhibited by a phobic client who refuses to leave home? A. Fantasy. B. Denial. C. Intellectualization. D. Symbolization.

Answer: Symbolization Symbolization (D) allows external objects to carry the internal emotional feeling through some act such as refusing to leave a "safe" harbor. (A) is the unconscious failure to acknowledge an event, thought or feeling. (C) is pretending, usually of a more desirable situation. (B) is using reason to avoid emotional conflicts.

A 43-year-old female client is receiving thyroid replacement hormone following a thyroidectomy. What adverse effects associated with thyroid hormone toxicity should the nurse instruct the client to report promptly to the healthcare provider? A. Tachycardia and chest pain. B. Weight gain and increased appetite. C. Dry skin and intolerance to cold. D. Tinnitus and dizziness.

Answer: Tachycardia and chest pain. Thyroid replacement hormone increases the metabolic rate of all tissues, so common signs and symptoms of toxicity include tachycardia and chest pain (B). (A, C, and D) do not indicate a thyroid hormone toxicity.

A client is being treated for osteoporosis with alendronate (Fosamax), and the nurse has completed discharge teaching regarding medication administration. Which morning schedule would indicate to the nurse that the client teaching has been effective? A. Take medication with breakfast, then take a 30 minute morning walk. B. Take medication, rest in bed for 30 minutes, eat breakfast, go for morning walk. C. Take medication, go for a 30 minute morning walk, then eat breakfast. D. Go for a 30 minute morning walk, eat breakfast, then take medication.

Answer: Take medication, go for a 30 minute morning walk, then eat breakfast. Alendronate (Fosamax) is best absorbed when taken thirty minutes before eating in the morning. The client should also be advised to remain in an upright position for at least thirty minutes after taking the medication to reduce the risk of esophageal reflux and irritation. (A) is the best schedule to meet these needs. (B, C, and D) do not meet these criteria.

Which task is best for a nurse to delegate to an unlicensed assistive personnel (UAP)? A. Assess a client's skin to see if there is any skin breakdown. B. Take the 4th set of vital signs on a client receiving a blood transfusion. C. Teach a client how to use an incentive spirometer. D. Monitor a postoperative client and notify a nurse if the client needs anything.

Answer: Take the 4th set of vital signs on a client receiving a blood transfusion. Taking vital signs (B) on a client receiving blood is only data collection, which is within the practice role of a UAP. The registered nurse (RN) must interpret the findings. (A) is too vague. (C) is teaching, which is always the responsibility of the RN. (D) asks for an assessment, which is within the scope of practice for an RN, not the UAP.

A client with chronic pancreatitis receives a new prescription for pancrelipase (Pancrease). Which instruction is most important for the nurse to include in this client's teaching? A. Take the medication when consuming food. B. Stay away from products containing alcohol. C. Ingest 8 oz of grapefruit juice with the medication. D. Avoid prolonged exposure to direct sunlight.

Answer: Take the medication when consuming food. With the loss of exocrine function for a client with chronic pancreatitis, replacement of pancreatic enzymes using pancrelipase (Pancrease) becomes necessary. Diarrhea and steatorrhea (fatty stools) indicate insufficient pancreatic enzymes are present to digest dietary fats and other of nutrients, so pancrelipase, a fat-digesting enzyme, should be consumed with any type of food (D). (A, B, and C) are not related to the administration of Pancrease.

A mother expresses concern to the nurse about the behavior of her 15-year-old adolescent who is frequently finding fault and criticizing her. What information should the nurse provide? A. Teens create psychological distance from parents in order to separate from them. B. The family value system may need to be changed to meet the teen's changing needs. C. Parents should relinquish their relationship with their teen to the teen's peers. D. Conflicts in the parent-teen relationship are to be expected during adolescence.

Answer: Teens create psychological distance from parents in order to separate from them. Although a mutually respectful parent-adolescent relationship is important, an adolescent may use critical and fault-finding behavior as a mechanism to separate from the parent (B). Changing the family's value system to meet the teen's needs (A) does not provide consistency for an adolescent who is examining oneself. (C) does not provide guidance or boundary setting that is needed to foster judgment during adolescence. Although (D) may occur as an adolescent struggles for independence, healthy family dynamics foster the parent-teen relationship even though it may not seem as important to the teen as it was in earlier years.

A female client comes to an outpatient therapy appointment intoxicated. The spouse tells the nurse, "There wasn't anything I could do to stop her drinking this morning." What intervention should the nurse take at this time? A. Arrange for emergency admission to a detoxification unit. B. Tell the client that therapy cannot take place while she is intoxicated. C. Talk to the spouse about strategies to limit the client's drinking. D. Have the client admitted to the inpatient psychiatric unit.

Answer: Tell the client that therapy cannot take place while she is intoxicated. Therapy sessions are designed to confront the issues that the client with alcohol dependence may be experiencing. If the client presents inebriated, a therapeutic and confrontational meeting cannot occur (D) because the client's judgment is altered. (A and C) are not necessary at this time. (B) is ineffective.

Which statement best describes durable power of attorney for health care? A. The healthcare decisions made by another person designated by the client are not legally binding. B. Directions regarding care in the event of a terminal or irreversible condition must be documented to ensure that they are legally binding. C. Instructions about actions to be taken in the event of a client's terminal or irreversible condition are not legally binding. D. The client signs a document that designates another person to make legally binding healthcare decisions if client is unable to do so.

Answer: The client signs a document that designates another person to make legally binding healthcare decisions if client is unable to do so. The durable power of attorney is a legal document or a form of advance directive that designates another person to voice healthcare decisions when the client is unable to do so. A durable power of attorney for health directives is legally binding (A). (B, C and D) do not include the legal parameters that must be determined by the client in the event the client is unable to make a healthcare decision, which can be changed by the client at any time.

A female client with severe depression is given information about the risks, benefits, alternatives, and expected outcomes of electroconvulsive therapy (ECT) and signs the informed consent for treatment. After the client's family leaves, the client tells the nurse, "I signed the papers because my husband told me I will be deported if my depression is not cured." What information should the nurse report to the healthcare provider? A. The client is not competent to sign permission for treatment. B. All the elements of informed consent were met. C. The client's consent may have been coerced. D. The woman may not fully understand the risks and benefits.

Answer: The client's consent may have been coerced. Informed consent requires that the choice is freely given. Although the staff acted ethically and observed the client's right to give informed consent, the decision may have been coerced (A) based on family pressure. (B, C, and D) are not accurate.

The healthcare provider discontinues prednisone, a glucocorticoid, for a client with chronic obstructive pulmonary disease. What instructions should the nurse give the client about the regimen to follow? A. The dose must be tapered over the course of 7 to 10 days. B. Another glucocorticoid should be used to prevent cross-tolerance. C. Life-long treatment is common for chronic disease. D, The drug should be stopped immediately if no longer needed.

Answer: The dose must be tapered over the course of 7 to 10 days. To minimize the impact of adrenal insufficiency, withdrawal of exogenous glucocorticoids should be done by gradually decreasing the dosage over several days (C). Prolonged treatment with a glucocorticoid is not indicated for life (A) and can cause life-threatening adrenal insufficiency if abruptly terminated (B). Tapering the dosage should be done rather than substituting another glucocorticoid (D).

A client at 8-weeks gestation ask the nurse about the risk for a congenital heart defect (CHD) in her baby. Which response best explains when a CHD may occur? A. We don't really know what or when CHDs occur. B. The heart develops in the third to fifth weeks after conception. C. They usually occur in the first trimester of pregnancy. D. It depends on what the causative factors are for a CHD.

Answer: The heart develops in the third to fifth weeks after conception. The cardiovascular system is the first organ system to develop and function in the embryo. The blood vessel and blood formation begin in the third week, and the heart is developmentally complete in the fifth week (D). Regardless of the etiological factor, the heart is vulnerable during its period of development -- the third to fifth weeks. (A, B, and C) are inaccurate.

A single-parent mother brings her 3-year-old daughter to the emergency department after the child fell off a playground swing at school and hit her head. Which finding should prompt the nurse to advocate for continued hospital observation of the child instead of discharging the child to care at home? A. The mother states they do not have the money to pay for transportation home. B. The child had a 10 second loss of consciousness immediately after the fall. C. The mother is slurring her words and is not attentive to discharge instructions. D. The child indicates that she is tired and wants to take a nap.

Answer: The mother is slurring her words and is not attentive to discharge instructions Having a responsible adult to make on-going observations is the most important criteria for discharging anyone to their home after a head injury. The child (who needs observation) should not go home with an impaired adult (C). Alternative arrangements can be made regarding the follow-up care (A). The events of the head injury do not necessarily indicate the need for hospitalized observation (B). It is normal to be drowsy after a concussion (D); immediate intervention is needed if the child cannot be aroused from sleep.

An elderly client who requires frequent monitoring fell and fractured a hip. Which nurse is at greatest risk for a malpractice judgment? A. The nurse assigned to care for the client who was at lunch at the time of the fall. B. The nurse who transferred the client to the chair when the fall occurred. C. A nurse who worked the 7 to 3 shift at the hospital and wrote poor nursing notes. D. The charge nurse who completed rounds 30 minutes before the fall occurred.

Answer: The nurse who transferred the client to the chair when the fall occurred. The four elements of malpractice are: breach of duty owed, failure to adhere to the recognized standard of care, direct causation of injury, and evidence of actual injury. The hip fracture is the actual injury and the standard of care was "frequent monitoring." (C) implies that duty was owed and the injury occurred while the nurse was in charge of the client's care. There is no evidence of negligence in (A, B, and D).

The nurse is reviewing the use of the patient-controlled analgesia (PCA) pump with a client in the immediate postoperative period. The client will receive morphine 1 mg IV per hour basal rate with 1 mg IV every 15 minutes per PCA to total 5 mg IV maximally per hour. What assessment has the highest priority before initiating the PCA pump? A. The expiration date on the morphine syringe in the pump. B. The type of anesthesia used during the surgical procedure. C. The client's subjective and objective signs of pain. D. The rate and depth of the client's respirations.

Answer: The rate and depth of the client's respirations. A life-threatening side effect of intravenous administration of morphine sulfate, an opiate narcotic, is respiratory depression (B). The PCA pump should be stopped and the healthcare provider notified if the client's respiratory rate falls below 12 breaths per minute, and the nurse should anticipate adjustments in the client's dosage before the PCA pump is restarted. (A, C, and D) provide helpful information, but are not as high a priority as the assessment described in (B).

After abdominal surgery, a male client is prescribed low molecular weight heparin (LMWH). During administration of the medication, the client asks the nurse why he is receiving this medication. Which is the best response for the nurse to provide? A. This medication enhances antibiotics to prevent infection. B. This medication dissolves any clots that develop in the legs. C. This medication is a blood thinner given to prevent blood clot formation. D. This abdominal injection assists in the healing of the abdominal wound.

Answer: This medication is a blood thinner given to prevent blood clot formation Unfractionated heparin or low molecular weight heparin (LMWH) is an anticoagulant that inhibits thrombin-mediated conversion of fibrinogen to fibrin and is given prophylactically to prevent postoperative venous thrombosis (A) or to treat pulmonary embolism or deep vein thrombosis following knee and abdominal surgeries. Heparin does not dissolve clots but prevents clot extension or further clot formation (C). The anticoagulant heparin does not prevent infection (B) or influence operative wound healing (D).

A registered nurse (RN) is caring for several clients on a progressive care "step-down" unit. After assessing the clients, which clerical task should the nurse assign to a unlicensed assistive personnel (UAP)? A. Record the presence of blood-tinged urine and the hourly Foley output on the flow sheet. B. Document the type and amount of drainage on a new surgical dressing in the progress note. C. Transcribe the vital signs from a unit worksheet to the individual graphic page in the client charts. D. Chart pulse oximeter readings and type of breath sounds auscultated in the medical record.

Answer: Transcribe the vital signs from a unit worksheet to the individual graphic page in the client charts. Recording the vital signs on the graphic record (D) does not entail assessment or evaluation of the findings, so the UAP may perform this function. RNs may not delegate assessment or documentation responsibilities to UAPs. RNs must complete assessment activities and record findings in the medical record. The RN is responsible for including the evaluation of the vital signs in the nursing assessment. (A, B, and C) include activities that are within the scope of practice for the RN, but cannot be delegated to the UAP.

The registered nurse (RN) and unlicensed assistive personnel (UAP) are working together to provide care for a bedfast client needing total care, medications, and Foley catheter irrigation. How should the RN assign the client's care? A. UAP: Personal care, catheter irrigation, I&O. RN: Medications. B. UAP: Catheter irrigation, I&O. RN: Medications. Both provide personal care. C. UAP: Personal care, I&O. RN: Catheter irrigation, medications. D. UAP: Personal care. RN: Medications, catheter irrigation, I&O.

Answer: UAP: Personal care, I&O. RN: Catheter irrigation, medications. The RN is responsible for medication administration and sterile procedures such as catheter irrigation. The UAP is qualified to provide personal care and measure I&O. Based on these management concepts (D) provides the best assignment of the client's care.

Which statement correctly identifies a written learning objective for a client with peripheral vascular disease? A. After instruction, the nurse will ensure the client understands foot care rationale. B. The nurse will provide client instruction for daily foot care. C. Upon discharge, the client will list three ways to protect the feet from injury. D.The client will demonstrate proper trimming toenail technique.

Answer: Upon discharge, the client will list three ways to protect the feet from injury. An objective should contain four elements: who will perform the activity or acquire the desired behavior, the actual behavior that the learner will exhibit, the condition under which the behavior is to be demonstrated, and the specific criteria to be used to measure success. (C) is a concise statement that is a learning objective that defines exactly how the client will demonstrate mastery of the content. (A, B, and D) lack one or more of these elements.

Which action by the nurse is most helpful in communicating with a preschool-aged child? A. Use a doll to play and communicate. B. Speak clearly and directly to the child. C. Play a board game with the child. D. Approach when a parent is not present.

Answer: Use a doll to play and communicate. Communicating through play with a doll (B) or other toy gives time for the child to feel comfortable with a stranger. (A) may frighten some children and is usually not as effective as (B). To provide security and comfort, preschool-aged children should be approached when a parent is present, not (C). (D) is too advanced for a preschooler.

The nurse-manager observes that a staff nurse consistently fails to complete assigned care for clients who are obese. When counseling this employee, what issue is the priority concern? A. Reduction of client complaints. B. Dissatisfaction of co-workers. C. Poor time management skills. D. Violation of ethical principles.

Answer: Violation of ethical principles. The priority concern is the lack of fair and equal treatment of obese clients assigned to this staff nurse for care. This reflects a violation of the ethical principle of justice (A). Counseling the nurse about (B) is important because using time effectively allows the nurse to ensure that all clients receive fair and equal treatment, but this is of less concern than (A). (C and D) may also be important concerns, but they are secondary to ensuring justice.

The mother of a 2-year-old boy consults the nurse about her son's increased temper tantrums. The mother states, "Yesterday he threw a fit in the grocery store, and I did not know what to do. I was so embarrassed. What can I do if this occurs again?" Which recommendation is best for the nurse to provide this mother? A. Paddle him gently as soon as the behavior is initiated. B. Walk away from him and ignore the behavior. C. Quietly remind him that others are watching him. D. Immediately put him in "time-out."

Answer: Walk away from him and ignore the behavior. The best approach for a toddler is to ignore the attention-seeking behavior (D). The parent should be somewhat nearby, within view of the child but should avoid reinforcing the behavior in any way. Tantrums can sometimes be avoided by talking to the child before the situation occurs. (A, B, and C) would all provide attention for the inappropriate behavior.

When conducting a hygiene class for adolescent girls, it is important for the nurse to include which instruction about preventing toxic shock syndrome? A. Douche every month following menstruation. B. Wash your hands before inserting a tampon. C. Use super absorbent tampons. D. Wear cotton underwear.

Answer: Wash your hands before inserting a tampon The single most effective means of preventing infection is handwashing (A). (B and D) are contraindicated. (C) is not related to prevention of toxic shock syndrome.

The nurse is assigning care for a 4-year-old child with otitis media and is concerned about the child's increasing temperature over the past 24 hours. When planning care for this child, it is important for the nurse to consider that A. only an RN should be assigned to monitor this child's temperature. B. a tympanic measurement of temperature will provide the most accurate reading. C. the licensed practical nurse should be instructed to obtain rectal temperatures on this child. D. the healthcare provider should be asked to prescribe the method for measurement of the child's temperatures.

Answer: a tympanic measurement of temperature will provide the most accurate reading. (B) A tympanic membrane sensor is an excellent site because both the eardrum and hypothalamus (temperature-regulating center) are perfused by the same circulation. The sensor is unaffected by cerumen and the presence of suppurative or unsuppurative otitis media does not effect measurement. RULE OF THUMB: for management--sterile procedures should be assigned to licensed personnel. Management skills will be tested on the NCLEX! An RN is not required (A). Rectal temperature measurement (C) is less accurate because of the possibility of stool in the rectum. (D) is unnecessary

A client is admitted to the hospital with a medical diagnosis of pneumococcal pneumonia. The nurse knows that the prognosis for gram-negative pneumonias (such as E. coli, Klebsiella, Pseudomonas, and Proteus) is very poor because A. they occur in healthy young adults who have recently been debilitated by an upper respiratory infection. B. gram-negative organisms are more resistant to antibiotic therapy. C. they occur in the lower lobe alveoli which are more sensitive to infection. D. gram-negative pneumonias usually affect infants and small children.

Answer: gram-negative organisms are more resistant to antibiotic therapy. The gram-negative organisms are resistant to drug therapy (B) which makes recovery very difficult. Gram-negative pneumonias affect all lobes of the lung (C). The mean age for contracting this type of pneumonia is 50 years (A and D), and it usually strikes debilitated persons such as alcoholics, diabetics, and those with chronic lung diseases.

The nurse would be correct in withholding a dose of digoxin in a client with congestive heart failure without specific instruction from the healthcare provider if the client's A. blood pressure is 104/68. B. serum digoxin level is 1.5. C. serum potassium level is 3. D. apical pulse is 68/min.

Answer: serum potassium level is 3. Hypokalemia (C) can precipitate digitalis toxicity in persons receiving digoxin which will increase the chance of dangerous dysrhythmias (normal potassium level is 3.5 to 5.5 mEq/L). The therapeutic range for digoxin is 0.8 to 2 ng/ml (toxic levels=>2 ng/ml); (B) is within this range. (B) would not warrant the nurse withholding the digoxin. The nurse should withhold the digoxin if the apical pulse is less than 60/min (D).

A 77-year-old female client is admitted to the hospital. She is confused, has no appetite, is nauseated and vomiting, and is complaining of a headache. Her pulse rate is 43 beats per minute. Which question is a priority for the nurse to ask this client or her family on admission? "Does the client A. have her own teeth or dentures?" B. take aspirin and if so, how much?" C. take nitroglycerin?" D. take digitalis?"

Answer: take digitalis?" Elderly persons are particularly susceptible to digitalis intoxication (D) which manifests itself in such symptoms as anorexia, nausea, vomiting, diarrhea, headache, and fatigue. Although it is important to obtain a complete medication history (B and C), the symptoms described are classic for digitalis toxicity, and assessment of this problem should be made promptly. (A) is irrelevant.

A client has taken steroids for 12 years to help manage chronic obstructive pulmonary disease (COPD). When making a home visit, which nursing function is of greatest importance to this client? Assess the client's A. blood pressure, both standing and sitting. B. temperature. C. pulse rate, both apically and radially. D. skin color and turgor.

Answer: temperature It is very important to check the client's temperature (B). Infection is the most common factor precipitating respiratory distress. Clients with COPD who are on maintenance doses of corticosteroids are particularly predisposed to infection. (A and C) are important data for baseline and ongoing assessment, but they are not as important as temperature measurement for this client who is taking steroids. Assessment of skin color and turgor is less important (D).

The father of an 8-year-old tells the nurse he is interested in seeing his child succeed in soccer. The nurse talks with the boy, who expresses a sincere interest in playing chess and feels like a failure at soccer. How should the nurse respond to this father? A. The father should decrease his expectations to give the son a chance to succeed. B. The child has an introverted personality and should be encouraged to play isolated games. C. The child should be given opportunities to achieve a sense of competency in an area he chooses. D. The father should encouraged the son to participate in team sports instead of less physical activities.

Answer:The child should be given opportunities to achieve a sense of competency in an area he chooses. According to Erickson, the developmental stage "Industry versus inferiority" builds feelings of confidence, competence, and industry if there is achievement in an area of interest. If a child believes that he or she cannot measure up to society's expectations, the child loses confidence and may not find pleasure in the activity. Children should be encouraged to do the things they enjoy and succeed in (D). The father does not need to decrease his expectations (A), but should be encouraged to shift the expectation to an activity the child takes pleasure in. (B) does not encourage autonomy. (C) can cause a feeling of inadequacy.

The nurse is using the Ages and Stages Questionnaire (ASQ) to screen a 12-month-old infant during a well-child visit. When the parents ask the nurse the reason for this procedure, which response provides the best explanation? A. This test measures intellectual ability and screens for possible learning difficulties later in school. B. The procedure tests cognitive, physical, and psychological areas of development. C. The examination screens for early speech difficulties so early treatment can begin. D. This tool identifies achievement of development milestones in infants and young children.

Answer:This tool identifies achievement of development milestones in infants and young children. The ASQ is a screening tool for children one month to 5.5 years of age to identify strengths and developmental- social-emotional delays in normal early developmental milestones (A). (B, C, or D) are not the focus of the ASQ.

Arrange the order of donning personal protective equipment (PPE) while caring for a client with isolation precautions. Apply clean gloves within the gown Apply the cover gown, pull the sleeves down to the wrists, and tie the gown securely at the neck and waist Bring the glove cuffs over the edge of the gown sleeves Apply eyewear or goggles snugly around the face and eyes Apply either a surgical mask or a respirator around the mouth and nose

Apply the cover gown, pull the sleeves down to the wrists, and tie the gown securely at the neck and waist Apply either a surgical mask or a respirator around the mouth and nose Apply eyewear or goggles snugly around the face and eyes Apply clean gloves within the gown Bring the glove cuffs over the edge of the gown sleeves When preparing to enter an isolation room, the nurse first needs to apply a cover gown pull the sleeves down to wrist, and tie securely at neck and waist. The nurse should wear either a surgical mask or a respirator around his or her mouth and nose. If necessary, apply eyewear or goggles snugly around the face and eyes. Next, the nurse should wear gloves within the gown and pull the glove cuffs over the gown sleeves.

The student nurse prepares a concept map while caring for a client recovering from surgery. What is the first step that the student nurse should take when preparing the concept map? Multiple choice question Assess the client and gather information. Arrange cues into clusters that form patterns. Identify patterns reflecting the client's problem. Identify specific nursing diagnoses for the client.

Arrange cues into clusters that form patterns. A concept map is a visual representation of the connection between the client's many health problems. The first step is to arrange all the cues into clusters that form patterns. This helps the nurse identify specific nursing diagnoses for the client. During the assessment stage, the nurse assesses the client and gathers information. This step is performed before preparing the concept map. After placing all cues into clusters, the nurse begins to identify patterns reflecting the client's problem. The concept map helps the nurse obtain a holistic view of the client's needs. The next step is to identify specific diagnoses so that appropriate nursing interventions can be provided.

Which activity by the community nurse can be considered an illness prevention strategy? Multiple choice question Encouraging the client to exercise daily Arranging an immunization program for chicken pox Teaching the community about stress management Teaching the client about maintaining a nutritious diet

Arranging an immunization program for chicken pox An illness prevention program protects people from actual or potential threats to health. A chickenpox immunization program is an illness prevention program. It motivates the community to prevent a decline in health or functional levels. A health promotion program encourages the client to maintain the present levels of health. The nurse promotes the health of the client by encouraging the client to exercise daily. Wellness education teaches people how to care for themselves in a healthy manner. The nurse provides wellness education by teaching about stress management. The nurse promotes the health of the client by teaching the client to maintain a nutritious diet.

A nurse is caring for a client admitted with cardiovascular disease. During the assessment of the client's lower extremities, the nurse notes that the client has thin, shiny skin; decreased hair growth; and thickened toenails. What might this indicate? Multiple choice question Venous insufficiency Arterial insufficiency Phlebitis Lymphedema

Arterial insufficiency Clients experiencing arterial insufficiency present with extremities that become pale when elevated and dusky red when lowered. Lower extremities may also be cool to touch, pulses may be absent or mild, and skin may be shiny and thin with decreased hair growth and thickened nails. Clients with venous insufficiency often have normal-colored extremities, normal temperature, normal pulses, marked edema, and brown pigmentation around the ankles. Phlebitis is an inflammation of a vein that occurs most often after trauma to the vessel wall, infection, and immobilization. Lymphedema is swelling in one or more extremities that is a direct result of impaired flow of the lymphatic system.

Arrange the events of communication throughout the nursing process in chronological order. Performing verbal, visual, auditory, and tactile health teaching activities Documenting expected outcomes Identifying the factors affecting the outcomes Assessing the medical records and diagnostic tests Intrapersonal analysis of the assessment findings

Assessing the medical records and diagnostic tests Intrapersonal analysis of the assessment findings Documenting expected outcomes Performing verbal, visual, auditory, and tactile health teaching activities Identifying the factors affecting the outcomes The first step of communication throughout the nursing process is assessment, which involves assessing medical records and diagnostic tests. The second step is nursing diagnosis, which involves the intrapersonal analysis of assessment findings. The third step is planning, which involves the documentation of expected outcomes. The fourth step is implementation, which involves performing verbal, visual, auditory, and tactile health teaching activities. The final step is evaluation, which involves identifying the factors affecting the outcomes.

What should be the priority action of the nurse who is caring for a client with a leg in traction(broken leg)?

Assessing the skin integrity should come before assessing the broken bone or assessing for mobility because skin breakdown can occur quickly

A nurse is explaining the nursing process to a nursing assistant. Which step of the nursing process should include interpretation of data collected about the client? 1 Evaluation 2 Assessment 3 Nursing interventions 4 Proposed nursing care

Assessment An actual or potential client health problem is based on the analysis and interpretation of the data previously collected during the assessment phase of the nursing process. Gathering data is included in the client's assessment. Nursing interventions are based on the earlier steps of the nursing process. The plan of care includes nursing actions to meet client needs. The needs first must be identified before nursing actions are planned.

An adolescent is taken to the emergency department of the local hospital after stepping on a nail. The puncture wound is cleansed and a sterile dressing applied. The nurse asks about tetanus immunization. The adolescent responds that all immunizations are up to date. Penicillin is administered, and the client is sent home with instructions to return if there is any change in the wound area. A few days later, the client is admitted to the hospital with a diagnosis of tetanus. Legally, what is the nurse's responsibility in this situation? Multiple choice question The nurse's judgment was adequate, and the client was treated accordingly. The possibility of tetanus was not foreseen because the client was immunized. Nurses should routinely administer immunization against tetanus after such an injury. Assessment by the nurse was incomplete, and as a result the treatment was insufficient.

Assessment by the nurse was incomplete, and as a result the treatment was insufficient. The nurse's data collection was not adequate because the nurse did not ask about the date of the previous tetanus inoculation. The nurse failed to support the life and well-being of a client. The nurse's assessment was not thorough in regard to determining the date of immunization. It was essential to determine when the client was last immunized; for a "tetanus-prone" wound, like a puncture from a rusty nail, some form of tetanus immunization usually is given. Administering immunization against tetanus is not an independent function of the nurse.

Which nursing activities are examples of primary prevention? Select all that apply. 1 Preventing disabilities 2 Correcting dietary deficiencies 3 Establishing goals for rehabilitation 4 Assisting with immunization programs 5 Facilitating a program about smoking cessation

Assisting with immunization programs Facilitating a program about smoking cessation Immunization programs prevent the occurrence of disease and are considered primary interventions. Stopping smoking prevents the occurrence of disease and is considered a primary intervention. Preventing disabilities is a tertiary intervention. Correcting dietary deficiencies is a secondary intervention. Establishing goals for rehabilitation is a tertiary intervention.

Which physical assessment technique involves listening to the sounds of the body? Multiple choice question Palpation Inspection Percussion Auscultation

Auscultation Auscultation involves listening to the sounds of the body. Palpation involves using the sense of touch to assess and collect data. An inspection involves the nurse carefully looking to collect data. Percussion involves tapping the skin with the fingertips to vibrate underlying tissues and organs.

A client was admitted to a surgical unit in an unconscious state due to head trauma. Which site would be most appropriate to obtain the client's temperature? Multiple choice question Oral Axilla Temporal artery Tympanic membrane

Axilla The axilla would be the most appropriate site to obtain a temperature measurement in a client who is unconscious due to head trauma. The oral route is not accessible when the client is unconscious. Because the client is in a surgical unit, his or her head may be covered. Therefore, obtaining a temperature measurement through the temporal artery or tympanic membrane may not be possible.

The nurse is preparing to administer a unit of packed red blood cells to a client. Which type of IV fluid should be used to facilitate the infusion of blood for this client? A. Lactated Ringer's B. Normal saline C. Dextrose 5% and water D. Dextrose 5% and normal saline

B. NS

What clinical finding does a nurse anticipate when admitting a client with an extracellular fluid volume excess? Rapid, thready pulse Distended jugular veins Elevated hematocrit level Increased serum sodium level

Because of fluid overload in the intravascular space, the neck veins become visibly distended. Rapid, thready pulse and elevated hematocrit level occur with a fluid deficit. If sodium causes fluid retention, its concentration is unchanged; if fluid is retained independently of sodium, its concentration is decreased.

When permitted by the client, the nurse should always take the time to keep the family informed about what is happening to the client. The purpose of this approach is that informed families will be what? 1 Able to decrease the client's anxiety 2 More relaxed when interacting with the client 3 Less likely to cause problems with the nursing staff 4 Better equipped to undertake necessary family role changes

Better equipped to undertake necessary family role changes Early notification provides an opportunity to prepare for change. The ability to decrease the client's anxiety, families being more relaxed, and families being less likely to cause problems with nursing staff may be secondary gains, but are not the primary purpose.

A nurse is caring for an elderly client with dementia who has developed dehydration as a result of vomiting and diarrhea. Which assessment best reflects the fluid balance of this client? Multiple choice question Skin turgor Intake and output results Client's report about fluid intake Blood lab results

Blood lab results Blood lab results provide objective data about , as well as about hemoglobin and hematocrit. Skin turgor is not a reliable indicator of hydration status for the elderly client because it is generally decreased with age. Intake and output results provide data only about fluid balance, but do not present a comprehensive picture of the client's fluid and electrolyte status; therefore this is not the best answer. The client's report about fluid intake is subjective data in general and not reliable because this client has dementia and therefore has memory problems .

The client is receiving high-flow intravenous (IV) fluid replacement therapy. Which nursing assessment findings are consistent with fluid volume overload? Select all that apply.

Bounding pulse Presence of dependent edema Neck vein distention in the upright position

The nurse explains the Cheyne-Stokes breathing pattern to a student nurse. The nurse knows the student nurse needs further education when the student states: "Cheyne-Stokes respirations commonly occur in patients in metabolic acidosis" "Cheyne-stokes respirations commonly occur in patient with uncontrolled diabetes" "Cheyne-Stokes respirations commonly occur in patients actively dying" "Cheyne-Stokes respirations commonly occur in patients with fluid volume overload"

C. Respiration pattern in patients that are actively dying.

A client is diagnosed with acquired immunodeficiency syndrome (AIDS). When examining the client's oral cavity, the nurse assesses white patchy plaques on the mucosa. The nurse recognizes that this finding most likely represents what opportunistic infection? Multiple choice question Cytomegalovirus Histoplasmosis Candida albicans Human papillomavirus

Candida albicans White patchy plaques on the oral mucosa would most likely be a result of C. albicans, a yeastlike fungal infection. This condition is also known as "thrush." Cytomegalovirus may cause a serious viral infection in persons with human immunodeficiency virus (HIV), resulting in retinal, gastrointestinal, and pulmonary manifestations. Histoplasmosis is an infection caused by inhalation of spores of the fungus Histoplasma capsulatum and is characterized by fever, malaise, cough, and lymphadenopathy. Human papillomavirus typically manifests as warts on the hands and feet, as well as mucous membrane lesions of the oral, anal, and genital cavities. It may be transmitted without the presence of warts through body fluids, with some forms associated with cancerous and precancerous conditions.

A nurse is applying a dressing to a client's surgical wound using sterile technique. While engaging in this activity, the nurse accidentally places a moist sterile gauze pad on the cloth sterile field. What physical principle is applicable for causing the sterile field to become contaminated? 1 Dialysis 2 Osmosis 3 Diffusion 4 Capillarity

Capillarity When a sterile surface becomes wet, microorganisms from the unsterile surface below the sterile field will be drawn up, contaminating the sterile field. The absorption of fluids by gauze results from the adhesion of water to the gauze threads; the surface tension of water causes contraction of the fiber, pulling fluid up the threads. Dialysis is separation of substances in solution using their differing rates of diffusion through a membrane. Osmosis refers to movement of water through a semipermeable membrane. Diffusion is movement of molecules from a high to a low concentration.

The nurse assesses an elderly client with a diagnosis of dehydration and recognizes which finding as an early sign of dehydration? Multiple choice question Sunken eyes Dry, flaky skin Change in mental status Decreased bowel sounds

Change in mental status adults are sensitive to changes in fluid and electrolyte levels, especially sodium, potassium, and chloride. These changes will manifest as a change in mental status and confusion. It is difficult to assess dehydration in older adults based on sunken eyes, dry skin, and decreased bowel sounds because these can be prominent as general normal findings in the elderly client.

Cheyne-Stokes Respiration

Cheyne-Stokes respiration is also known as periodic respiration, with cycles of respiration that are increasingly deeper then shallower with possible periods of apnoea.

A client complains of chronically recurring ulcers on the lower leg. Upon assessment, a nurse finds the absence of hair growth on the legs and asks the client to consult the primary healthcare provider immediately. Which condition does the nurse suspect? Multiple choice question Phlebitis Clubbing Occlusion Circulatory insufficiency

Circulatory insufficiency Chronically recurring ulcers in the lower legs or the feet accompanied by an absence of leg hair growth are serious signs of circulatory insufficiency. This finding requires a primary healthcare provider's intervention. The symptoms of phlebitis include localized redness, tenderness, and swelling over the vein sites of the calves. Clubbing is caused by insufficient oxygenation at the periphery. This can result from chronic emphysema and congenital heart disease. Occlusion is characterized by pain, pallor, pulselessness, paresthesias, and paralysis.

A nurse is preparing to administer an ophthalmic medication to a client. What techniques should the nurse use for this procedure? Select all that apply. Clean the eyelid and eyelashes. Place the dropper against the eyelid. Apply clean gloves before beginning of procedure. Instill the solution directly onto cornea. Press on the nasolacrimal duct after instilling the solution.

Clean the eyelid and eyelashes. Apply clean gloves before beginning of procedure. Press on the nasolacrimal duct after instilling the solution. Cleaning of the eyelids and eyelashes helps to prevent contamination of the other eye and lacrimal duct. Application of gloves helps to prevent direct contact of the nurse with the client's body fluids. Applying pressure to the nasolacrimal duct prevents the medication from running out of the eye. The dropper should not touch the eyelids or eyelashes in order to prevent contamination of the medication in the dropper. The medication should not be instilled directly onto the cornea because cornea has many pain fibers and is therefore very sensitive. The medication is to be instilled into the lower conjunctival sac.

A nurse is assessing clients with gastrointestinal problems. Which client does the nurse suspect to have shigellosis?

Client 2; Food consumed: milk and seafood; Symptoms: Abdominal cramps and diarrhea; Onset of symptoms: 12 hours after ingestion Shigellosis is a food-borne disease and may be due to the ingestion of milk products, seafood, or salad. The symptoms of infection include abdominal cramps and severe diarrhea and can occur 12 hours after ingestion. Therefore, shigellosis is suspected in client 2. Client 1, who has symptoms of severe abdominal cramps, pain, vomiting, diarrhea, perspiration, headache, and fever after consuming custard or processed meats, may have a Staphylococcus infection. These symptoms may appear 3 days after ingestion contaminated foods. Client 3, who has symptoms of severe diarrhea, fever, headache, and breathing difficulty after consuming soft cheese, meat, or unpasteurized milk, may have an Escherichia coli infection. These symptoms may appear 3 days after ingestion of contaminated food. Client 4 with symptoms of severe diarrhea, cramps, and vomiting after consuming milk, custards, egg dishes, or sandwich fillings may have salmonellosis. These symptoms may appear 4 days after ingestion of those foods.

A nurse performs lung assessments of four clients. The details are given below. Which client has inflamed pleura? Client A- Site: Overall Lung; Type of Sound: Inspiratory & Expiratory wheezes Client B- Site: Trachea & Bronchi; Type of Sound: Coarse crackles Client C- Site: Right & Left lung bases; Types of Sound: Fine crackles Client D- Site: Anterior lateral lung; Type of Sound: Frictional rub

Client D The breathing sounds in a pleural rub or an inflamed pleura are of a dry or grating quality that is heard in the lower portion of the anterior lateral lung, as observed in client D. High-pitched, continuous musical sounds heard all over the lung are wheezing breath sounds heard when there is a high-velocity airflow through severely narrowed or an obstructed airway. Loud, low-pitched, rumbling coarse sounds heard in the trachea and bronchi are rhonchi, which are observed during muscular spasm or when fluid or mucus is present the in larger airways. Fine crackles, medium crackles, and coarse crackles heard in client C are heard in lung bases due to random and sudden reinflation of groups of alveoli, which causes a disruptive passage of air through the small airways.

The nurse is preparing discharge instructions for a client who has begun to demonstrate signs of early Alzheimer dementia. The client lives alone. The client's adult children live nearby. According to the prescribed medication regimen, the client is to take medications six times throughout the day. What is the priority nursing intervention to assist the client with taking the medication? 1 Contact the client's children and ask them to hire a private duty aide who will provide round-the-clock care. 2 Develop a chart for the client, listing the times the medication should be taken. 3 Contact the primary healthcare provider and discuss the possibility of simplifying the medication regimen. 4 Instruct the client and client's children to put medications in a weekly pill organizer.

Contact the primary healthcare provider and discuss the possibility of simplifying the medication regimen. Contacting a medical care provider and discussing the possibility of simplifying the client's medication regimen will make it possible to use a weekly pill organizer : an empty pill box will remind the client who has a short-term memory deficit due to Alzheimer dementia that medication was taken and will prevent medication being taken multiple times. The client does not require 24-hour supervision because the client is in the onset of the Alzheimer dementia, and the major issue is a short-term memory loss. A chart may be complex and difficult to understand for the client and will require the client to perform cognitive tasks multiple times on daily basis that may be beyond the client's ability. Use of the weekly pill organizers will be difficult with the current medication regimen when the client has to take medications six times a day; the medication regimen has to be simplified first.

What principle of teaching specific to an older adult should the nurse consider when providing instruction to such a client recently diagnosed with diabetes mellitus? 1 Knowledge reduces general anxiety. 2 Capacity to learn decreases with age. 3 Continued reinforcement is advantageous. 4 Readiness of the learner precedes instruction.

Continued reinforcement is advantageous. Neurologic aging causes forgetfulness and a slower response time; repetition increases learning. That knowledge reduces general anxiety, that the capacity to learn decreases with age, and that the readiness of the learner precedes instruction reflect principles applicable to all learning regardless of the client's age. Learning occurs, but it may take longer.

A client who sustained head injuries is admitted to the hospital. During assessment of cranial nerves, the nurse notices that the client lost the perception of taste, especially in the anterior portion of the tongue. Which cranial nerve might have been injured in this client? Cranial nerve X Cranial nerve IX Cranial nerve XII Cranial nerve VII

Cranial nerve VII Cranial nerve VII is the facial nerve. Injury to the facial nerve limits the sensory impulses from the anterior two-thirds of the tongue, along with altered facial expressions. Cranial nerve X is the vagus nerve, injury to which causes limitation of palatal movements. Cranial nerve IX is the glossopharyngeal nerve. Injury to this nerve results in loss of taste impulses from the posterior one-third of the tongue. Cranial nerve XII is the hypoglossal nerve, damage of which results in improper movements of the tongue.

A client has been admitted with a medical diagnosis of dehydration secondary to diuretic therapy. What nursing physical assessment criteria would the nurse expect to find? A. Distended neck and peripheral veins B. Increased BP with clear breath sounds C. Fill bounding pulse with increased rate D. Dry mucous membranes with decreased salivation

D. Dry mucous membranes with decreased salivation

The nurse is waiting for 1 unit of packed red blood cells to arrive for her patient. The nurse calls the blood bank and they say it may take a little more than an hour to send the blood. What is an appropriate IV fluid to administer to the patient while waiting? 0.45% Normal Saline D5W 0.9% normal saline Lactated ringers

D. LR

A client is diagnosed with third space syndrome. What does this mean? A. Fluid has shifted into the vascular space. B. There is an isotonic fluid volume excess. C. The client had a 10 lb. water weight loss. D. The client may have fluid in his peritoneal cavity.

D. The client may have fluid in his peritoneal cavity Rationale: Remember 3rd spacing is when fluid is "hanging out" and moved from the vascular space into the interstitial space

The client reports difficulty in breathing. The nurse auscultates lung sounds and assesses the respiratory rate. What is the purpose of the nurse's action? Data collection Data validation Data clustering Data interpretation

Data collection The nurse is gathering objective data to support the subjective data. The client's report of difficulty breathing is subjective data that needs to be supported by data from physical examination. The nurse reviews the database after data collection to decide if it is accurate and complete. This step is called data validation. Grouping of data that forms a pattern is called data clusters. The nurse uses critical thinking to interpret the data and analyze it before it is classified and organized into data clusters.

The nurse understands that the action of an antidiuretic hormone (ADH) is to do what? Reduce blood volume Decrease water loss in urine Increase urine output Initiate the thirst mechanism

Decrease water loss in urine

A client reaches the point of acceptance during the stages of dying. What response should the nurse expect the client to exhibit? 1 Apathy 2 Euphoria 3 Detachment 4 Emotionalism

Detachment When an individual reaches the point of being intellectually and psychologically able to accept death, anxiety is reduced and the individual becomes detached from the environment. Although detached, the client is not apathetic, but still may be concerned and use time constructively. Although resigned to death, the individual is not euphoric. In the stage of acceptance, the client is no longer angry or depressed.

The nurse is assessing a young client who presents with recurrent gastrointestinal disorders. On further assessment, the nurse learns that the client is experiencing job-related pressure. What is the most important nursing intervention for this client? Multiple choice question Educate the client on managing stress. Teach the client to maintain a balanced diet. Instruct the client to have regular health checkups. Ask the client to use sunscreen when working outdoors.

Educate the client on managing stress. The client is experiencing job-related pressure, so the nurse should educate the client about managing stress as it is a lifestyle risk factor. Stress threatens both mental health and physical well-being. Stress is associated with illnesses such as heart disease, cancer, and gastrointestinal disorders. The nurse teaches the client to maintain a balanced diet as a primary preventive care to promote health. The nurse should instruct the client to have regular health checkups as a primary preventive measure. The nurse should ask the client to use sunscreen when working outdoors to avoid excess sun exposure and prevent skin cancer.

The nurse administers 2 units of salt-poor albumin to a client with portal hypertension and ascites. The nurse explains to the client that the purpose of the albumin is to: Provide nutrients. Increase protein stores. Elevate the circulating blood volume. Divert blood flow away from the liver temporarily.

Elevate the circulating blood volume. Increasing oncotic pressure increases the client's circulating blood volume; salt-poor albumin pulls interstitial fluid into the blood vessels, restoring blood volume and limiting ascites. Nutrients are provided by total parenteral nutrition, not salt-poor albumin. Salt-poor albumin is not given to increase protein stores. Salt-poor albumin has no effect on diverting blood flow away from the liver.

While assessing the eyes of a client, a healthcare provider notices there is an obstruction to the outflow of aqueous humor. Which additional finding might be noted to support a diagnosis of glaucoma? Multiple choice question Blurred central vision Increased opacity of the lens Elevated intraocular pressure Changes in retinal blood vessels

Elevated intraocular pressure In glaucoma, there is an obstruction of the outflow of aqueous humor due to an intraocular structural damage, which may result from elevated intraocular pressure. Blurred central vision is seen in macular degeneration. Increased opacity of the lens may be seen in cataracts. Retinopathy may result from the changes in retinal blood vessels.

Which intervention by the nurse helps the family feel in control when the client is to be discharged home? 1 Instruct the family to ensure the client's room is safe. 2 Ask the family to ensure that the client has only low-fat meals. 3 Ask the family to coordinate with the staff at the rehabilitation center. 4 Ensure a family member is confident about changing dressings correctly.

Ensure a family member is confident about changing dressings correctly. The nurse should identify a family member who is capable and willing to learn how to change the client's dressings. The nurse should teach a family member and have that member demonstrate the process to ensure the procedure is executed correctly. This gives confidence to the client and family, who will feel in control when the client is discharged home. The nurse should not only instruct but also offer suggestions about rearranging the client's room to make it safe. The nurse should offer suggestions and ask the family for ideas on how to prepare low-fat meals that meet their ethnic considerations. The nurse should coordinate with the nursing staff at the rehabilitation center to ensure continuity of care.

Which first line medication would the nurse state is used to treat anaphylactic reactions?

Epinephrine

During a survey, the community nurse meets a client who has not visited a gynecologist after the birth of her second child. The client says that her mother or sister never had annual gynecologic examinations. Which factor is influencing the client's health practice? Multiple choice question Spiritual belief Family practices Emotional factors Cultural background

Family practices Family practices influence the client's perception of the seriousness of diseases. The client does not feel the need to seek preventive care measures because no family member practices preventive care. The client is not influenced by spiritual beliefs in this instance. An individual's spiritual beliefs and religious practices may restrict the use of certain forms of medical treatment. Emotional factors such as stress, depression, or fear may influence an individual's health practice; however, this client does not show signs of being affected by emotional factors. The client is said to be influenced by cultural background if he or she follows certain beliefs about the causes of illness and uses customary practices to restore health.

Arrange the steps involved in applying a surgical mask. Incorrect Hold the mask by its two strings or loops Secure two top ties at the back of head, with ties above the ears Tie two lower ties snugly around the neck with the mask well around the chin Find the top edge of mask Gently pinch the upper metal band around the bridge of the nose

Find the top edge of mask Hold the mask by its two strings or loops Secure two top ties at the back of head, with ties above the ears Tie two lower ties snugly around the neck with the mask well around the chin Gently pinch the upper metal band around the bridge of the nose If a mask is properly applied, it fits the mouth and nose securely so that pathogens and body fluids cannot enter or escape through the sides. First, the mask's top edge should be identified and the mask should be held by its two strings. Then, the two top ties are tied at the back of the head, with the ties above the ears. The lower two ties are tied around the neck with the mask well around the chin. Then, the upper metal band should be pinched around the bridge of the nose.

The nurse observes that an older client seldom eats the meat on the meal trays. The nurse discusses this observation with the client, and the client states, "I only eat meat once a week because old people don't need protein every day." What does the nurse determine that the client needs to be taught about? 1 Need for home-delivered meals 2 Foods that meet basic nutritional needs 3 Effect of aging on the need for some foods 4 Need for meat at least once per day throughout life

Foods that meet basic nutritional needs The need for nutrients, including protein, that meet basic nutritional needs continues throughout life. The priority is to educate the client, although home-delivered meals may be one way to provide adequate nutrition. Aging has no effect on the specific nutrients needed; however, it may influence digestion or absorption of food. Protein is needed every day, but it does not have to be in the form of meat.

A mother is worried about the sudden behavioral changes in her child. The child has suddenly developed a fear of certain people and places. The child's school performance is declining rapidly, and the child has developed poor relationships with his or her peers. After assessing the physical findings of the child, the nurse suspects child abuse. Which physical findings might have led the nurse to this suspicion? Multiple choice question Sunken eyes and loss of weight Uncommunicative and uninteractive with others Foreign bodies in the rectum, urethra, or vagina Strangulation marks on neck from rope burns or bruises

Foreign bodies in the rectum, urethra, or vagina One of the physical findings that may be required to confirm child abuse is the presence of foreign bodies in the rectum, urethra, or vagina. Weight loss and sunken eyes may be a physical finding for older adult abuse. When the abuse is related to an intimate partner, the nurse may observe strangulation marks on the neck from rope burns or bruises. Staying isolated and not communicating with others are behavioral findings that may be related to older adult abuse.

The spouse of a comatose client who has severe internal bleeding refuses to allow transfusions of whole blood because they are Jehovah's Witnesses. The client does not have a Durable Power of Attorney for Healthcare. What action should the nurse take? 1 Institute the prescribed blood transfusion because the client's survival depends on volume replacement. 2 Clarify the reason why the transfusion is necessary and explain the implications if there is no transfusion. 3 Phone the primary healthcare provider for an administrative prescription to give the transfusion under these circumstances. 4 Give the spouse a treatment refusal form to sign and notify the primary healthcare provider that a court order now can be sought.

Give the spouse a treatment refusal form to sign and notify the primary healthcare provider that a court order now can be sought. The client is unconscious. Although the spouse can give consent, there is no legal power to refuse a treatment for the client unless previously authorized to do so by a power of attorney or a healthcare proxy; the court can make a decision for the client. Explanations will not be effective at this time and will not meet the client's needs. Instituting the prescribed blood transfusion and phoning the primary healthcare provider for an administrative prescription are without legal basis, and the nurse may be held liable.

A client is receiving total parenteral nutrition (TPN) through a central venous access device. The nurse discovers that the TPN bag is empty and the next bag has not been received yet from the pharmacy. What is the most appropriate action for the nurse to take? Perform a finger stick glucose test and call the primary healthcare provider with the results. Hang a bag of 10% dextrose at the ordered TPN rate and place an urgent request for the next TPN bag. Discontinue the infusion and flush the IV line with saline solution until the next TPN bag is ready. Hang a bag of 5% dextrose at a keep-open rate and notify the nurse manager of the occurrence.

Hang a bag of 10% dextrose at the ordered TPN rate and place an urgent request for the next TPN bag.

The nurse is performing a weight assessment for different people in a community. Which question should the nurse ask a client to determine a disease-related change in weight? Multiple choice question Do you follow a strict calorie intake? Have you notices any changes in the social aspects of eating? Are you taking diuretics or insulin? Have you noticed any unintentional weight loss in the past six months?

Have you noticed any unintentional weight loss in the past six months? Unintentional or undesired weight loss during a certain period of time may indicate a weight change due to a disease, such as gastrointestinal problems. A strict calorie intake in a permitted limit is not related to any disease. Assessing the social aspects of a client's eating habits determines any lifestyle changes that may cause a weight change. Diuretics and insulin may cause weight loss or weight gain; this change is not disease-related.

An older adult with a history of diabetes reports giddiness, excessive thirst, and nausea. During an assessment, the nurse notices the client's body temperature as 105° F. Which condition does the nurse suspect in the client? Multiple choice question Heat stroke Heat exhaustion Accidental hypothermia Malignant hyperthermia

Heat stroke Older adults are more at a risk of heat stroke. Symptoms of heat stroke include giddiness, excessive thirst, nausea, and increased body temperature. Heat exhaustion is indicated by a fluid volume deficient. Heat exhaustion occurs when profuse diaphoresis results in excess water and electrolyte loss. Accidental hypothermia usually develops gradually and goes unnoticed for several hours. When the skin temperature drops below 95° F, the client suffers from uncontrolled shivering, memory loss, depression, and poor judgment. Malignant hyperthermia is an adverse effect of inhalational anesthesia that is indicated by a sudden rise in body temperature in intraoperative or postoperative clients.

The nurse tells a client undergoing diuretic therapy to avoid working in the garden on hot summer days. What condition is the nurse trying to prevent in this client? Multiple choice question Frostbite Heatstroke Hypothermia Hyperthermia

Heatstroke Clients undergoing diuretic therapy are at risk of heatstroke when exposed to temperatures higher than 40° C. Frostbite occurs when the body is exposed to ice-cold temperatures. Hypothermia is a condition in which the skin temperature drops below 36° C. Hyperthermia occurs when the body is exposed to temperatures higher than 38.5° C.

A client reports to the hospital with skin lesions. Upon physical examination, the nurse notices circumscribed elevations of the skin, measuring about 0.5 × 0.5 cm. The lesions are filled with serous fluid. What is the suspected cause of these skin lesions? Multiple selection question Venous stasis ulcer Arterial insufficiency Staphylococcal infection Herpes simplex infection

Herpes simplex infection Circumscribed elevated skin lesions filled with serous fluid smaller than 1 cm are called vesicles. Vesicles are found in conditions such as herpes simplex infection and chicken pox. Venous stasis ulcers are characterized by deep loss of skin surface that extends to the dermis and is associated with frequent bleeding. The appearance of shiny and translucent skin with loss of normal furrow indicates arterial insufficiency. In a staphylococcal infection, the skin lesion is similar to that of vesicle, but is filled with pus instead of serous fluid.

A client has a history of a persistent cough, hemoptysis, unexplained weight loss, fatigue, night sweats, and fever. Which risk should be assessed? Multiple choice question Lung cancer Cerebrovascular disease Cardiopulmonary alterations Human immunodeficiency virus (HIV) infection

Human immunodeficiency virus (HIV) infection A client with a history of persistent cough, hemoptysis, unexplained weight loss, fatigue, night sweats, or fever may have a human immunodeficiency virus (HIV) infection or tuberculosis. Lung cancer and cerebrovascular disease are risks to be assessed in the client with a history of tobacco or marijuana use. Cardiopulmonary alterations may be present in a client with a persistent cough (productive or nonproductive), sputum streaked with blood, or voice changes.

Which type of breathing pattern alteration is manifested with hypercarbia? Multiple choice question Eupnea Tachypnea Hypoventilation Kussmaul's respiration

Hypoventilation Hypercarbia may occur during hypoventilation. The respiratory rate is abnormally low and the depth of ventilation is depressed in hypoventilation. In eupnea, the normal rate and depth of respiration is interrupted while singing. The rate of breathing is regular, but abnormally rapid in tachypnea. Respirations are abnormally deep, regular, and the rate is increased in Kussmaul's respirations.

A client is admitted to the hospital for an elective surgical procedure. The client tells a nurse about the emotional stress of recently disclosing being a homosexual to family and friends. What is the nurse's first consideration when planning care? 1 Exploring the client's emotional conflict 2 Identifying personal feelings toward this client 3 Planning to discuss this with the client's family 4 Developing a rapport with the client's healthcare provider

Identifying personal feelings toward this client Nurses must identify their own feelings and prejudices because these may affect the ability to provide objective, nonjudgmental nursing care. Exploring a client's emotional well-being can be accomplished only after the nurse works through one's own feelings. The focus should be on the client, not the family. Health team members should work together for the benefit of all clients, not just this client.

The nurse is caring for a non-ambulatory client with a reddened sacrum that is unrelieved by repositioning. What nursing diagnosis should be included on the client's plan of care? Risk for pressure ulcer Risk for impaired skin integrity Impaired skin integrity, related to infrequent turning and repositioning Impaired skin integrity, related to the effects of pressure and shearing force

Impaired skin integrity, related to the effects of pressure and shearing force The impaired skin integrity is physiologically a result of unrelieved pressure and shearing force. This is supported by the data provided that the client is non-ambulatory and has a reddened sacrum. Risk for pressure ulcer is not an approved NANDA-I nursing diagnosis. The client's problem is not being "at risk" because the client already has an actual problem. Not enough information is provided to make the assumption that the impaired skin integrity is related to infrequent turning and repositioning.

After an eye assessment, the nurse finds that both of the client's eyes are not focusing on an object simultaneously and appear crossed. What could be the cause for this condition? Multiple choice question Loss of elasticity of the lens Impairment of the extraocular muscles Obstruction of the aqueous humor outflow Progressive degeneration of the center of the retina

Impairment of the extraocular muscles Strabismus is a condition where the eyes appear crossed; this condition is caused by the impairment of the extraocular muscles. A loss of lens elasticity may lead to presbyopia, which causes impaired near vision. An obstruction of the aqueous humor outflow may lead to glaucoma. The progressive degeneration of the center of the retina indicates macular degeneration and leads to blurred central vision.

The nurse recognizes that a common conflict experienced by older adults is the conflict between what? Multiple choice question Youth and old age Retirement and work Independence and dependence Wishing to die and wishing to live

Independence and dependence A common conflict confronting older adults is between the desire to be taken care of by others and the desire to be in charge of their own destiny. The conflict between the young and old age may occur but is not common. The conflict between the retirement and working may occur but is not common. The conflict between wishing to die and wishing to live may occur but is not common.

The nurse is evaluating the actions of a client with pneumonia performing incentive spirometry. Which action by the client indicates a need for correction?

Inhaling air fully before inserting the mouth piece

A nurse is discussing weight loss with an obese individual with Ménière disease. Which suggestion by the nurse is most important? Multiple choice question Limit intake to 900 calories per day. Enroll in an exercise class. Get involved in diversionary activities when there is an urge to eat. Keep a diary of all foods eaten each day.

Keep a diary of all foods eaten each day. Keeping a record of what one eats helps to limit nonconscious and nervous eating by making the individual aware of intake. Limiting calories to 900 per day is a severe restriction that requires a primary healthcare provider's prescription. Exercise causes rapid head movements, which may precipitate a Ménière attack. Although diversionary activities are a therapeutic intervention, the nurse first should make suggestions that help increase the client's awareness of personal eating habits.

A client with a suspected dysrhythmia is to wear a Holter monitor for 24 hours at home. What should the nurse instruct the client to do? Keep a record of the day's activities Avoid going through laser-activated doors Record the pulse and blood pressure every four hours Delay taking prescribed medications until the monitor is removed

Keep a record of the day's activities The purpose of monitoring is to correlate dysrhythmias with the client's reported activity. Laser-activated doors have no effect on a Holter monitor and will not affect the readings. Recording the pulse and blood pressure every four hours is not required for interpretation of the test. The client should take medication as prescribed and note it in the activities diary.

A client shows an increase in rate respirations that are abnormally deep and regular. What condition would the nurse expect? Multiple choice question Hypoventilation Biot's respiration Kussmaul's respiration Cheyne-Stokes respiration

Kussmaul's respiration Kussmaul's respiration is an alteration in the breathing process that is characterized by an increased and abnormal deep and regular rate of respiration. A client suffering from hypoventilation would have an abnormally low respiratory rate and the depth of ventilation is depressed. In Biot's respiration, respirations are abnormally shallow for two to three breaths, followed by irregular periods of apnea. An irregular respiratory rate and depth characterized by alternating periods of apnea and hyperventilation would be observed in a client with Cheyne-Stokes respiration.

Which positioning should be avoided while assessing a client with a history of asthma? Multiple choice question Sitting Supine Dorsal recumbent Lateral recumbent

Lateral recumbent The lateral recumbent position is used to assess heart function. A client with asthma or other respiratory problems may not tolerate the lateral recumbent position. The sitting position is used to assess the heart, thorax, and lungs; this position should be avoided in physically weakened clients. The supine position is used to assess the heart, abdomen, extremities, and pulses. The dorsal recumbent position is used for an abdominal assessment and to assess the head, neck, and lungs.

A client suffers hypoxia and a resultant increase in deoxygenated hemoglobin in the blood. What are the best sites to assess this condition? Select all that apply.

Lips Mouth Nail Beds

Which finding is inferred from a grade 4 intensity of heart murmurs? Multiple choice question Thrill is easily palpable Quiet and clearly audible thrill Loud murmur associated with thrill Moderately loud murmur without thrill

Loud murmur associated with thrill Grade 4 indicates loud murmurs with an associated thrill. A thrill is a fine vibration that is felt by palpation. A grade 5 intensity is characterized by an easily palpable thrill. A grade 2 intensity is characterized by quiet and clearly audible murmurs. A moderately loud murmur without a thrill is noted as grade 3.

While undergoing a soapsuds enema, the client reports abdominal cramping. What action should the nurse take? 1 Immediately stop the infusion. 2 Lower the height of the enema bag. 3 Advance the enema tubing 2 to 3 inches (5 to 7.5 cm). 4 Clamp the tube for 2 minutes and then restart the infusion.

Lower the height of the enema bag. Abdominal cramping during a soapsuds enema may be due to too rapid administration of the enema solution. Lowering the height of the enema bag slows the flow and allows the bowel time to adapt to the distention without causing excessive discomfort. Stopping the infusion is not necessary. Advancing the enema tubing is not appropriate. Clamping the tube for several minutes and then restarting the infusion may be attempted if slowing the infusion does not relieve the cramps.

A senior high school student, whose immunization status is current, asks the school nurse which immunizations will be included in the precollege physical. Which vaccine should the nurse tell the student to expect to receive? Hepatitis C (HepC) Influenza type B (HIB) Measles, mumps, rubella (MMR) Diphtheria, tetanus, pertussis (DTaP)

Measles, mumps, rubella (MMR) Individuals born after 1957 should receive one additional dose of MMR vaccine if they are students in postsecondary educational institutions. Currently there is no vaccine for hepatitis C. The HIB immunization is unnecessary. If the student received an additional DTaP at age 12, it is not necessary. A booster dose of tetanus toxoid (Td) should be received every 10 years.

After gastric surgery a client has a nasogastric tube in place. What should the nurse do when caring for this client? Monitor for signs of electrolyte imbalance. Change the tube at least once every 48 hours. Connect the nasogastric tube to high continuous suction. Assess placement by injecting 10 mL of water into the tube.

Monitor for signs of electrolyte imbalance.

After recovery from a modified neck dissection for oropharyngeal cancer, the client receives external radiation to the operative site. For which most critical reaction to the radiation should the nurse assess the client? Multiple choice question Dry mouth Skin reactions Mucosal edema Bone marrow suppression

Mucosal edema The mucosal lining of the oral cavity, oropharynx, and esophagus is sensitive to the effects of radiation therapy; the inflammatory response causes mucosal edema that may progress to an airway obstruction. A decrease in salivary secretions resulting in dry mouth may interfere with nutritional intake, but it is not life threatening. Erythema of the skin may cause dry or wet desquamation, but it is not life threatening. Radiation to the neck area should not produce as significant bone marrow suppression as radiation to the other sites.

A nurse is assessing a client's nails and finds a slight convex curve at the angle from the skin to nail base of about 160 degrees. Which condition does the nurse suspect? Multiple choice question Clubbing Paronychia Koilonychia Normal finding

Normal finding The client's nail, which has a slight convex curve at the angle from the skin to nail base of about 160 degrees, is normal. In clubbing, there is a change in the angle between the nail and the nail base that is larger than 180 degrees. Paronychia is the inflammation of the skin at the base of nail. Koilonychia is the concave curves on the nail.

The nurse has gathered data on a newly admitted client and is attempting to write the nursing diagnoses and develop a plan of care. What should the nurse be aware of when using the problem-etiology-signs and symptoms (PES) format? 1 Signs and symptoms come last in the diagnostic process. 2 Nursing interventions are derived from the etiology statement. 3 The only allowable diagnoses are nursing diagnoses. 4 Nursing diagnoses deal only with actual or potential illness problems.

Nursing interventions are derived from the etiology The etiology, or cause, of the problem provides direction for selection of nursing interventions. It is important to remember that gathering the signs and symptoms, or "S" in the acronym, comes first in the diagnostic process, even though the format is described as PES. Collaborative problems are potential or actual complications, diseases, or treatment that nurses handle most commonly with other healthcare providers. A wellness diagnosis may be identified when an individual is in transition from a specific level of wellness to a higher level of wellness. This diagnosis begins with "Readiness for enhanced," followed by the higher level of wellness desired.

What are the priority nursing interventions for a client with neutropenia in an emergency department? Select all that apply.

Obtain blood cultures immediately Administer antibiotic sTAT as prescribed --Monitor for pruritis and rashes & teach the client hygiene AFTER they have been stabilized; not a priority

Nurses care for clients in a variety of age groups. In which age group is the occurrence of chronic illness the greatest? Multiple choice question Older adults Adolescents Young children Middle-aged adults

Older adults The incidence of chronic illness increases in older adults because of the multiple stresses of aging. Younger individuals have greater physiologic reserves, and chronic illnesses are not common.

A client expresses concern about being exposed to radiation therapy because it can cause cancer. What should the nurse emphasize when informing the client about exposure to radiation? 1 The dosage is kept at a minimum. 2 Only a small part of the body is irradiated. 3 The client's physical condition is not a risk factor. 4 Nutritional environment of the affected cells is a risk factor.

Only a small part of the body is irradiated. Current radiation therapy accurately targets malignant lesions with pinpoint precision, minimizing the detrimental effects of radiation to healthy tissue. The dose is not as significant as the extent of tissue being irradiated. When radiation therapy is prescribed, the healthcare provider takes into consideration the ability of the client to tolerate the therapy, determining that the benefit outweighs the risk. Nutritional environment of the affected cells does not influence radiation's effect.

A registered nurse (RN) must assess the body temperature of a client with a history of epilepsy. Which site for measuring temperature is contraindicated in this client? Multiple choice question Skin Axilla Oral cavity Temporal artery

Oral cavity The oral cavity is not a preferred site to measure the body temperature of a client with epilepsy, oral surgery, trauma, or shaking chills. Epileptic clients become rigid during seizures and any sudden seizure attack during temperature measurement poses the risk of breaking the thermometer in the mouth, lacerations, accidental mercury ingestion, and possibly aspirating the broken pieces. The skin, axilla, and temporal artery are sites that can be safely used to measure topical body temperature in an epileptic client.

Which assessing technique involves tapping a client's skin with the fingertips to cause vibrations in the underlying tissues? Multiple choice question Palpation Inspection Percussion Auscultation

Percussion Percussion is the process of tapping the body parts with the fingers or hands to determine the consistency and borders of the body organs. Palpation is the act of feeling with the hand by applying pressure to the body surface to determine the condition of the skin and underlying tissues. Inspection is the process of visual observation of the body during physical examination. Auscultation means to listen to the internal sounds of the body.

A client with internal bleeding is in the intensive care unit (ICU) for observation. At the change of shift an alarm sounds, indicating a decrease in blood pressure. What is the initial nursing action? Multiple choice question Perform an assessment of the client before resuming the change-of-shift report. Continue the change-of-shift report and include the decrease in blood pressure. Lower the diastolic pressure limits on the monitor during the change-of-shift report. Turn off the alarm temporarily and alert the oncoming nurse to the decrease in blood pressure.

Perform an assessment of the client before resuming the change-of-shift report. The cause of the alarm should be investigated and appropriate intervention instituted; after the client's needs are met, then other tasks can be performed. An alarm should never be ignored; the client's status takes priority over the change-of-shift report. The diastolic pressure limit has been prescribed by the primary healthcare provider and should not be changed for the convenience of the nurse. Alarms always should remain on; the alarm indicates that the client's blood pressure has decreased and immediate assessment is required.

A client who underwent thyroid surgery is unable to speak and communicate. The nurse initially uses closed-ended questions to assess the client's needs. Once the client is stable, the nurse provides a small white board for the client to write and communicate to others. Which critical thinking attitude has the nurse demonstrated? 1 Humility 2 Discipline 3 Risk taking 4 Perseverance

Perseverance Perseverance is finding effective solutions to problems by trying various approaches. A critical thinker would demonstrate perseverance by not becoming satisfied with the solution unless the solution is near perfect. Humility is accepting one's limitations. Discipline is effective management of time and resources. Risk taking is pushing oneself beyond the limits to find solutions to the problem.

What is the most important factor relative to a therapeutic nurse-client relationship when a nurse is caring for a client who is terminally ill? 1 Knowledge of the grieving process 2 Personal feelings about terminal illness 3 Recognition of the family's ability to cope 4 Previous experience with terminally ill clients

Personal feelings about terminal illness To be effective in a relationship with a client, the nurse must know and understand personal feelings about terminal illness and death. Knowledge alone is not enough to ensure an effective nurse-client relationship. Although the family is an important part of a client's support system, the client's feelings are more important to the relationship. Previous experiences can be positive or negative and will not guarantee an effective nurse-client relationship.

A client with a history of heart disease has been receiving a calcium channel blocker and morphine sulfate for pain from abdominal surgery. When getting the client out of bed, the nurse first should have the client sit on the edge of the bed with feet on the floor. What untoward client response can be prevented by this nursing action? Abdominal pain Respiratory distress Sudden hemorrhage Postural hypotension

Postural hypotension After administration of certain antihypertensives or opioids, a client's neurocirculatory reflexes may have some difficulty adjusting to the force of gravity when

Alternative therapy measures have become increasingly accepted within the past decade, especially in the relief of pain. Which methods qualify as alternative therapies for pain? Select all that apply. 1 Prayer 2 Hypnosis 3 Medication 4 Aromatherapy 5 Guided imagery

Prayer Hypnosis Aromatherapy Guided imagery Prayer is an alternative therapy that may relax the client and provide strength, solace, or acceptance. The relief of pain through hypnosis is based on suggestion; also, it focuses attention away from the pain. Some clients learn to hypnotize themselves. Aromatherapy can help relax and distract the individual and thus increase tolerance for pain, as well as relieve pain. Guided imagery can help relax and distract the individual and thus increase tolerance for pain, as well as relieve pain. Analgesics, both opioid and nonopioid, long have been part of the standard medical regimen for pain relief, so they are not considered an alternative therapy.

A client is being discharged from the hospital with an indwelling urinary catheter. The client asks about the best way to prevent infection and keep the catheter clean. Which would be appropriate for the nurse to include in the client teaching? 1 Once a day, clean the tubing with a mild soap and water, starting at the drainage bag and moving toward the insertion site. 2 After cleaning the catheter site, it is important to keep the foreskin pushed back for 30 minutes to ensure adequate drying. 3 Clean the insertion site daily using a solution of one part vinegar to two parts water. 4 Replace the drainage bag with a new bag once a week.

Replace the drainage bag with a new bag once a week. It is recommended to change the bag at least once a week. Once a day, the client should wash the first inches of the catheter, starting at the insertion site and moving outward. The foreskin should be pushed forward as soon as the foreskin has been cleaned and dried. The drainage bag, not the insertion site, should be cleaned with the vinegar and water solution.

A primary nurse completes a nursing assessment of all assigned clients and develops a care plan for each client. Which element of decision-making does the nurse execute in this situation? 1 Authority 2 Autonomy 3 Responsibility 4 Accountability

Responsibility Responsibility refers to duties and activities that an individual is employed to perform. Authority refers to the legitimate power to give commands and make final decisions specific to a given issue. Autonomy refers to freedom of choices and the responsibility for the choices. Accountability refers to individuals being answerable for their actions.

A client is admitted to the hospital with severe diarrhea, abdominal cramps, and vomiting after eating. These symptoms have lasted 5 days. Upon further assessment, the primary healthcare provider finds that the symptoms occurred after the client ate eggs, salad dressings, and sandwich fillings. Which food borne disease would be suspected in this client? Multiple choice question Listeriosis Shigellosis Salmonellosis Staphylococcus

Salmonellosis A client with salmonellosis will experience severe diarrhea, abdominal cramps, and vomiting; these symptoms last as long as 5 days after the intake of contaminated food. This disorder may be caused by Salmonella typhi or Salmonella paratyphi. The causative organism is usually present in such foods as eggs, salad dressings, and sandwich fillings. A client with listeriosis will experience severe diarrhea, fever, headache, pneumonia, meningitis, and endocarditis 3 to 21 days after infection. The symptoms of shigellosis range from cramps and diarrhea to a fatal dysentery that lasts for 3 to 14 days. Pain, vomiting, diarrhea, perspiration, headache, fever, and prostration lasting for 1 or 2 days are the symptoms of a Staphylococcus infection.

Which site is best used to inspect a client who is suspected to have jaundice? Multiple choice question Skin Palm Sclera Conjunctiva

Sclera The sclera is the best site to inspect for jaundice. Because the skin may become pale due to anemia or jaundice, a skin inspection is not recommended. The palms and conjunctiva are inspected to assess pallor.

In what position should the nurse place a client recovering from general anesthesia? 1 Supine 2 Side-lying 3 High Fowler 4 Trendelenburg Turning the client to the side promotes drainage of secretions and prevents aspiration, especially when the gag reflex is not intact. This position also brings the tongue forward, preventing it from occluding the airway when it is in the relaxed state. The risk for aspiration is increased when the supine position is assumed by a semi-alert client. High Fowler position may cause the neck to flex in a client who is not alert, interfering with respirations. Trendelenburg position is not used for a postoperative client, because it interferes with breathing.

Side-lying Turning the client to the side promotes drainage of secretions and prevents aspiration, especially when the gag reflex is not intact. This position also brings the tongue forward, preventing it from occluding the airway when it is in the relaxed state. The risk for aspiration is increased when the supine position is assumed by a semi-alert client. High Fowler position may cause the neck to flex in a client who is not alert, interfering with respirations. Trendelenburg position is not used for a postoperative client, because it interferes with breathing.

When teaching about aging, the nurse explains that older adults usually have what characteristic? Multiple choice question Inflexible attitudes Periods of confusion Slower reaction times Some senile dementia

Slower reaction times A decrease in neuromuscular function slows reaction time. The ability to be flexible has less to do with age than with character. Confusion is not necessarily a process of aging, but it occurs for various reasons such as multiple stresses, perceptual changes, or medication side effects. Most older adults do not have organic mental disease.

What are the principal ions found in the extracellular fluids? A. Sodium and chloride B. Potassium and phosphate C. Sodium and potassium D. Potassium and protein

Sodium and Chloride

When nurses are conducting health assessment interviews with older clients, what step should be included? Multiple choice question Leave a written questionnaire for clients to complete at their leisure. Ask family members rather than the client to supply the necessary information. Spend time in several short sessions to elicit more complete information from the clients. Keep referring to previous questions to ascertain that the information given by clients is correct.

Spend time in several short sessions to elicit more complete information from the clients. Spending time in several short sessions reduces client fatigue and compensates for a shortened attention span, which is common in the older adult. The questionnaire may never be completed if it is left for the client to complete at their leisure. Asking family members rather than the client to supply the necessary information is degrading to the client; the client should be asked initially and, if necessary, family can be asked to fill in details later. Constantly referring to previous questions may be overwhelming and create feelings of anger and resentment.

While caring for a client receiving blood transfusion care, the nurse notices that the client is having an acute hemolytic reaction. What is the priority nursing intervention in this situation?

Stop the blood transfusion immediately

Ten minutes after initiating a blood transfusion, a client reports lumbar pain. What is the next nursing action? Stop the transfusion. Obtain the vital signs. Assess the pain further. Increase the flow of normal saline.

Stop the transfusion. This is a sign of an acute hemolytic transfusion reaction, indicating that the recipient's blood is incompatible with the transfused blood; pain is caused by hemolysis, agglutination, and capillary plugging in the kidneys. Obtaining the vital signs and assessing the pain further are unsafe actions; more incompatible blood will be infused, increasing the severity of the transfusion reaction. Increasing the flow of normal saline is unsafe; the transfusion must be stopped first, and then normal saline should be infused to keep the line patent and to maintain blood volume.

A family has undergone the emotional transition of accepting a new generation of members into the family system. Which changes in the family's status are required to proceed developmentally? Select all that apply. 1 Taking on parental roles 2 Adjusting to a reduction in family size 3 Development of intimate peer relationships 4 Adjusting the marital system to make space for children 5 Realigning relationships to in-laws and grandchildren

Taking on parental roles Adjusting the marital system to make space for children A family with more young children undergoes an emotional transition of accepting a new generation of members. These changes include taking on parental roles and adjusting the marital system to make space for children to proceed developmentally. Adjusting to a reduction in family size is required for the family life-cycle stage of children leaving the family home. The development of intimate peer relationships is required for an unattached young adult. Realigning relationships to in-laws and grandchildren is required for the family life-cycle stage of children leaving the home to start their own lives.

Which assessment should the nurse exclude when dealing with a client with receptive and expressive aphasia? Multiple choice question Ask the client to read simple sentences aloud Point to a familiar object and ask the client to name it Test the mental status by asking for feedback from the client Ask the client to respond to simple verbal commands such as "Stand up"

Test the mental status by asking for feedback from the client Receptive and expressive aphasia are the two types of aphasia. A client with receptive is unable to understand written or verbal speech. A client with expressive aphasia understands written and verbal speech but cannot write or speak appropriately. A client with aphasia may not have the mental ability to give feedback; asking for feedback is ineffective. Asking the client to read simple sentences aloud is an effective way of dealing with this client. Pointing to a familiar object and asking the client to name it is also effective. A client with aphasia can understand simple verbal commands.

A nursing student under the supervision of a registered nurse is performing a pulse assessment. While preparing to assess the client, the registered nurse asks the nursing student to check the apical pulse after assessing the radial pulse. What could be the reason behind for this change? Multiple choice question The client may have a dysrhythmia The client may have physiologic shock The client underwent surgery earlier in the day The cient may have peripheral artery disease

The client may have a dysrhythmia A client with dysrhythmia may have an intermittent or abnormal radial pulse. For this condition, the registered nurse should advise the nursing student to assess the apical pulse because it will be more accurate. If the client is in shock, then assessing the carotid or femoral pulse would be appropriate. The femoral pulse is preferred to assess a client with peripheral artery disease.

Which client is likely to have a health promotion nursing diagnosis? Multiple choice question The client with acute pain due to appendicitis. The client who is willing to take a 30-minute walk daily. The elderly client with dementia admitted to the healthcare facility. The client with reduced cognitive ability while recovering from surgery.

The client who is willing to take a 30-minute walk daily. A health promotion nursing diagnosis is a clinical judgment of an individual's desire to increase well-being. A client who is willing to take a 30-minute walk daily is expressing a desire to improve health behavior. The nurse identifies a health promotion nursing diagnosis for this client. Acute pain due to appendicitis is an actual nursing diagnosis. The nurse selects an actual nursing diagnosis when there is sufficient assessment data to establish the nursing diagnosis. It describes the client's response to a particular health condition. A risk nursing diagnosis describes an individual's response to health conditions that may develop in a vulnerable individual. The elderly client with dementia may have a risk nursing diagnosis for confusion. The client recovering from surgery has reduced cognitive ability and may have a risk nursing diagnosis for confusion or falls

At the beginning of the shift at 7:00 am, a client has 650 mL of normal saline solution left in the intravenous bag, which is infusing at 125 mL/hr. At 9:30 am the healthcare provider changes the IV solution to lactated Ringer solution, which is to infuse at 100 mL/hr. What total amount of intravenous solution should the client have received by the end of the 8-hour shift? Record your answer using a whole number. ___ mL

The client will have absorbed 313 mL of solution before the healthcare provider changes the prescription (2.5 hours × 125 mL/hour = 312.5, rounded up to 313); for the remaining 5.5 hours of the shift, the client will have received 550 mL (5.5 hours × 100 mL/hour), for a total of 863 mL.

Which feature is characteristic of a risk nursing diagnosis? Multiple choice question The diagnosis does not have related factors. The diagnosis can be used in any health state. The defining characteristics support the diagnostic judgment. The defining characteristics are supported by a client's readiness.

The diagnosis does not have related factors. A risk nursing diagnosis describes human responses to health conditions that may develop in a vulnerable individual, family, or community. Risk diagnoses do not have related factors or defining characteristics because they have not occurred yet. A risk diagnosis has risk factors that help the nurse plan preventive measures. A health promotion nursing diagnosis can be applied to any individual with a desire to enhance health behaviors in any health state. An actual diagnosis is formed when the defining characteristics support the diagnostic judgment. There must be sufficient nursing assessment data to establish an actual diagnosis. A health promotion nursing diagnosis is a clinical judgment of an individual's readiness to increase well-being.

How does the nurse identify an illness as chronic? Multiple selection question The illness is reversible and often severe. The illness persists for longer than six months. The client may develop life threatening relapse. The symptoms are intense and appear abruptly. The illness affects the functioning of one or more systems.

The illness persists for longer than six months. The client may develop life threatening relapse. The illness affects the functioning of one or more systems. A chronic illness usually lasts longer than six months. The client with chronic illness often fluctuates between maximal functioning and serious health relapses that may be life threatening. The illness affects the functioning of one or more systems. A chronic illness is irreversible, whereas an acute illness is reversible and often much more severe than a chronic illness. The client with acute illness develops intense symptoms that appear abruptly and often subside after a relatively short period.

The nurse is developing a nursing diagnosis for a client after surgery. The nurse documents the "related to" factor as first time surgery. Which assessment activity enabled the nurse to derive this conclusion? Multiple choice question The nurse notes nonverbal signs of discomfort. The nurse observes the client's position in bed. The nurse asks the client to explain the surgery. The nurse asks the client to rate the severity of pain.

The nurse asks the client to explain the surgery. The nurse must assess the client's knowledge about the surgery to determine if the client is aware of the outcome of surgery. The nurse observes for nonverbal signs of discomfort because some clients may not state that they are in pain. The nurse observes the client's positioning in bed to determine any abnormal signs such as discomfort or pain. The nurse asks the client to rate the severity of pain to determine a nursing diagnosis of pain related to a surgical wound.

Which diagnosis made by the nurse is helpful in providing the right nursing interventions for the client? Multiple choice question The nurse understands that the client has pain due to a tracheostomy. The nurse identifies that the client is anxious about the cardiac catheterization. The nurse realizes that the client has diarrhea and needs the bedpan frequently. The nurse identifies that the client is not aware of perineal care and has impaired skin integrity.

The nurse identifies that the client is not aware of perineal care and has impaired skin integrity. The nurse observes that the client has impaired skin integrity due to lack of knowledge about perineal care. The nurse identifies the need for educating the client about perineal care. This nursing diagnosis is correct as it will help enhance the client's health outcomes. The nursing diagnosis should identify the problem caused by a treatment such as tracheostomy, not the treatment itself. A tracheostomy is a medical condition and should not be included in the nursing diagnosis. This client is likely to have pain following the trauma of the surgical incision. The nursing diagnosis should contain the client's response to the medical procedure rather than the medical procedure itself. The client is probably anxious due to lack of knowledge about the need for cardiac catheterization or the outcome of the procedure rather than the catheterization itself. A correct diagnosis helps the nurse put the client at ease by providing necessary teaching. The nurse should plan nursing interventions after identifying the client's problem. Therefore, the nurse should identify that the client has diarrhea due to food intolerance. This helps the nurse select appropriate interventions rather than just one intervention of offering bedpan.

The nurse who works in a birthing unit understands that newborns may have impaired thermoregulation. Which nursing interventions may help prevent heat loss in the newborns? Select all that apply. 1 The nurse keeps the newborn covered in warm blankets. 2 The nurse keeps the newborn under the radiant warmer. 3 The nurse places the newborn on the mother's abdomen. 4 The nurse measures the newborn's temperature regularly. 5 The nurse encourages the mother to feed the newborn well to maintain the fluid balance.

The nurse keeps the newborn covered in warm blankets. The nurse keeps the newborn under the radiant warmer. The nurse places the newborn on the mother's abdomen. Newborns have impaired thermoregulation due to immaturity of the body systems. Therefore, the nurse performs interventions to prevent heat loss in the newborn. Covering the newborn with warm blankets helps to prevent heat loss. The nurse keeps the newborn under the radiant warmer to help maintain the body temperature. Placing the newborn on the mother's abdomen helps to promote warmth through skin-to-skin contact. Regular measurement of temperature may help in assessing any significant change; however, it may not help prevent heat loss. Ensuring that the newborn is fed well does not help to prevent heat loss.

An elderly adult suffered an injury after falling down in the washroom. The primary healthcare provider performed a surgical procedure on the client and orders a blood transfusion. A family member of the client mentions that blood transfusions are not permitted in their community. What should the nurse do in order to handle the situation? The nurse should wait for the court's order to give blood to the client. The nurse should proceed with the transfusion in order to save the client's life. The nurse should inform the primary healthcare provider and not give blood to the client. The nurse should explain to the family member that the client needs this transfusion.

The nurse should inform the primary healthcare provider and not give blood to the client.

Which situation is an example of a protective touch that may elicit negative feelings in a client? 1 The nurse provides a back massage to a client. 2 The nurse obtains a blood sample for laboratory tests. 3 The nurse holds a client while assisting with ambulation. 4 The nurse withdraws from a client to escape the situation.

The nurse withdraws from a client to escape the situation. The nurse may use protective touch and withdraw from the client to escape a tension-filled situation. This type of protective touch may protect the nurse emotionally but elicit negative feelings in a client. The nurse enhances the self-esteem and improves the mental well-being of a client by providing a back massage. This intervention is called a caring touch or nonverbal communication. The nurse performs task-oriented touching while obtaining a blood sample for laboratory tests. The nurse provides a protective touch that protects the client while assisting with ambulation.

A nurse is reviewing a client's serum electrolyte laboratory report. What is a comparison between blood plasma and interstitial fluid? 1 They both contain the same kinds of ions. 2 Plasma exerts lower osmotic pressure than does interstitial fluid. 3 Plasma contains more of each kind of ion than does interstitial fluid. 4 Sodium is higher in plasma, whereas potassium is higher in interstitial fluid

They both contain the same kinds of ions. Blood plasma and interstitial fluid are both part of the extracellular fluid and are of the same ionic composition. The osmotic pressure is the same. The composition is the same. The main cation of both extracellular fluids is sodium.

A nurse is caring for a client with diarrhea. The nurse anticipates a decrease in which clinical indicator? Multiple choice question Pulse rate Tissue turgor Specific gravity Body temperature

Tissue turgor Skin elasticity will decrease because of a decrease in interstitial fluid. The pulse rate will increase to oxygenate the body's cells. Specific gravity will increase because of the greater concentration of waste particles in the decreased amount of urine. The temperature will increase, not decrease.

A nurse is caring for a client with diarrhea. The nurse anticipates a decrease in which clinical indicator? Pulse rate Tissue turgor Specific gravity Body temperature

Tissue turgor Skin elasticity will decrease because of a decrease in interstitial fluid. The pulse rate will increase to oxygenate the body's cells. Specific gravity will increase because of the greater concentration of waste particles in the decreased amount of urine. The temperature will increase, not decrease.

Furosemide

Treats fluid retention (edema) and high blood pressure. This medicine is a diuretic (water pill). Can take with food if med upsets client's stomach. This medicine may cause the following problems: --Low levels of minerals in your blood, such as potassium and sodium --Blood sugar level changes --Hearing problems

While assessing a client who sustained a road traffic accident, a nurse notices that the client is unable to clench his teeth. Which cranial nerve might have been affected? Multiple choice question Facial nerve Trochlear nerve Abducens nerve Trigeminal nerve

Trigeminal nerve The trigeminal nerve provides sensory innervation to the facial skin and motor innervation to the muscles of the jaw. A client with a damaged trigeminal nerve will be unable to clench his teeth. The facial nerve provides sensory and motor innervations for facial expressions. The trochlear nerve is involved in downward and inward eye movements. The abducens nerve helps in the eyeball's lateral movement.

Following assessment, a nurse documents auscultation of course rhonchi in the anterior upper lung fields bilaterally that clears with coughing. What would be the cause of these sounds? Multiple choice question Parietal pleura rubbing against visceral pleura Random, sudden reinflation of groups of alveoli Turbulence due to muscular spasm and fluid or mucus in the larger airways High-velocity airflow through severely narrowed or an obstructed airway

Turbulence due to muscular spasm and fluid or mucus in the larger airways Loud, low pitched, rumbling coarse sounds heard over the trachea and bronchi are due to turbulence caused by muscular spasm when fluid or mucous is present in the larger airways. Pleural rub produces a dry or grating quality sound, best heard in the lower portion of the anterior lateral lung. Random and sudden reinflation of groups of alveoli produces crackling sounds predominantly heard in the left and right lung bases. High-velocity airflow through severely narrowed or obstructed airways results in a wheezing sound heard all over the lung.

Which pulse site is used to perform Allen's test? Multiple choice question Ulnar Brachial Femoral Dorsalis pedis

Ulnar The ulnar pulse site is used to perform Allen's test. The brachial pulse site is used to assess the status of circulation to the lower arm and to auscultate blood pressure. The femoral site is used to assess the character of the pulse during physiological shock or cardiac arrest. The dorsalis pedis site is used to assess the status of circulation in the foot.

A client has a pressure ulcer that is full thickness with necrosis into the subcutaneous tissue down to the underlying fascia. The nurse should document the assessment finding as which stage of pressure ulcer? Stage I Stage II Stage III Unstageable

Unstageable A pressure ulcer with necrotic tissue is unstageable. The necrotic tissue must be removed before the wound can be staged. A stage I pressure ulcer is defined as an area of persistent redness with no break in skin integrity. A stage II pressure ulcer is a partial-thickness wound with skin loss involving the epidermis, dermis, or both; the ulcer is superficial and may present as an abrasion, blister, or shallow crater. A stage III pressure ulcer involves full thickness tissue loss with visible subcutaneous fat. Bone, tendon, and muscle are not exposed.

An intravenous (IV) solution of 1000 mL 5% dextrose in water is to be infused at 125 mL/hr to correct a client's fluid imbalance. The infusion set delivers 15 drops/mL. To ensure that the solution will infuse over an eight-hour period, at how many drops per minute should the nurse set the rate of flow? Record the answer using a whole number. ______ gtts/min

Use the following formula to solve the problem. Drops per minute = total volume in drops (total mL x drop factor)/Total time in minutes (hours x 60) Drops per minute = 1000 mL x 15/8 x 60 = 15,000/480 = 31.25 = 31 gtts/min

Incentive Spirometry

Used to improve inspiratory muscle action and to prevent or reverse atelectasis in a client with pneumonia. The client is instructed to exhale fully and then insert the mouthpiece. Inhaling the air before inserting the mouthpiece may cause harm to the client and therefore needs correction. After the process is completed, the volume of air inspired is recorded. A client with pneumonia is instructed to perform 10 breaths per session every hour while awake. Taking a long, slow, deep breath keeping the mouthpiece in place helps to improve inspiratory muscle action.

The nurse is caring for a client whose forehead feels warm to the touch. The nurse uses a thermometer and obtains the client's temperature. What is the nurse doing? Multiple choice question Validation Assessment Interpretation Documentation

Validation The nurse is validating the presence of fever in the client. Validation is the process of gathering more assessment data. It involves clarifying vague or unclear data. Assessment is the first step of the nursing process. It involves collecting information from the client and secondary sources. During interpretation, the nurse recognizes that further observations are needed to clarify information. Data documentation is the last part of a complete assessment. The nurse must document facts in a timely, thorough, and accurate manner to prevent information from getting lost.

The nurse performs a respiratory assessment and auscultates breath sounds that are high pitched, creaking, and accentuated on expiration. Which term best describes the findings? Multiple choice question Rhonchi Wheezes Pleural friction rub Bronchovesicular

Wheezes Wheezes are one of the most common breath sounds assessed and auscultated in clients with asthma and chronic obstructive pulmonary disease (COPD). Wheezes are produced as air flows through narrowed passageways. Rhonchi are coarse, rattling sounds similar to snoring and are usually caused by secretions in the bronchial airways. A pleural friction rub is an abrasive sound made by two acutely inflamed serous surfaces rubbing together during the respiratory cycle. Bronchovesicular sounds are intermediate between bronchial (upper) and vesicular (lower) breath sounds; they are normal when heard between the first and second intercostal spaces anteriorly and posteriorly between scapulae.

The nurse reviews the medical record of a client who is eligible to receive end-of-life care. What are the criteria for a client to receive this type of care? Select all that apply. When the client is nearing death When the expected death of the client is within 6 months When the client seeks no aggressive disease management When a family member has signed an informed consent form When the client has been issued a "do not resuscitate" order

When the expected death of the client is within 6 months When the client seeks no aggressive disease management When the client has been issued a "do not resuscitate" order

. Emphysema

alveoli are destroyed as a result of damaging exposure - scar tissue replaces stretchy elastin fibers, lose elastic recoil, lungs inflate, but don't deflate: barrel chest. Airways can also be obstructed (mucus).

Normal Hemoglobin Count

amount of hemoglobin in a volume of blood (how much is available for oxygen to latch onto). Male: 14.0 - 17.5 g/dL, Female: 12.3 - 15.3 g/dL

A client is receiving fresh frozen plasma (FFP). The nurse would expect to see improvement in which condition? Thrombocytopenia Oxygen deficiency Clotting factor deficiency Low hemoglobin

clotting factor deficiency

The client receives a prescription for tap water enemas until clear. The nurse is aware that no more than two enemas should be given at one time to prevent the occurrence of what? Hypercalcemia Hypocalcemia Hyperkalemia Hypokalemia

hypokalemia

Kussmaul breathing

is a deep and labored breathing pattern often associated with severe metabolic acidosis, particularly diabetic ketoacidosis (DKA) but also kidney failure.

A nurse assesses a client's serum electrolyte levels in the laboratory report. What electrolyte in intracellular fluid should the nurse consider most important? Sodium Calcium Chloride Potassium

potassium

The nurse assesses bilateral +4 peripheral edema while assessing a client with heart failure and peripheral vascular disease. What is the pathophysiological reason for the excessive edema? Shift of fluid into the interstitial spaces Weakening of the cell wall Increased intravascular compliance Increased intracellular fluid volume

shift of fluid into the interstitial spaces

Antitussive

suppresses cough. Example: codeine (affects respiratory center, decreases urge to cough)

Oxygen Toxicity

too much of a good thing (more than 50% O2 for longer than 48-72 hours). Cataracts, seizures, shortness of breath, blindness in premature babies put in incubators with high oxygen levels - damaged retinas.


Kaugnay na mga set ng pag-aaral

LWW - Ch. 58: Assessment & Management of Pt.'s with Breast Disorders

View Set

Landforms Exam 2 Possible Questions

View Set

Cancer (Presentation Questions) NRS210

View Set

Section 5.1 Quiz - Chemistry/Chemistry Honors

View Set

Chapter 13 Eye Diagnostic Procedures

View Set